You are on page 1of 107

Company Sample Papers

Before solving papers please go through


Data Interpretation ( PGA Book no – 10 )
Data Sufficiency ( PGA Book no – 9)
Syllogisms ( www.youtube.com/user/pgapune ) Syllogisms solved examples
Cube Cutting ( www.youtube.com/user/pgapune )
Puzzles ( www.youtube.com/user/pgapuzzles )
Verbal Ability Tests ( PGA Book no – 4 )
Reading Comprehension ( PGA Book no – 8 )
Quantitative Ability Tests ( PGA Book no – 1 )
Encrypting Algorithms / Mathematical Reasoning
( www.youtube.com/user/pgapune )
Pankaj Gandhi’s Academy
rd
3 Floor, Above Hotel Woodland, Near Sharda Center, Nal Stop, Pune. Contact No. : 8600972993, 9850424051.
I TEST PAPER – 1

1. There are 6561 balls out of them 1 is heavy. Find the min. no. of times the balls have to be weighed for finding out
the heavy ball.
(a) 8 (b) 16 (c) 32 (d) 12

2. If I walk with 30 miles/hr I reach 1 hour before and if i walk with 20 miles/hr I reach 1 hour late. Find the distance
between 2 points and the exact time of reaching destination is 11 am then find the speed with which it walks.
(a) 240miles and 12miles/hr (b) 120miles and 24 miles/hr
(c) 240miles and 24 miles/hr (d) 120miles and 12 miles/hr

3. When you reverse the digits of age of father you will get the age of son. One year ago the age of father was twice
that of son's age. What are the current ages of father and son?
(a) 73 & 37 (b) 67 & 76 (c) 87 & 78 (d) 72 & 27

4. In a class there are less than 500 students. When it is divided by 3 it gives a whole number. Similarly when it is
divided by 4, 5 or 7 gives a whole number. Find the no. of students in the class
(a) 140 (b) 420 (c) 210 (d) 60

5. A coffee seller has two types of coffee Brand A costing 5 bits per pound and Brand B costing 3 bits per pound. He
mixes two brands to get a 40 pound mixture. He sold this at 6 bits per pound. The seller gets a profit of 33½ percent.
How much he has used Brand A in the mixture?
(a) 30 pounds (b) 60 pounds (c) 75 pounds (d) 15pounds

Directions 6-10: In each question below are given three Statements followed by three Conclusions numbered I, II and
III. You have to take the given Statements to be true even if they seem to be at variance from commonly known facts.
Read all the conclusions and then decide which of the given Conclusions logically follows from the given Statements
disregarding commonly known facts.

6. Statements: Some cycles are busses. All cars are buses. Some buses are trains.
Conclusions:
I. All cares are cycles. II. Some trains are buses. III. Some trains are cars.
(a) None follows (b) Only I and II follow (c) Only l and III follow (d) Only II and III follow (e) None of these

7. Statements: All pencils are sticks. Some sticks are notes. All diaries are notes.
Conclusions:
I. Some notes are diaries. II. Some sticks are pencils. III. Some diaries are sticks.
(a) All follow (b) Only I follow (c) Only I and II follow (d) Only II follows (e) None of these

8. Statements: Some buds are leaves. No leaf is fruit. Some fruits are buds.
Conclusions:
I. Some fruits are leaves. II. All buds are fruits. III. Some leaves are buds.
(a) Only I or II follows (b) Only III follows (c) Only II follows (d) None follows (e) None of these

9. Statements: Some birds are animals. All animals are rivers. Some rivers are lions.
Conclusions:
I. Some lions are animals II. Some rivers are birds III. No animal is lion
(a) Only II follows (b) Only either I or III follows (c) I and II follows (d) Only either II or III follow (e) None of
these

10. Statements: All boxes are pans. Some boxes are jugs .Some jugs are glasses.
Conclusions:
I. Some glasses are boxes II. No glass is box III. Some jugs are pans IV. No jug is pan
(a) Only I and II follows (b) Either I or II and III follows
(c) Only III follows (d) Either I or II, and either III or IV follow (e) None of these

Directions 11-15: Use the following answer choices for the questions below.
(a) Statement 1 alone is sufficient but statement 2 alone is not sufficient to answer the question asked.
(b) Statement 2 alone is sufficient but statement 1 alone is not sufficient to answer the question asked.
(c) Both statements 1 and 2 together are sufficient to answer the question but neither statement is sufficient alone.

Pankaj Gandhi’s Academy


rd
3 Floor, Above Hotel Woodland, Near Sharda Center, Nal Stop, Pune. Contact No. : 8600972993, 9850424051.
(d) Each statement alone is sufficient to answer the question.
(e) Statements 1 and 2 are not sufficient to answer the question asked and additional data is needed to answer the
statements.

11. If the average size of 3 accounts is $1 million, is the smallest account less than $500,000?
1. The largest account is $1.3 million.
2. One of the accounts is $0.7 million. ANS: C

12. Is the product of x and y greater than 60?


1. The sum of x and y is greater than 60.
2. Each of the variables is greater than 2. ANS: C

13. What is the value of y?


1. y - 3 = 2 ANS: A
2. y2 = 25

14. What was the percent increase of Company A's stock between June 1 and June 30, 2000?
1. The stock gained $5 in value during June 2000.
2. The stock rose 12% during the first half of the month. ANS: E

15. Which company reported the larger dollar increase in earnings?


1. Company A reported that its earnings increased by 5%. ANS: E
2. Company B reported that its earnings increased by 7%.

16. Ramesh starting from a fixed point goes 15 km towards North and then after turning to his right he goes 15 km.
then he goes 10, 15 and 15 metres after turning to his left each time. How far is he from his starting point?
(a) 5 metres (b) 10 metres (c) 20 metres (d) 15 metres (e) Cannot be determined

17. Sonalika goes 12 km towards North from a fixed point and then she goes 8 km towards South from there. In the
end she goes 3 km towards east. How far and in what direction is she from her starting point?
(a) 7 km East (b) 5 km West (c) 7 km West (d) 5 km North-East (e) None of these

18. Sunita goes 30 km towards North from a fixed point, then after turning to her right she goes 15 km. After this she
goes 30 km after turning to her right. How far and in what direction is she from her starting point?
(a) 45 km, East (b) 15 km, East (c) 45 km, West (d) 45 Km, North (e) None of these

19. Kanchan goes 5 m towards east from a fixed point N and then 35 km after turning to her left. Again she goes 10
metres after turning to her right. After this she goes 35 m after turning to her right. How far is she from N?
(a) 40 m (b) At N (c) 10 m (d)15 m (e) None of these

20. Shri Prakash walked 40 metres facing towards North. From there he walked 50 metres after turning to his left.
After this he walked 40 metres after turning to his left. How far and in what direction is he now from his starting point?
(a) 40 m, North (b) 50 m, West (c) 10 m, East (d) 10 m, West (e) None of these

Directions 21 - 25: Read the following information carefully and answer the questions given it.
There are six persons A B C D E and F in a school. Each of the teachers teaches two subjects, one compulsory
subject and the other optional subject. D's optional subject was History while there others have it as compulsory
subject. E and F have Physics as one of their subjects. F's compulsory subject is Mathematics which is an optional
subject of both C and E. History and English are A's subjects but in terms of compulsory and optional subjects, they
are just reverse of those of D's. Chemistry is an optional subject of only one of them. The only female teacher in the
school has English as her compulsory subject.

21. What is C's compulsory subject?


(a) History (b) Physics (c) Chemistry (d) English (e) Mathematics

22. Who is a female member in the group?


(a) A (b) B (c) C (d) D (e) E

23. Which of the following has some compulsory and optional subjects as those of F's?
(a) D (b) B (c) A (d) C (e) None of these

Pankaj Gandhi’s Academy


rd
3 Floor, Above Hotel Woodland, Near Sharda Center, Nal Stop, Pune. Contact No. : 8600972993, 9850424051.
24. Disregarding which is the compulsory and which is the optional subject, who has the same two subject
combination as F?
(a) A (b) B (c) E (d) D (e) None of these

25. Which of the following groups has History as the compulsory subject?
(a) A,C,D (b) B,C,D (c) C,D (d) A,B,C (e) A,D

I TEST PAPER - 2
1. Joe's age, Joe's sister's age and Joe's fathers age sums up to a century. When son is as old as his father, Joe's
sister will be twice as old as now. When Joe is as old as his father then his father is twice as old as when his sister
was as old as her father
(a)Joe=20 sister=30 father=50 (b) Joe=25 sister=35 father=55
(c) Joe=15 sister=25 father=45 (d) Joe=10 sister=20 father=40

2. At 6'o clock, clock ticks 6 times. The time between first and last ticks was 30sec. How much time it takes at
12'o clock?
(a) 66 sec (b) 60sec (c) 120sec (d) 30sec

3. Three friends divided some bullets equally. After all of them shot 4 bullets the total no.of remaining bullets is equal
to that of one has after division. Find the original number divided.
(a) 18 (b) 28 (c) 20 (d) 25

4. A ship went on a voyage after 180 miles a plane started with 10 times speed that of the ship. Find the distance
when they meet from starting point.
(a) 150 (b) 200 (c) 100 (d) 300

5. A man sold two cows for Rs. 210 at a total profit of 5 %. He sold one cow at a loss of 10% and another at a profit of
10%. What is the price of each cow?
(a ) Rs.150 & Rs. 50 (b) Rs. 250 & Rs. 100 (c) Rs.125 and Rs.25 (d) Rs. 300 and Rs.100

Directions 6-10: are to be answered on the following data


A causes B or C, but not both F occurs only if B occurs D occurs if B or C occurs
E occurs only if C occurs J occurs only if E or F occurs D causes G, H or both
H occurs if E occurs G occurs if F occurs
6. If A occurs which of the following must occurs?
I. F and G II. E and H III. D
(a) I only (b) II only (c) III only (d) I,II & III (e) I & II (or) II & III but not both

7. If B occurs which must occur


(a) D (b) D and G (c) G and H (d) F and G (e) J

8. If J occurs which must have occurred


(a) E (b) either B or C (c) both E & F (d) B (e) both B & C

9. Which may occurs as a result of cause not mentioned


I. D II. A III. F
(a) I only (b) II only (c) I & II (d) II & III (e) 1,11 & III

10. E occurs which one cannot occurs


(a) A (b) F (c) D (d) C

Directions 11-15: Each problem consists of a problem followed by two statements. Decide whether the data in the
statements are sufficient to answer the question. Select your answer according to whether
(A) Statement 1 alone is sufficient, but statement 2 alone is not sufficient to answer the question
(B) Statement 2 alone is sufficient, but statement 1 alone is not sufficient to answer the question
(C) Both statements taken together are sufficient to answer the question, but neither statement alone is sufficient
(D) Each statement alone is sufficient
(E) Statements 1 and 2 together are not sufficient, and additional data is needed to answer the question
11. Is x > 0?
1. -2x < 0 2. x3 > 0
(a) A (b) B (c) C (d) D (e) E

Pankaj Gandhi’s Academy


rd
3 Floor, Above Hotel Woodland, Near Sharda Center, Nal Stop, Pune. Contact No. : 8600972993, 9850424051.
12. A certain straight corridor has four doors, A, B, C and D (in that order) leading off from the same side. How far
apart are doors B and C?
1. The distance between doors B and D is 10 meters. 2. The distance between A and C is 12 meters.
(a) A (b) B (c) C (d) D (e) E

13. Given that x and y are real numbers, what is the value of x + y?
1. (x2 - y2) / (x-y) = 7 2. (x + y)2 = 49
(a) A (b) B (c) C (d) D (e) E

14. Two socks are to be picked at random from a drawer containing only black and white socks. What is the
probability that both are white?
1. The probability of the first sock being black is 1/3. 2. There are 24 white socks in the drawer.
(a) A (b) B (c) C (d) D (e) E

15. A bucket was placed under a dripping tap which was dripping at a uniform rate. At what time was the bucket full?
1. The bucket was put in place at 2pm. 2. The bucket was half full at 6pm and three-quarters full at 8pm.
(a) A (b) B (c) C (d) D (e) E

Directions 16-20: Read the following information carefully and answer the questions given below it.
Rohit, Kunal ,Ashish and John are students of a school. Three of them stay far from the school and one near it. Two
study in class IV, one in class V and one in class VI. They study Hindi, mathematics, Social Science. One is good at
all the four subjects while another weak in all of these. Rohit stays far from the school and good at Mathematics only
while Kunal is weak in Mathematics only and stays close to the school. Neither of these two nor Ashish studies in
class VI. One who is good at all the subjects studies in class V.
16. Name of the boy who is good at all the subjects.
(a) Rohit (b) Kunal (c) Ashish (d) John

17. Name the boy who is weak in all the subjects


(a) Rohit (b) Kunal (c) Ashish (d) John

18. Which two boys are good at Hindi?


(a) Rohit and Kunal (b) Kunal and Ashish (c) Ashish and John (d) John and Rohit

19. Which two boys are good at Mathematics?


(a) Rohit and Ashish (b) Kuna! and Ashish (c) John and Ashish (d) Rohit and John

20. Other than Rohit and the boy good at all the subjects, who else stays far from the school?
(a) Rohit (b) Kunal (c) Ashish (d) John

Directions 21-25: The table above shows imports for three types of steel over a six month period. Use this
information to answer the following questions.

Pankaj Gandhi’s Academy


rd
3 Floor, Above Hotel Woodland, Near Sharda Center, Nal Stop, Pune. Contact No. : 8600972993, 9850424051.
21. Which month showed the largest decrease in total imports over the previous month?
(a) Feb (b) Mar (c) Apr (d) May (e) June

22. What was the total value of sheet steel (in $) imported over the 6 month period?
(a) 56,750 (b) 75,300 (c) 55,550 (d) 42,370 (e) 44,750

23. What was the percentage of scrap steel imported in the 6 month period?
(a) 37.5 (b) 35.2 (c) 36.1 (d) 31.2 (e) 38.3

24. What was the difference (in thousands of tons) between coil steel and sheet steel imports in the first 3 months of
the year?
(a) 10 (b) 16 (c) 18 (d) 19 (e) 20

25. What was the approximate ratio of sheet steel and coil steel imports in the first 3 months of the year?
(a) 11:9 (b) 8:9 (c) 7:11 (d) 3:8 (e) 7:4
I Sample paper - 3
Analytical & Logical reasoning :
1. A family I know has several children. Each boy in this family has as many sisters as brothers but each girl has twice
as many brothers as sisters. How many brothers and sisters are there?
(a) 4 boys and 3 girls (b) 3 boys and 4 girls (c) 5 boys and 4 girls (d) 4 boys and5 girls

2. In a soap company a soap is manufactured with 11 parts. For making one soap you will get 1 part as scrap. At the
end of the day u have 251 such scraps. From that how many soaps can be manufactured?
(a)25 (b)35 (c)30 (d)15

3. There is a 5 digit no. 3 pairs of sum is eleven each. Last digit is 3 times the first one. 3 rd digit is 3 less than the
th
second. 4 digit is 4 more than the second one. Find the digit.
(a)25629 (b)25296 (c)29256 (d)62925

4. Every day a cyclist meets a train at a particular crossing. The road is straight before the crossing and both are
travelling in the same direction. The cyclist travels with a speed of 10 kmph. One day the cyclist comes late by 25 min.
and meets the train 5km before the crossing. What is the speed of the train?
(a) 60 km/hr (b) 75km/hr (c) 45km/hr (d) 90kmm/hr

5. Two twins have certain peculiar characteristics. One of them always lies on Monday, Wednesday, and Friday. The
other always lies on Tuesdays, Thursdays and Saturdays. On the other days they tell the truth. You are given a
conversation.
Person A-- today is Sunday my name is Anil
Person B -- today is Tuesday, my name is Bill
what day is today?

Pankaj Gandhi’s Academy


rd
3 Floor, Above Hotel Woodland, Near Sharda Center, Nal Stop, Pune. Contact No. : 8600972993, 9850424051.
(a)Monday (b)Tuesday (c) Thursday (d) Friday

Directions 6-10: Each problem consists of a problem followed by two statements. Decide whether the data in the
statements are sufficient to answer the question. Select your answer according to whether:
(A) Statement 1 alone is sufficient, but statement 2 alone is not sufficient to answer the question
(B) Statement 2 alone is sufficient, but statement 1 alone is not sufficient to answer the question
(C) Both statements taken together are sufficient to answer the question, but neither statement alone is sufficient
(D) Each statement alone is sufficient
(E) Statements 1 and 2 together are not sufficient, and additional data is needed to answer the question
6. If x and y are both positive integers, how much greater is x than y?
x + y = 20
x=
(a) A (b)B (c) C (d) D (e) E

7. Fifty percent of the articles in a certain magazine are written by staff members. Sixty percent of the articles are on
current affairs. If 75 percent of the articles on current affairs are written by staff members with more than 5 years
experience of journalism, how many of the articles on current affairs are written by journalists with more than 5 years
experience?
20 articles are written by staff members. Of the articles on topics other than current affairs, 50 percent
are by staff members with less than 5 years experience.
(a) A (b) B (c) C (d) D (e) E

8. Is x y > 0?
x/y < 0
x+y<0
(a) A (b) B (c) C (d) D (e) E

9. One number, n, is selected at random from a set of 10 integers. What is the probability that n + 13 = 0?
The largest integer in the set is 13.
The arithmetic mean of the set is zero.
(a) A (b) B (c) C (d) D (e) E

10. Is w a whole number?


3w is an odd number.
2w is an even number.
(a) A (b)B (c) C (d) D (e) E

Directions (Question 11 to 13): Read the following information carefully and answer the questions given below it.
i) There are six friends A, B, C, D, E and F
ii) Each one is proficient in one of the games, namely Badminton, Vollyball, Cricket, Hockey, Tennis and Polo
iii) Each owns a different colored car, namely yellow, green, black, white, blue and red.
iv) D plays Polo and owns a yellow colored car
v) C does not play either Tennis or Hockey and owns neither blue nor yellow colored car
vi) E owns a white car and plays Badminton
vii) B does not play Tennis; he owns a red coloured car.
viii) A plays Cricket and owns a black car
11. Who plays Volleyball?
(a) B (b) C (c) F (d) Data inadequate (e) None of these

12. Which colored car F owns?


(a) Green (b) Blue (c) Either Green or Blue (d) Data inadequate (e) None of these

13. Which of the following combinations of color of car and game played is not correct?
(a) Yellow-Polo (b) Green-Tennis (c) Black - Cricket (d) Red-Hockey (e) None of these

14. In a group of six women, there are four dancers, four vocal musicians, one actress and three violinists. Girija and
Vanaja are among the violinists while Jalaja and Shailaja do not know how to play on the violin. Shailaja and Tanuja
are among the dancers. Jalaja, Vanaja, Shailaja and Tanuja are all vocal musicians and two of them are also violinists
.If Pooja is an actress, who among the following is both a dancer and violinist?
(a) Jalaja (b) Shailaja (c) Tanuja (d) Pooja

Pankaj Gandhi’s Academy


rd
3 Floor, Above Hotel Woodland, Near Sharda Center, Nal Stop, Pune. Contact No. : 8600972993, 9850424051.
15. Salay walked 10 m towards West from his house. Then he walked 5 m turning to his left. After this he walked 10 m
turning to his left and in the end he walked 10 m turning to his left. In what direction is he now from his starting point?
(a) South (b) North (c) East (d) West (e) None of these

16. Manish goes 7 km towards South-East from his house, then he goes 14 km turning to West. After this he goes 7
km towards North West and in the end he goes 9 km towards East. How far is he from his house?
(a) 5 km (b) 7 km (c) 2 km (d) 14 km (e) None of these

17. Laxman went 15 km from my house, then turned left and walked 20 km. He then turned east and walked 25 km
and finally turning left covered 20kms. How far was he from his house?
(a) 5 km (b) 10 km (c)40km (d) 80 km (e) None of these

18. The door of Aditya's house faces the east. From the back side of his house, he walks straight 50 meters, then
turns to the right and walks 50 meters, then turns towards left and stops after walking 25 meters. Now Aditya is in
which direction from the starting point?
(a) South-East (b) North-East (c) South-West (d) North-West (e) None of
these

19. P, Q, R and S are playing a game of carrom. P, R, and S, Q are partners. S is to the right of R who is facing west.
Then Q is facing?
(a) North (b) South (c) East (d) West (e) None of these

20. A clock is so placed that at 12 noon its minute hand points towards north-east. In which direction does its hour
hand point at 1.30 p.m?
(a) North (b) South (c) East (d) West (e) None of these

21. A man walks 30 metres towards South. Then, turning to his right, he walks 30 metres. Then turning to his left, he
walks 20 metres. Again he turns to his left and walks 30 metres. How far is he from his initial position?
(a) 20 metres (b) 30 metres (c) 60 metres (d) 80 metres (e) None of these

I TEST PAPER - 4
Directions for Questions 1-2: Answer the Questions based on the information given below Seven university cricket
players are to be honoured at a special luncheon. The players will be seated on the dais along one side of a single
rectangular table.
♦ A and G have to leave the luncheon early and must be seated at the extreme right end of the table, which is closest
to the exit.
♦ B will receive the Man of the Match award and must be in the center chair.
♦ C and D who are bitter rivals for the position of wicket keeper, dislike one another and should be seated as far apart
as possible.
♦ E and F are best friends and want to sit together.

1. Which of the following may not be seated at either end of the table?
(a) C (b) D (c) G (d) F

2. Which of the following pairs may not be seated together?


(a) E and A (b) B and D (c) C and F (d) G and D

Directions for Questions 3-4: Answer the Questions based on the information given below
A, B, C, D, E and F are a group of friends from a club. There are two housewives, one lecturer, one architect, one
accountant and one lawyer in the group. There are two married couples in the group. The lawyer is married to D who
is a housewife. No lady in the group is either an architect or an accountant. C, the accountant, is married to F who is a
lecturer. A is married to D and E is not a housewife.

3. What is the profession of E?


(a) Lawyer (b) Architect (c) Lecturer (d) Accountant

4. How many members of the group are male?


(a) 2 (b) 3 (c) 4 (d) None of these

Pankaj Gandhi’s Academy


rd
3 Floor, Above Hotel Woodland, Near Sharda Center, Nal Stop, Pune. Contact No. : 8600972993, 9850424051.
5. Three friends went to a restaurant. After dinner, they paid their bill and noticed a bowl of mints. Sita took 1/3 of the
mints, but returned 4. Fatima took 1/4 of what left but returned three. Eswari then took half of the remaining but
retuned 2. The bowl had only 17 mints left. How many mints were there originally in the bowl?
(a) 38 (b) 31 (c) 41 (d) None of these

Directions for questions 6 to 10: In the figures given below, C and D should follow the same logical relation as A
and B. Find the appropriate picture from the list in the answer figures that can take the place of D (?)

6. Ans :3 7. Ans :1 8. Ans :2 9. Ans :2 10. Ans :2

Directions for questions 11 to 13:


The owner of the mansion has been murdered. The visitors of the mansion were Aditya, Vijay and Puneet. The
following additional information is also given:
(i) The murderer, who was one of the three visitors, arrived at the mansion, later than at least one of the other two
visitors.
(ii) A detective, who was one of the three visitors, arrived at the mansion earlier than at least one of the two visitors.
(iii) The detective arrived at the mansion at mid-night.
(iv) Neither Aditya nor Vijay arrived at the mansion after mid-night.
(v) The earlier arriver between Vijay and Puneet was not the detective.
(vi) The later arriver between Aditya and Puneet was not the murderer.

11. Who arrived at the mansion earliest?


(a) Puneet (b) Aditya (c) Vijay (d) Data insufficient

12. Who is the murderer?


(a) Puneet (b) Aditya (c) Vijay (d) Data insufficient

13. Who is the detective?


(a) Puneet (b) Vijay (c) Aditya (d) Data insufficient

Direction for Question 14-18:


The problems below contain a question and two statements giving certain data. You have to decide whether the data
given in the statements are sufficient for answering the questions. The answer is correct.
(1) If Statement I alone is sufficient but Statement II alone is not sufficient.
(2) If Statement II alone is sufficient but Statement I alone is not sufficient.
(3) If both statements I and II together are sufficient but neither of statements alone is sufficient.
(4) If each statement alone is sufficient.
(5) If Statement I and II together are not sufficient.

14. Ten children are sitting at a table. What is the average age of 10 children?
I. The average age of 6 youngest children is 12 years. II. The average age of 4 older children is 16 years. ANS : C

Pankaj Gandhi’s Academy


rd
3 Floor, Above Hotel Woodland, Near Sharda Center, Nal Stop, Pune. Contact No. : 8600972993, 9850424051.
15. What are the speeds of two trains 80 yards and 85 yards long respectively? Assume that former is faster than
later.
I They take 7.5 seconds to pass each other when running in the opposite direction.
II They take 37.5 seconds to pass each other when running in the same direction. ANS : C

16. What is the probability that 2 men and one woman chosen to form a committee are all Indians?
I. There are 4 men and 3 women in all. II. There are 3 Indians in all. ANS : C

17. How many right circular cylinders of equal size can be made from a metal of volume 200 cm3
I. The height of each cylinder is equal to its diameter. II The slant surface area of each cylinder is 45 sq.cm. ANS : C

18. A certain 4-litre solution of vinegar and water consists of x litre of vinegar and y liters of water. How man liters of
vinegar do the solution contain?
I. x/4=3/8 II. y/4=5/8 ANS : D

Directions for Question 19-24:


Based on the statement in the question, mark the most logical pair of statement that follow

19. If one succeeds. One makes merry.


(1) One did not succeed (2) One did not make merry (3) One succeeded (4) One made merry
(a) 2, 3 (b) 3, 2 (c) 1, 2 (d) 2, 1

20. If one gets lucky. One will win the prize.


(1) Ramesh did not get the prize (2) Ramesh got the prize (3) Ramesh didn’t get lucky (4) Ramesh got lucky
(a) 1,3 (b) 3,1 (c) 2,4 (d) None of these

21. Either he will shout or they will fire.


(1) He shouted. (2) He did not shout. (3) They fired (4) They did not fire
(a) 1,4 (b) 2,3 (c) 4,1 (d) Both (b) & (c)

Directions for Question 22-24:


Based on the statement in the question, mark the most logical Set among options

22. A. Pomfret is a fish. B. Pomfret is not a fish. C. Pomfret will not lay eggs.
D. Some fish lay eggs. E. All fish lay eggs. F. Pomfret may lay eggs.
(a) DFA (b) ADF (c) BDF (d) EBC

23. A. All copper is metal. B. All bronze is non-metal. C. Some metal is silver.
D. Some metal is not silver. E. No copper is bronze. F. Some silver is not metal.
(a) ABF (b) ACB (c) ABE (d) CDF

24. A. T may be A. B. A is T. C. T is G. D. Some G are A. E. G may be A. F. No A is T.


(a) CDA (b) BCF (c) DEF (d) ACB

Directions for Questions 25-28 :


The data in the table below gives the number and value of lost and found articles in City Women’s College for two
consecutive years 2003 and 2004. The data is based on the cases reported to the cases reported to the dean of the
college.
Articles Lost and Found 2003 2004
No. of Articles Value (in Rs) No. of Articles Value (in Rs)
Hanky 20 1950 10 950
Lipsticks 175 5750 120 12050
Perfume 155 7950 225 17350
Makeup Kit 1040 127050 860 10800
Talcum Powder 1135 34950 1070 24400
Hand Bag 355 8250 240 6350
All other items 1175 187150 975 93150

Pankaj Gandhi’s Academy


rd
3 Floor, Above Hotel Woodland, Near Sharda Center, Nal Stop, Pune. Contact No. : 8600972993, 9850424051.
25. By what percentage is average price per hanky in 2004 more or less than average price per hanky in 2003
(a) 5% (b) 2.5 % (c) 8% (d) Cannot be Determined

26. The difference between total number of items lost and found in both the years is
(a) 1435 (b) 1937 (c) 1235 (d) 1538

27. Which of the following articles can be considered to be most expensive in the year 2003?
(a) Handbag (b) Perfume (c) Make-up Kit (d) talcum Powder

28. How many Lost and Found articles are showing a decreasing trend both number wise and
value-wise from year 2003 to year 2004 in the above table?
(a) 3 (b) 4 (c) 5 (d) cannot be determined

Directions for questions 29-30:


A survey on a sample of 25 new cars being sold at a local auto dealer was conducted to see which of the three
popular options – air conditioning, radio and power windows – were already installed. The survey found:
♦ 15 had air conditioning
♦ 2 had air conditioning and power windows but no radios
♦ 12 had radio
♦ 6 had air conditioning and radio but no power windows
♦ 11 had power windows
♦ 1 had radio and power windows but no air conditioning
♦ 3 had all three options.

29. What is the number of cars that had none of the options?
(a) 4 (b) 3 (c) 1 (d) 2

30. How many cars have exactly one of the three options?
(a) 8 (b) 7 (c) 11 (d) None of these

I TEST PAPER - 5
Directions to Solve: (1-5)
In each of the following questions, find out which of the answer figures (1), (2), (3) and (4) completes the figure
matrix?
1. Select a suitable figure from the four alternatives that would complete the figure matrix.

(a) 1 (b) 2 (c) 3 (d) 4

2. Select a suitable figure from the four alternatives that would complete the figure matrix.

Pankaj Gandhi’s Academy


rd
3 Floor, Above Hotel Woodland, Near Sharda Center, Nal Stop, Pune. Contact No. : 8600972993, 9850424051.
(a) 1 (b) 2 (c) 3 (d) 4

3. Select a suitable figure from the four alternatives that would complete the figure matrix.

(a) 1 (b) 2 (c) 3 (d) 4

4. Select a suitable figure from the four alternatives that would complete the figure matrix.

(a) 1 (b) 2 (c) 3 (d) 4

5. Select a suitable figure from the four alternatives that would complete the figure matrix.

(a) 1 (b) 2 (c) 3 (d) 4

6.

Pankaj Gandhi’s Academy


rd
3 Floor, Above Hotel Woodland, Near Sharda Center, Nal Stop, Pune. Contact No. : 8600972993, 9850424051.
ANS : 6
7.

8.

ANS: Y

9.

10.

ANS : 4:15

Pankaj Gandhi’s Academy


rd
3 Floor, Above Hotel Woodland, Near Sharda Center, Nal Stop, Pune. Contact No. : 8600972993, 9850424051.
Directions to Solve: (11-15)
The following table gives the percentage of marks obtained by seven students in six different subjects in an
examination:
The Numbers in the brackets give the Maximum Marks in Each Subject.
Student Subject (Max. Marks)
Maths Chemistry Physics Geography History Computer
Science
(150) (130) (120) (100) (60) (40)
Ayush 90 50 90 60 70 80
Aman 100 80 80 40 80 70
Sajal 90 60 70 70 90 70
Rohit 80 65 80 80 60 60
Muskan 80 65 85 95 50 90
Tanvi 70 75 65 85 40 60
Tarun 65 35 50 77 80 80

11. What are the average marks obtained by all the seven students in Physics? (Rounded off to two digit after
decimal)
(a) 77.26 (b) 89.14 (c) 91.37 (d) 96.11

12. The number of students who obtained 60% and above marks in all subjects is?
(a) 1 (b) 2 (c) 3 (d) None

13. What was the aggregate of marks obtained by Sajal in all the six subjects?
(a) 409 (b) 419 (c) 429 (d) 449

14. In which subject is the overall percentage the best?


(a) Maths (b) Chemistry (c) Physics (d) History

15. What is the overall percentage of Tarun?


(a) 52.5% (b) 55% (c) 60% (d) 63%

Directions 16-20: On a rolling 6 dice, it is found that:


(i) Three of dice show the same number
(ii) Only one die shows 6
(iii) Not more than 3 dice show 4 or more.

16. What is the maximum possible total numbers on the faces, if the three dice having same number show 2?
(a) 14 (b) 22 (c) 11 (d) 9

17. What is the maximum total if 4 of the dice show less than 4?
(a) 28 (b) 31 (c) 17 (d) 22

18. What would be the maximum total if 3 dice are faulty and have only 5 on all faces?
(a) 30 (b) 25 (c) 34 (d) Not possible

19. If only 1 die shows 1, what is the maximum number of dice with number greater than 4?
(a) 3 (b) 1 (c) 2 (d) Cannot be determined

20. What is the max number that can be on the face of the 3 dice which shows the same number?
(a) 2 (b) 4 (c) 3 (d) 5

Directions 21-25: In each of the following questions two statements are given and these statements are followed by
two conclusions numbered (1) and (2). You have to take the given two statements to be true even if they seem to be
at variance from commonly known facts. Read the conclusions and then decide which of the given conclusions
logically follows from the two given statements, disregarding commonly known facts.

21. Statements: Some actors are singers. All the singers are dancers.
Conclusions: 1. Some actors are dancers. 2. No singer is actor.
(a) Only (1) conclusion follows (b) Only (2) conclusion follows (c) Either (1) or (2) follows
(d) Neither (1) nor (2) follows (e) Both (1) and (2) follow

Pankaj Gandhi’s Academy


rd
3 Floor, Above Hotel Woodland, Near Sharda Center, Nal Stop, Pune. Contact No. : 8600972993, 9850424051.
22. Statements: All the harmoniums are instruments. All the instruments are flutes.
Conclusions: 1. All the flutes are instruments. 2. All the harmoniums are flutes.
(a) Only (1) conclusion follows (b) Only (2) conclusion follows (c) Either (1) or (2) follows
(d) Neither (1) nor (2) follows (e) Both (1) and (2) follow

23. Statements: Some mangoes are yellow. Some tixo are mangoes.
Conclusions: 1. Some mangoes are green. 2. Tixo is a yellow.
(a) Only (1) conclusion follows (b) Only (2) conclusion follows (c) Either (1) or (2) follows
(d) Neither (1) nor (2) follows (e) Both (1) and (2) follow

24. Statements: Some ants are parrots. All the parrots are apples.
Conclusions: 1. All the apples are parrots. 2. Some ants are apples.
(a) Only (1) conclusion follows (b) Only (2) conclusion follows (c) Either (1) or (2) follows
(d) Neither (1) nor (2) follows (e) Both (1) and (2) follow

25. Statements: Some papers are pens. All the pencils are pens.
Conclusions: 1. Some pens are pencils. 2. Some pens are papers.
(a) Only (1) conclusion follows (b) Only (2) conclusion follows (c) Either (1) or (2) follows
(d) Neither (1) nor (2) follows (e) Both (1) and (2) follow

Directions 26-30: Each problem consists of three statements. Based on the first two statements, the third statement
may be true, false, or uncertain.

26. (i) A fruit basket contains more apples than lemons.


(ii) There are more lemons in the basket than there are oranges.
(iii) The basket contains more apples than oranges.
If the first two statements are true, the third statement is
(a) True (b) False (c) Uncertain

27. (i) The Shop and Save Grocery is south of Greenwood Pharmacy.
(ii) Rebecca's house is northeast of Greenwood Pharmacy.
(iii) Rebecca's house is west of the Shop and Save Grocery.
If the first two statements are true, the third statement is
(a) True (b) False (c) Uncertain

28. (i) Joe is younger than Kathy.


(ii) Mark was born after Joe.
(iii) Kathy is older than Mark.
If the first two statements are true, the third statement is
(a) True (b) False (c) Uncertain

29. (i) On the day the Barton triplets are born, Jenna weighs more than Jason.
(ii) Jason weighs less than Jasmine.
(iii) Of the three babies, Jasmine weighs the most.
If the first two statements are true, the third statement is
(a) True (b) False (c) Uncertain

30. (i) The temperature on Monday was lower than on Tuesday.


(ii) The temperature on Wednesday was lower than on Tuesday.
(iii) The temperature on Monday was higher than on Wednesday
If the first two statements are true, the third statement is
(a) True (b) False (c) Uncertain

Pankaj Gandhi’s Academy


rd
3 Floor, Above Hotel Woodland, Near Sharda Center, Nal Stop, Pune. Contact No. : 8600972993, 9850424051.
I TEST PAPER - 6
Directions 1-5: Read following data and answer questions.
(i) A, B, C, D, E & F are six sportsmen who plays following games - cricket, hockey, soccer, tennis, archery &
swimming.
(ii) Two of them belong to Kerala, two from Rajasthan & remaining two from Karnataka.
(ii) C is not from Rajasthan & plays soccer.
(iii) Sportsmen playing archery & hockey are not from Rajasthan.
(iv) E is a swimmer & D is a tennis player.
(v) F and D are from Kerala. E is from Rajasthan & B from Karnataka.
1. Who plays cricket?
(a) F (b) B (c) A (d) None of these

2. Which are the two players playing for the state of Karnataka?
(a) AE (b) CF (c) BC (d) None of these

3. Which sports does F play?


(a) Soccer (b) Archery (c) Hockey (d) data inadequate

4. Which of the following pair of sportsmen are each from Kerala and Karnataka?
(a)AE (b) FC (c) DF (d) None of these

5. Which of the combination of sport and state is not correct?


(a) A- Rajasthan (b) D - Karnataka (c) F-Kerala (d) None of these

Directions 6-10: In each of the following questions two statements are given and these statements are followed by
two conclusions numbered (I) and (II). You have to take the given two statements to be true even if they seem to be at
variance from commonly known facts. Read the conclusions and then decide which of the given conclusions logically
follows from the two given statements, disregarding commonly known facts.
6. Statements: All ants are elephants. Some elephants are parrots. Some parrots are crocodiles.
Conclusion: I. Some parrots are ants. II. Some ants are crocodiles.
(a) Only (a) follows (b) Both (a) & (b) follows (c) Only (b) follws (d) None of these

7. Statement: Pilots marry fair girls. Sahana is very fair.


Conclusion: I. Sahana marries a pilot. II. Sahana does not marry a pilot.
(a) Only (a) follows (b) Both (a) & (b) follows (c) Only (b) follows (d) None of these

8. Statement: All phones are black. Salma has a Nokia phone


Conclusion: I. All Nokia phones are black II. Salma’s phone is black in colour.
(a) Only (a) follows (b) Both (a) & (b) follows (c) Only (b) follows (d) None of these

9. Statement: Rain causes floods. Floods cause destruction.


Conclusion: I. Rain causes destruction. II. All floods are caused by rains.
(a) Only (a) follows (b) Both (a) & (b) follows (c) Only (b) follows (d) None of these

10. Statement: All cycles are cars. All cars are trucks. Some cycles are bullock-carts. .
Conclusion: I. Some bullock-carts are trucks. II. Some cars are bullock-carts.
(a) Only (a) follows (b) Both (a) & (b) follows (c) Only (b) follows (d) None of these

Directions 11-15: Study the table given below & answer the following questions which are based on students of
different classes participating in different activities. Also the ratio of number of boys and girls in each class is given.
VI VII VIII IX X Total
Class
Activity
Drama 8 4 6 2 4 24
Singing 12 8 10 14 16 60
Painting 16 12 10 12 8 58
Quiz 4 8 6 8 2 28
Dance 6 8 10 12 14 50
Total 46 40 42 48 44 220
Class VI VII VIII IX X
Ratio 15 : 6 5:3 4:3 3:5 5:6

Pankaj Gandhi’s Academy


rd
3 Floor, Above Hotel Woodland, Near Sharda Center, Nal Stop, Pune. Contact No. : 8600972993, 9850424051.
11. Which class has the highest percentage of students in Quiz?
(a) IX (b) VIII (c) VI (d) None of these

12. Which class has lowest percentage of students in Drama?


(a) VI (b) IX (c) VIII (d) None of these

13. If 40% of the boys in class X are in singing team find the number of girls in the singing team of class X.
(a) 8 (b) 6 (c) 10 (d) None of these

14. If 40% of the girls in class VII are in painting team, find the % percentage of boys in the same team.
(a) 10 (b) 16 (c) 20 (d) None of these

15. Which of the following statements is definitely true?


(a) The number of boys is more than number of girls in each class.
(b) In class VIII, all the teams have equal number of boys and girls.
(c) Quiz is the least popular game.
(d) Class IX has maximum number of girls.

16. A wall clock loses 10 minutes every 1 hour. In 1 hour as per the wall clock, a table clock gets 10 minutes ahead of
it. In 1 hour as per the table clock an alarm clock falls 5 minutes behind it. In 1 hour as per the alarm clock, a
wristwatch gets 5 minutes ahead it. At noon, all 4 timepieces were set correctly. To the nearest minutes, what time will
the wrist show when the correct time is 6 p.m. on the same day?
(a) 6 pm (b) 5:48 pm (c) 5:42 pm (d)6:30 pm

17. A mixture of 80 litres of milk and water contain 10% water how much water must be added to make water 20% in
the new mixture?
(a)10 (b)16 (c) 12 (d) 14

18. All the students of class are told to sit in circle shape. Here the boy at the 6th position is exactly opposite to 16th
boy. Total number of boys in the class?
(a)20 (b) 22 (c)21 (d)None of these

19. A company contracts to paint 3 houses. Mr. Brown can paint a house in 6 days while Mr. Black would take 8 days
and Mr. Blue 12 days. After 8 days Mr. Brown goes on vacation and Mr. Black begins to work for a period of 6 days.
How many days will it take Mr. Blue to complete the contract?
(a) 7 (b) 8 (c) 11 (d) 12

20. A goat is tied to one corner of a square plot of side 12m by a rope 7m long. Find the area it can graze?
(a) 38.5 sq.m (b) 155 sq.m (c) 144 sq.m (d) 19.25 sq.m

21. The average age of 10 members of a committee is the same as it was 4 years ago, because an old member has
been replaced by a young member. Find how much younger is the new member?
(a) 40 (b) 20 (c) 30 (d) 35

22. If the list price of a book is reduced by Rs. 5, then a person can buy 5 more books for Rs. 300. The original cost of
the book is
(a) Rs. 15 (b) Rs. 20 (c) Rs. 25 (d) Rs. 30

23. A digital wristwatch was set accurately at 8.30 a.m and then lost 2 seconds every 5 minutes. What time was
indicated on the watch at 6.30 p.m of the same day if the watch operated continuously that time ?
(a) 5:56 (b) 5:58 (c) 6.00 (d) 6.23 (e) 6.26

24. A train overtakes 2 persons walking at 3 km/hr and 5 km/hr respectively in the same direction and completely
passes them in 8 seconds and 10 seconds respectively. Find the speed of the train.
(a) 15 km/hr (b) 13 km/hr (c) 10 km/hr (d) 10 km/hr (e) None of these

25. Given following sequence, find the next term in the series:
3, 6, 13, 26, 33, 66, ___
(a)53 (b) 54 (c)63 (d) None of these

26. Find the 3 digit no. whose last digit is the square root of the first digit and second digit is the sum of the other two
digits.

Pankaj Gandhi’s Academy


rd
3 Floor, Above Hotel Woodland, Near Sharda Center, Nal Stop, Pune. Contact No. : 8600972993, 9850424051.
(a)462 (b)953 (c)786 (d) None of these

27. 85 are married, 80 have cars, 75 have phones, and 70 have houses. How many minimum persons are married,
have phone, car and houses on a scale of 100.
(a) 15 (b) 16 (c) 70 (d) None of these

28. A man can row three-quarters of a kilometre against the stream in 11 (1/4) minutes and down the stream in 7
(1/2)minutes. The speed (in km/hr) of the man in still water is:
(a) 2 (b) 3 (c) 4 (d)5

29. The speed of a boat in still water in 15 km/hr and the rate of current is 3 km/hr. The distance travelled downstream
in 12 minutes is:
(a) 1.2 km (b) 1.8 km (c) 2.4 km (d)3.6 km

I TEST PAPER - 7
1. A bag contains 3 red, 5 yellow and 4 green balls. 3 balls are drawn randomly. What is the probability that the balls
contain no yellow ball?
(a)7/44 (b)6/44 (c)7/22 (d)6/22
2. How many three digit numbers can be formed using 2,3,4 and 5 with none of the digits being repeated
(a)12 (b)24 (c)36 (d)48

3. White cube painted red on 2 faces. How many different ways can it be done (two paintings are same if on suitable
rotation, one painted can be carried to other)
(a)0 (b) 3 (c) 5 (d) None of these

4. When I add 4 times my age 4 years from now to 5 times my age 5 years from now ,I get 10 times my current age.
How old will I be 3 years from now?
(a)41 (b)44 (c)47 (d)50

5. What is the next number of the following sequence: 10, 1, 9, 2, 8, 3, 7, 4, 6, 5, 5, 6, 4, 7, 3, 8, 2,....


(a)11 (b)4 (c)1 (d)9

6. What is the next number of the following sequence: 2, 4, 16, 512,.....


(a)1024 (b)65536 (c)256 (d)128

7. What is the next number in the series: 3,32,405,?


(a)5832 (b)6144 (c)9876 (d)4324

8. There are two containers on a table. A and B. A is half full of wine, while B, which is twice A's size, is one quarter
full of wine. Both containers are filled with water and the contents are poured into a third container C. What portion of
container C's mixture is wine?
(a)25% (b)40% (c)33.33% (d)50%

9. In town of 500 people, 285 read Hindu and 212 read Indian express and 127 read Times of India 20 read Hindu and
times of India and 29 read Hindu and Indian express and 35 read times of India and Indian express. 50 read no news
paper. Then how many read only one paper?
(a)447 (b)383 (c)321 (d)45

10. Suppose 8 monkeys take 8 minutes to eat 8 bananas.How many minutes would it take 3 monkeys to eat 3
bananas?
(a)8 (b)3 (c)2 (d)8/3

Directions 11-15: A cube is coloured orange on one face, pink on the opposite face, brown on one face and silver on
a face adjacent to the brown face. The other two faces are left uncoloured. It is then cut into 125 smaller cubes of
equal size. Now, answer the following questions based on the above statements:
11. How many cubes have at least one face coloured pink ?
(a) 1 (b)9 (c) 16 (d) 25

12. How many cubes have all the faces uncoloured ?


(a) 24 (b)36 (c) 48 (d) 64

13. How many cubes have at least two faces coloured ?

Pankaj Gandhi’s Academy


rd
3 Floor, Above Hotel Woodland, Near Sharda Center, Nal Stop, Pune. Contact No. : 8600972993, 9850424051.
(a) 19 (b)20 (c) 21 (d) 23

14. How many cubes are coloured orange on one face and have the remaining faces incoloured ?
(a) 8 (b) 12 (c) 14 (d)7

15. How many cubes one coloured silver on one face, orange or pink on another face and have four uncoloured faces
?
(a) 8 (b) 10 (c) 12 (d) 16

Directions 16-20: In each of the following questions two statements are given and these statements are followed by
two conclusions numbered (I) and (II). You have to take the given two statements to be true even if they seem to be at
variance from commonly known facts. Read the conclusions and then decide which of the given conclusions logically
follows from the two given statements, disregarding commonly known facts.

16. Statements: I. All poles are guns. II. Some boats are not poles.
Conclusions : I. All guns are boats. II. Some boats are not guns
(a) Only I can be true always (b) Only II can be true always
(c) Both I and II can be true always (d) Both I and II cannot be true always

17. Statements: I. All dogs are jackals. II. Some jackals are crows.
Conclusions: I. Some dogs are crows. II. All dogs are crows.
(a) Only I can be true always (b) Only II can be true always
(c) Both I and II can be true always (d) Both I and II cannot be true always

18. Statements : I. Some calendars are sticks. II. No stick is flower


Conclusions : I. Some calendars are flowers. II. No calendar is flower.
(a) Only I can be true always (b) Only II can be true always
(c) Both I and II can be true always (d) Both I and II cannot be true always

19. Statements : I. Some phones are watches. II. All watches are guns.
Conclusions : I. All guns are watches. II. Some guns are phones.
(a) Only I can be true always (b) Only II can be true always
(c) Both I and II can be true always (d) Both I and II cannot be true always

20. Statements : I. All vegetables are green. II. Some greens are fruits.
Conclusions: I. Some fruits are vegetables. II. No fruits is vegetable.
(a) Only I can be true always (b) Only II can be true always
(c) Both I and II can be true always (d) Both I and II cannot be true always

Directions for Questions 21-25: Each question given below has a problem and two statements numbered I and II
giving certain information. You have to decide if the information given in the statements are sufficient for answering
the problem. Indicate your answer as
(a) if the data in statement I alone are sufficient to answer the question;
(b) if the data in statement II alone are sufficient to answer the question;
(c) if the data in either in I or II alone are sufficient to answer the question;
(d) if the data even in both the statements together are not sufficient to answer the question;
(e) if the data in both the statements together are needed;

21. Find the value of X? ANS: A


(a) X3+1 (b) X4+1=0

22. Who is shortest among P,Q,R & S?


(a) P is taller than Q. (b) R & Q are shorter than S. ANS: d

23. In an election there are A,B,C participants find the individual votes of each other ANS: d
(a) A has got 1500 more than B (b) C has got half more than what a has got

24. Is P height is less than 170cms? ANS: c


(a) If we add 5 P’s height become more than 170cms. (b) If we add 2 P’s height become less than 170cms.

25. X is a whole number


(a) X3+1=0 (b) X4-1=0 ANS: A

Pankaj Gandhi’s Academy


rd
3 Floor, Above Hotel Woodland, Near Sharda Center, Nal Stop, Pune. Contact No. : 8600972993, 9850424051.
26. The ratio between the speeds of two trains is 7 : 8. If the second train runs 400 kms in 4 hours, then the speed of
the first train is:
(a) 70 km/hr (b) 75 km/hr (c) 84 km/hr (d) 87.5 km/hr

27. In covering a distance of 30 km, Celine takes 2 hours more than Sam. If Celine doubles his speed, then he would
take 1 hour less than Sam. Celine's speed is:
(a)5 (b)6 (c)7.5 (d)9

28. It takes eight hours for a 600 km journey, if 120 km is done by train and the rest by car. It takes 20 minutes more, if
200 km is done by train and the rest by car. The ratio of the speed of the train to that of the cars is:
(a) 2 : 3 (b) 3 : 2 (c) 3 : 4 (d) 4 : 3

29. PQRS is a rectangle having area 50. RX=1/4SR. What is the area of PQXS?
(a) 37.5 (b) 43.75 (c) 12 (d) 3

30. If Jacob sold a watch costing Rs.400 to john at 15% profit and john sold the same to Sudhir at 15% profit, what is
the price paid by Sudhir?
(a) 460 (b) 510 (c) 529 (d) 560

I TEST PAPER - 8
Directions 1-5: In each question, a set of six statements is given, followed by four answer choices. Each of the
answer choices has a combination of three statements from the given set of six statements. You are required to
identify the answer choices in which the statements are logically related.
1. A. All cats are goats
B. All Goats are dogs.
C. No goats are cows
D. No goats are dogs
E. All Cows are dogs
F. All dogs are cows
(a) FAB (b) ABE (c) AFB (d) ABF

2. A. Some lids are nibs


B. All hooks are lids.
C. All hooks are nibs
D. No lid is a nib
E. No lid is a hook
F. No nib is hook
(a) EFD (b) BCA (c) DEA (d) CDA

3. A. All MBA’s are logical


B. Sudhir is rational.
C. Sudhir is a logical MBA
D. Sudhir is a man
E. Some men are MBA’s
F. All men are rational.
(a) DEC (b) EAF (c) BCF (d) FDB

4. A. Competitive examinations are tough to pass.


B. There is heavy competition in any field.
C. No student can pass MAT
D. Very few students can pass MAT.
E. MAT is a competitive examination
F. MAT is tough to pass
(a) AEF (b) ABC (c) DFB (d)CDE

5. A. All Pens are knives


B. All knives are spoons
C. No knives are pens
D. No knives are spoons.
E. All pens are spoons.
F. All spoons are pens.
(a) ABE (b) ABF (c) AFE (d) DBE

Pankaj Gandhi’s Academy


rd
3 Floor, Above Hotel Woodland, Near Sharda Center, Nal Stop, Pune. Contact No. : 8600972993, 9850424051.
Directions 6-10: From the given answer choices for the questions below select which statement is necessary to
answer the questions.
6. By selling an article what is the profit percent gained?
(i) 5% discount is given on list price. (ii) If discount is not given 20% profit would be gained.
(iii) The cost price of the article is Rs 5000/-
(a) Only (i) and 2 (b) Only (i) and (iii) (c) Only (ii) and (iii) (d) All (i), (ii) and (iii) together
(e) None of these

7. What is the present age of father?


(i) Difference in ages of father and son is 24 years. (ii) Present age of son is of present age of father.
(iii) After 6 years from now, the age of son will be of father’s age.
(a) Only (i) and (ii) (b) Only (ii) and (iii) (c) Only (i) and (iii) (d) All (i), (ii) and (iii) are together required
(e) Any two of the statements are required to answer the question.

8. What is the cost of fencing a rectangular plot?


(i) Length of the plot is twice its breadth. (ii) Area of the plot is 288 square metres.
(iii) Cost of fencing is Rs 4/- per metre.
(a) Only (ii) and (iii) (b) Only (i) and (iii) (c) Only (i) and (ii)
(d) All the three statements (i), (ii) and (iii) are required to answer the question
(e) Question cannot be answered even with all the three statements together

9. What is the length of the train?


(i) The train crosses a signal pole in 18 seconds. (ii) Train crosses a 200 m platform in 30 seconds.
(iii) Speed of the train is 60 kmph.
(a) Only (i) and (iii) (b) Only (ii) and (iii) (c) Only (iii) and (i) or (ii) only (d) Only (i) or (ii)
(e) None of these

10. What will be the compound interest earned on an amount of Rs 5000/- in two years?
(i) The simple interest on the same amount at the same rate of interest in five years is Rs 2000/-
(ii) The compound interest and the simple interest earned in one year are the same.
(iii) The amount becomes more than double on compound interest in 10 years.
(a) Only (i) and (ii) (b) Only (i) (c) Only (ii) and (iii) (d) Only (i) and (iii) (e) None of these

11. Two trains starting at same time, one from Bangalore to Mysore and other in opposite direction arrive at their
destination 1 hr and 4 hours respectively after passing each other. How much faster is one train from other?
(a) Twice (b) Thrice (c) 4 times (d) None of these

12. There are 6 volumes of books on a rack kept in order (i.e. vol.1, vol. 2 and so on ). Give the position after the
following changes were noticed. All books have been changed. Vol.5 was directly to the right of Vol.2. Vol.4 has Vol.6
to its left and both weren't at Vol.3's place. Vol.1 has Vol.3 on right and Vol.5 on left. An even numbered volume is at
Vol.5's place. Find the order in which the books are kept now.
(a) 2,5,6,4,1,3 (b) 5,1,3,2,4,6 (c) 2,5,1,3,6,4 (d) None of these

13. In how many ways a committee, consisting of 5 men and 6 women can be formed from 8 men and 10 women?
(a) 266 (b) 5040 (c) 11760 (d) 86400

14. There is a safe with a 5 digit No. The 4th digit is 4 greater than second digit, while 3rd digit is 3 less than 2nd digit.
The 1st digit is thrice the last digit. There are 3 pairs whose sum is 11. Find the number.
(a) 941823 (b) 65292 (c) 941853 (d) None of these

15. In a town 65% people watched the news on television, 40% read a newspaper and 25% read a
newspaper and watched the news on television also. What percent of the people neither watched the
news on television nor read a news paper ?
(a) 5 (b) 10 (c) 15 (d) 20

16. 3 men finish painting a wall in 8 days. Four boys do the same job in 7 days. In how many days will 2 men and 2
boys working together paint two such walls of the same size?
(a) 6 6/13 days (b) 3 3/13 days (c) 9 2/5 days (d) 12 12/13 days (e) None of these

Pankaj Gandhi’s Academy


rd
3 Floor, Above Hotel Woodland, Near Sharda Center, Nal Stop, Pune. Contact No. : 8600972993, 9850424051.
17. If an article with marked price of Rs.400 is sold at successive discount of 10%,25% and 15%,what is the price the
customer has to pay?
(a) 360 (b) 300 (c) 230 (d) 270

18. Alpha, Beta, gamma, delta and epsilon are friends and have birthdays on consecutive days though may not be in
order. Gamma is as many days old to Alpha as Beta is younger to Epsilon. Delta is two days older then Epsilon.
Gamma’s Birthday is on Wednesday. Tell when is Alpha's birthday?
(a) Friday (b) Saturday (c) Sunday (d) None of these

19. My rack contains 8 Red colour ties, 13 violate colour ties,10 Blue colour ties, 5 Pink colour ties, 4 green colour
ties. If electricity gone and I want at least two ties of same colour then how many ties I should take out from my rack?
(a) 7 ties (b) 6 ties (c) 4 ties (d) None of these

20. Harry is a friend of Axy and Amy. Two statements are given about them.
1. If one of Axy and Amy is oldest then another is youngest.
2. Either Harry is the oldest or Amy is the youngest.
Who is the oldest?
(a) Axy (b) Amy (c) Data inadequate (d) None of these

21. In a group of persons traveling in a bus, 6 persons can speak Tamil, 15 can speak Hindi and 6 can speak Gujarati.
In that group, none can speak any other language. If 2 persons in the group can speak two languages and one person
can speak all the three languages, then how many persons are there in the group ?
(a) 21 (b) 23 (c) 22 (d)24

Directions 22-25: Professor Kittredge’s literature seminar includes students with varied tastes in poetry. All those in
the seminar who enjoy the poetry of browing also enjoy the poetry of eliot. Those who enjoy the poetry of eliot despise
the poetry of coleridge.Some of those who enjoy the poetry of eliot also enjoy the poetry of Auden. All of those who
enjoy the poetry of Coleridge also enjoy the poetry of Donne. Some of those who enjoy the poetry of Auden also
despise the poetry of Coleridge. All of those who enjoy the poetry of Donne also enjoy the poetry of Frost.

22. Miss Garfield enjoys the poetry of Donne. Which of the following must be true?
(a) She may or may not enjoy the poetry of coleridge (b) She does not enjoy the poetry of Browing
(c) She does not enjoy the poetry of eliot (d) She enjoys the poetry of Coleridge

23. Mr.Huxtable enjoys the poetry of Browing. He may also enjoy any of the following poets except
(a) Auden (b) Coleridge (c) Donne (d) Eliot (e) Frost

24. Miss Inaguchi enjoys the poetry of Coleridge. Which of the following must be false?
(a) She does not enjoy the poetry of Auden (b) She enjoys the poetry of Donne
(c) She enjoys the poetry of Frost (d) She does not enjoy the poetry of Browning
(e) She may enjoy the poetry of Eliot

25. Based on the information provided, which of the following statements concerning the members of the seminar
must be true?
(a) All those who enjoy the poetry of eliot also enjoy the poetry of Browning.
(b) None of those who despise the poetry of Frost enjoy the poetry of Auden.
(c) Some of those who enjoy the poetry of Auden despise the poetry of Coleridge.
(d) None of those who enjoy the poetry of Browning despise the poetry of Donne.
(e) Some of those who enjoy the poetry of Frost despise the poetry of Donne.

26. There are two towers A and B. Their heights are 200ft and 150ft respectively and the foot of the towers are 250ft
apart. Two birds on top of each tower fly down with the same speed and meet at the same instant on the ground to
pick a grain. What is the distance between the foot of tower A and the grain?
(a) 90ft (b) 85 ft (c) 80ft (d) None of these

27. Grass in lawn grows equally thick and in a uniform rate. It takes 40 days for 40 cows and 60 days for 30 cows to
eat the whole of the grass. How many days does it take for 20 cows to do the same?
(a) 115 (b) 120 (c) 125 (d) None of these

28. Four tourists A, B, C, D and four languages English, German, French and Italian. They are not able to converse
among themselves in one language. Though A does not know English he can act as an interpreter between B and C.

Pankaj Gandhi’s Academy


rd
3 Floor, Above Hotel Woodland, Near Sharda Center, Nal Stop, Pune. Contact No. : 8600972993, 9850424051.
No one spoke both French and German. A knows German and was able to converse with D who doesn’t know a word
in German. Only one language was spoken by more than two persons. Each spoke two languages. Find who spoke
what.
(a) German, Italian (b) French, Italian (c) German, English (d) Italian, English

29. A man is going to a wedding party. He travels for 2 hrs when he gets a puncture. Changing tyres takes 10 mins.
The rest of the journey he travels at 30 miles/hr. He reaches 30 mins behind schedule. He thinks to himself that if the
puncture had occurred 30 miles later, he would have been only 15 mins late . Find the total distance travelled by the
man.
(a) 120 miles. (b) 130 miles (c) 100 miles (d) None of these

30. A grandma has many sons. Each son has as many sons as his brothers. What is her age if the product of the
number of her sons and grandsons plus number of her sons? (Age between 70 and 100).
(a) 81 (b) 80 (c) 78 (d) None of these

I TEST PAPER – 9

1. A person wants to meet a lawyer and as that lawyer is busy he asks him to come three days after the day before of
the day after tomorrow? On which day the lawyer asks the person to come if today is Sunday?
(a) Wednesday (b) Thursday (c) Sunday (d) None of these

2. A person is 80 years old in 490 and only 70 years old in 500 in which year is he born?
(a) 470 (b) 480 (c) 475 (d) None of these

3. A person says that their speed while going to a city was 10mph however while returning as there is no much traffic
they came with a speed of 15mph.What is their average speed?
(a) 14 mph (b) 10 mph (c) 12 mph (d) None of these

4. The boy goes to school everyday.On his way he reaches railway station when he has covered 1/3 of his journey & a
mill when he has covered 1/4 of his journey the time taken him to walk between railway station & mill is 5 mins.He
reaches railway station at 7.35am.When does he start from hous & when does he reach school?
(a) 7:15 to 8:15 (b) 7 to 8 (c) 7:15 to 8:30 (d) None of these

5. There are two stops between two stations X and Y. A bus starts from X and travels towards Y. It reaches stop 1 at
4.30 Pm when it covers 1/4 of its journey and reach stop 2 which is 1/3rd of the journey. Find the start time from
station X, if it takes 5 minutes to reach stop 2 from stop 1.
(a) 3.30pm (b) 4.00pm (c) 4.10pm (d) 3.04pm

6. There are 3 stations which are equidistant from each other, between two cities. A train travels at constant speed
starts from city 1 and reaches station 2 at 3.15pm. The time taken by the train to cross the distance between the
stations 2 and 3 is 15 minutes. Find at what time the train will reach city 1.
(a) 3.30 pm (b) 4.25 pm (c) 4.10 pm (d) 3.45 pm

7. A train starts from station 1 and travels towards station 4, it reaches station 2 at 2/3 of its journey and station 3 at
3/4 of its journey, the time taken to travel between station 2 and station 3 is 20 minutes and it reaches station 3 at
10.15pm. If it loss 10 minutes on each station, then at what time the train started from station 1 and when will reach
station 4 ?
(a) 7.15pm & 11.35pm (b) 7.15pm & 11.15pm (c) 6:50pm & 11:30pm (d) 8.25pm & 10.55pm

8. Two men start from opposite banks of a river. They meet 340 meters away from one of the banks on forward
journey. After that they meet at 170 meters from the other bank of the river on their backward journey. What will be the
width of the river (in meters)?
(a) 850 (b) 510 (c) 1700 (d) 1020

9. A bus P leaves from City 1 to City 2 and at the same time bus Q leaves from City 2 to City 1. They meet 720 km
away from City 1 and after reaching their destinations, both drivers halt for 2 hours. After that they start back and meet
400 km away from City 2 on their return journey. Find the ratio between the speeds of the two buses.
(a) 7:11 (b) 7:19 (c) 8:17 (d) 9:13

Pankaj Gandhi’s Academy


rd
3 Floor, Above Hotel Woodland, Near Sharda Center, Nal Stop, Pune. Contact No. : 8600972993, 9850424051.
10. There are two sets P and Q such that P contains numbers from 1 to 1000 and Q contains numbers from 1 to 1000.
A person X is allowed to select a random number from P and another person Y is allowed to select a random number
from Q. Find the probability of X getting number less than what Y has selected.
(a) 1/1000 (b) 999/1000 (c) 1/2000 (d) 999/2000

11. A man X selects a random number from 1 to 1000 and another man Y selects a random number from 1 to 1000.
What is the probability of Y getting number equal to what X has selected?
(a) 1/1000 (b) 2/1000 (c) 1/2000 (d) 2/2000

12. Three clocks were set to correct time. First one runs with the exact time. Second one loses one minute per day.
Third one gains one minute per day. After how many days will they show the same time?
(a)180 days (b)287 days & 12 hours (c)240 days (d)199 days

13. A salesman marks an item 60% above the cost price & offers 2 successive discounts of 25% & 15% on the
marked price. His profit is:
(a) 15% (b) 2% (c) 7.5% (d) 10 (e) None of these

14.Had it been sold at 55% loss, SP would have been Rs. 10.80. The C.P is:
(a) Rs. 26 (b)Rs. 28 (c)Rs. 36 (d) Rs. 24 (e) None of these

15.In a class, except 18 all are above 50 years.15 are below 50 years of age. How many people are there
(a) 30 (b) 33 (c) 36 (d) none of these.

16.A tree on first day grows 1/2 of its size second day, 1/3rd of its size on the previous day similarly than 1/4th and so
on. You have to calculate after how many days the tree will be 100 times of its original size.
(a) 198 (b) 200 (c) 195 (d) None of these

17. A+B+C+D=D+E+F+G=G+H+I=17 given D=4.Find value of G ?


(a) 5 (b) 1 (c) 2 (d) None of these

Directions 18-23:Each of the questions given below consists of a statement and / or a question and two statements
numbered I and II given below it. You have to decide whether the data provided in the statement(s) is / are sufficient to
answer the given question. Read both the statements and
(a) If the data in Statement I alone are sufficient to answer the question, while the data in Statement II alone are not
sufficient to answer the question.
(b) If the data in Statement II alone are sufficient to answer the question, while the data in Statement I alone are not
sufficient to answer the question.
(c) If the data either in Statement I or in Statement II alone are sufficient to answer the question.
(d) If the data even in both Statements I and II together are not sufficient to answer the question.
(e) If the data in both Statements I and II together are necessary to answer the question.

18. If q is an integer, then sqrt(p2 + q2) is an integer?


I. p2 + q2 is an integer II. p2 - 3q2 = 0 2 alone

19. Is p + q = zero?
I. pq < 0 II. p2 = q2. Both combined

20. Is a - b is greater than p - q?


I. a > p and b < q II. b = 7, q = 8, a = 14 and p = 12. 2 alone

21. Is the integer k is divisible by 40?


I. 8 is a factor of k II. 10 is a factor of K both combined

22. What is ( x2 / y2 ) + ( y2 / x2 ) ?
I. x/y + y/x = 6 II. x/y - y/x = 2 both combined
3
23. What is the probability that x - 8 = 0 when x is selected from a set of 8 integers?
I. The smallest number in the set is -11 II. The arithmetic mean of the set is 1/8. Both together also not sufficient

Directions 24-26: In each of the following questions two statements are given and these statements are followed by
two conclusions numbered (I) and (II). You have to take the given two statements to be true even if they seem to be at

Pankaj Gandhi’s Academy


rd
3 Floor, Above Hotel Woodland, Near Sharda Center, Nal Stop, Pune. Contact No. : 8600972993, 9850424051.
variance from commonly known facts. Read the conclusions and then decide which of the given conclusions logically
follows from the two given statements, disregarding commonly known facts.
24. Statement: Some watches are hens. All hens are wall clocks.
Conclusions:I. All watches are wall clocks II. Some wall clocks are hens.
(a) Only I (b) Only II
(c) Both I and II (d) Both I and II
25. Statements:All animals are monkeys. All monkeys are reptiles.
Conclusions:I. All reptiles are animals II. All animals are reptiles
(a) Only I (b) Only II
(c) Both I and II (d) Both I and II

26. Statements:Some Red Boxes are Green Boxes. All Red Boxes are Yellow Boxes.
Conclusions:I. Some Yellow Boxes are Green Boxes II. All Green Boxes are Red Boxes.
(a) Only I (b) Only II
(c) Both I and II (d) Both I and II

Directions 27-30: An artist has exactly seven paintings --- T,U,V,W,X,Y, and Z -- from which she must choose exactly
five to be in an exhibition. Any combination is acceptable provided it meets the following conditions:
(i) If T is chosen, X cannot be chosen
(ii) If U is chosen , Y must also be chosen
(iii) If V is chosen , X must also be chosen

27. Which one of following is an acceptable combination of paintings for inclusion in the exhibition?
(a) T,U,V,X,Y (b) T,U,V,Y,Z (c) T,W,X,Y,Z (d) U,V,W,Y,Z (e) U,V,W,Z,Y

28. If painting T is chosen to be among the paintings included in the exhibition which one of the following cannot be
chosen to be among the paintings included in the exhibition?
(a) U (b) V (c) W (d) Y (e) Z

29.Which one of the following substitutions can the artist always make without violating restrictions affecting the
combination of paintings given that the painting mentioned first was not, and the painting mentioned first was not, and
the painting mentioned second was, originally going to be chosen?
(a) T replaces V (b) U replaces Y (c) V replaces X (d) W replaces Y (e) Z replaces W

30. If the artist chooses painting V to be included among the paintings in the exhibit, which one of the following must
be true of that combination of paintings?
(a) T is not chosen (b) Y is not chosen (c) U is chosen (d) W is chosen (e) Z is chosen
I TEST PAPER - 10
1. If "football" is "cricket" ,"cricket" is "basketball" ,"basketball" is "volleyball","volleyball" is "khokho" and "khokho" is
cricket, which is not a ball game?
(a) Cricket (b) football (c) khokho (d) basketball

2. A and B can finish a piece of work in 20 days .B and C in 30 days and C and A in 40 days. In how many days will A
alone finish the job?
(a) 48 (b) 34 2/7 (c) 44 (d) 45

3. A man speaks the truth 3 out of 4 times. He throws a die and reports it to be a 6. What is the probability of it being a
6?
(a) 3/8 (b) 5/8 (c) 3/4 (d) None of these

4. Three clocks were set to true time and starts to run together, makes a beep sound at intervals of 8,10 and 12
minutes respectively. In 2 days, how many times do they will make the sound together?
(a) 12 (b) 15 (c) 25 (d) 30

5. The average mark of 10 students is 80%. Later it was found that for one student, instead of 60%, the recorded 90%,
by mistake. Now the corrected new percentage?
(a) 77 (b) 87 (c) 83 (d) None of these

Directions 6-10: The following questions are based on the information given below:
(i) A cuboid shaped wooden block has 6 cm length, 4 cm breadth and 1 cm height.
(ii) Two faces measuring 4 cm x 1 cm are coloured in black.

Pankaj Gandhi’s Academy


rd
3 Floor, Above Hotel Woodland, Near Sharda Center, Nal Stop, Pune. Contact No. : 8600972993, 9850424051.
(iii) Two faces measuring 6 cm x 1 cm are coloured in red.
(iv) Two faces measuring 6 cm x 4 cm are coloured in green.
(v) The block is divided into 6 equal cubes of side 1 cm (from 6 cm side), 4 equal cubes of side 1 cm(from 4 cm side).

6. How many cubes having red, green and black colours on at least one side of the cube will be formed ?
(a) 16 (b) 12 (c) 10 (d) 4

7. How many small cubes will be formed?


(a) 6 (b) 12 (c) 16 (d) 24

8. How many cubes will have 4 coloured sides and two non-coloured sides?
(a) 8 (b) 4 (c) 16 (d) 10

9. How many cubes will have green colour on two sides and rest of the four sides having no colour?
(a) 12 (b) 10 (c) 8 (d) 4

10. How many cubes will remain if the cubes having black and green coloured are removed ?
(a) 4 (b) 8 (c) 12 (d) 16

Directions 11-15: In each of the following questions two statements are given and these statements are followed by
two conclusions numbered (I) and (II). You have to take the given two statements to be true even if they seem to be at
variance from commonly known facts. Read the conclusions and then decide which of the given conclusions logically
follows from the two given statements, disregarding commonly known facts.

11. Statements: All Planets are moons.All Moons are Stars.


Conclusions: I. All Moons are Planets. II. All Planets are stars.
(a) Only Conclusion II follows (b) Only Conclusion I follows (c) Either I or II follows
(d) Neither I nor II follows (e) Both I and II follows.

12. Statements: All men are dogs . All dogs are cats
Conclusions: I. All men are cats. II. All cats are men
(a) Only Conclusion I follows (b) Only Conclusion II follows (c) Either I or II follows
(d) Neither I nor II follows (e) Both I and II follows.

13. Statements: Some players are singers . All singers are tall
Conclusions: I. Some players are tall II. All Players are tall
(a) Only Conclusion I follows (b) Only Conclusion II follows (c) Either I or II follows
(d) Neither I nor II follows (e) Both I and II follows.

14. Statements: All Coins are crows . Some crows are pens.
Conclusions: I. No pen is coin. II. Some coins are pens.
(a) Only Conclusion I follows (b) Only Conclusion II follows (c) Either I or II follows
(d) Neither I nor II follows (e) Both I and II follows.

15. Statements: All men are married .Some men are educated.
Conclusions: I. Some married are educated. II. Some educated are married.
(a) Only Conclusion I follows (b) Only Conclusion II follows (c) Either I or II follows
(d) Neither I nor II follows (e) Both I and II follows.

16. Daal is now being sold at Rs. 20 a kg. During last month its rate was Rs. 16 per kg. By how much percent should a
family reduce its consumption so as to keep the expenditure fixed?
(a) 20 % (b) 30 % (c) 40 % (d) None of these

17. Three types of tea a, b, c costs Rs. 95/kg, 100/kg and70/kg respectively. How many kgs of each should be
blended to produce 100 kg of mixture worth Rs.90/kg, given that the quantities of b and c are equal
(a)70,15,15 (b) 50, 25, 25 (c) 60, 20, 20 (d)40, 30, 30

18. Can you find out what day of the week was January 12, 1979?
(a) Saturday (b) Friday (c) Sunday (d)None of these

19. M men agree to purchase a gift for Rs. D. If three men drop out how much more will each have to contribute
towards the purchase of the gift?

Pankaj Gandhi’s Academy


rd
3 Floor, Above Hotel Woodland, Near Sharda Center, Nal Stop, Pune. Contact No. : 8600972993, 9850424051.
(a) D/ (M-3) (b) MD/3 (c) M/ (D-3) (d) 3D/ (M2-3M)

I TEST PAPER - 11
Directions 1-5: Each problem consists of a problem followed by two statements. Decide whether the data in the
statements are sufficient to answer the question. Select your answer according to whether:
(a) If the question can be answered by using one of the statement alone, but cannot be answered using the other
statement alone.
(b) If the question can be answered by using either statement alone.
(c) If the question can be answered by using both statements together,but cannot be answered using either statement
alone.
(d) If the question cannot be answered even by using both statements together.

1. A,B,C,D and E are sitting around a table on five chairs which are numbered 1,2,3,4, and 5.The highest aged
person sits on thehighest numbered chair and the least aged person sits on the lowest numbered chair.If their ages
are in A.P, find the age of C.
I. D is the oldest with the age 52 and he is sitting on chair 5.
II. The Common difference in the AP is 2 and C is sitting on chair 3. Ans: C

2. A Person is called tall if his height is greater than or equal to 6ft.How many of the five persons: A,B,C,D, E are tall?
I.True: One of the five persons height is 5ft. 9 inches and at least three of them possess the height greater than 6ft.
II.False: At least two of them have their height less than 6ft. Ans : C

3.Sukanta,Basisth,Stuti,Shikha,Mandar and Ritu are sitting around a circular table. Sukanta,Basisth and mandar are
males whilethe rest are females.Who are the neighbours of Sukanta?
I.Stuti does not want any male as her neighbour and shikha does not want to sit along the side of sukanta.
II.Mandar does not want any female as his neighbour and wants to sit to the left of Basisth. Ans : C

4.CAP,SIP,TAP,IPS,ECS are the top five coaching institutes in India.Which one is the oldest?
I.CAP is older than SIP and TAP but not older than IPS.
II.ECS is older than SIP and TAP but not older than CAP. Ans : C

5. A,B,C,D, E visits five different cities P,Q,R,S, T. Though not necessarily in the same order. A does not go to P;B
does notvisit Q, and so on. Which city is visited by D?
I.A and C visit the cities Q and S respectively,while B does not visit City T
II.D will visit either R or T. Ans : D

6. John had decided to divide his Rs.1000/- for his four children according to their ages. The elder child should be a
Rs.20/- extra for each than his younger child. What will be the share of Mahesh who is the youngest?
(a) 220 (b) 240 (c) 200 (d) None of these

7. A sum was put at SI at certain rate for 2 years. Had it been put at 3% higher rate, it would have fetched Rs.300
more. Find the Sum.
(a) 4500 (b) 5000 (c) 6000 (d) None of these

8. A train is to traverse between A &B. If it travels by 50Kmph it got late by 15 min. If it travels by 40 Kmph it got late
by 50 min. Find out the speed of the train.
(a) 60 kmph (b) 70 kmph (c) 28/5 kmph (d) None of these

9. If ‘Apple’ is called ‘Orange’, ‘Orange’ is called ‘Peach’, ‘Peach’ is called ‘Potato’, ‘Potato’ is called ‘Banana’,
‘Banana’ is called ‘Papaya’ and ‘Papaya’ is called ‘Guava’, which of the following grows underground ?
(a) Potato (b) Guava (c) Apple (d) Banana (e) None of these

10. In June a baseball team that played 60 games had won 30% of its game played. After a phenomenal winning
streak this team raised its average to 50%.How many games must the team have won in a row to attain this average?
(a) 12 (b) 20 (c) 24 (d) 30

11. Complete the series: 5, 20, 24, 6, 2, 8, ?


(a) 12 (b) 14 (c) 15 (d) None of these

12. There are two clocks one runs 1min/hrs faster and other 1min/hr slower when will the two clocks have time time
difference of 1 hr :
(a) 40 hrs (b) 30 hrs (c) 34 hrs (d) None of these

Pankaj Gandhi’s Academy


rd
3 Floor, Above Hotel Woodland, Near Sharda Center, Nal Stop, Pune. Contact No. : 8600972993, 9850424051.
13. Father's age is three years more than three times the son's age. After three years, father's age will be ten years
more than twice the son's age. What is the father's present age?
(a) 33 (b) 15 (c) 20 (d) None of these

14. Out of a total of 120 musicians in a club , 5% can play all the three instruments- Guitar, violin and Flute. It so
happens that the number of musicians who can play any two and only two of the above instruments is 30. The number
of musicians who can play the guitar alone is 40. What is the total number of those who can play violin alone or flute
alone ?
(a) 30 (b) 38 (c) 44 (d) 45

15. A boat covers a certain distance downstream in 1 hour, while it comes back in 1 hours. If the speed of the stream
be 3 kmph, what is the speed of the boat in still water?
(a) 12 kmph (b) 13 kmph (c) 14 kmph (d) 15 kmph (e) None of these

16. When Arthur is as old as his father Hailey is now, he shall be 5 times as old as his son Clarke is now. By then,
Clarke will be 8 times older than Arthur is now. The combined ages of Hailey and Arthur are 100 years. How old is
Clarke?
(a) 10 (b) 20 (c) 15 (d) None of these

17. A son and father goes for boating in river upstream . After rowing for 1 mile son notices the hat of his father falling
in the river. After 5 min. he tells his father that his hat has fallen. So they turn round and are able to pick the hat at the
point from where they began boating after 5min. Tell the speed of river?
(a) 4 miles/hr (a) 6 miles/hr (a) 5 miles/hr (d) None of these

18. If the price of cooking gas increases by 23%, what % of the consumption of gas is reduced by a hotel owner so
that the expenditure on gas remains the same?
(a) 18.7 (b) 18.5 (c) 18 (d) None of these

Directions 19-23: In each of the following questions two statements are given and these statements are followed by
two conclusions numbered (I) and (II). You have to take the given two statements to be true even if they seem to be at
variance from commonly known facts. Read the conclusions and then decide which of the given conclusions logically
follows from the two given statements, disregarding commonly known facts.
19. Statements:All cars are tables. Some children are tables
Conclusions:I. Some cars are children II. Some children are cars
(a) Only I (b) Only II
(c) Both I and II (d) None of these
20. Statements: All windows are needles.Some trees are windows
Conclusions:I. Some trees are needles II. Some trees are not needles
(a) Only I (b) Only II
(c) Both I and II (d) None of these

21.Statements:Some fools are intelligent.Some intelligent are great


Conclusions: I Some fools are great II. All great are intelligent
(a) Only I (b) Only II
(c) Both I and II (d) None of these

22. Statements: Some papers are files.Some files are pens


Conclusions:I. Some files are not pens II. Some pens are papers
(a) Only I (b) Only II
(c) Both I and II (d) None of these
23. Statements:Some nurses are nuns .Madhu is a nun
Conclusions: I. Some nuns are nurses II. Some nurses are not nuns
(a) Only I can be true (b) Only II can be true
(c) Both I and II (d) None of these

Directions 24-26:There are 6 members in a family K L M N O P. L is brother of O’s husband. M is sister of P N is


father of K and grandfather of P. There are 2 fathers, 3 brothers, and a mother.
24. Who is O’s husband?
(a) M (b) N (c) P (d) None of these

Pankaj Gandhi’s Academy


rd
3 Floor, Above Hotel Woodland, Near Sharda Center, Nal Stop, Pune. Contact No. : 8600972993, 9850424051.
25. How many males are there?
(a) 2 (b) 3 (c) 4 (d) None of these

26. Group of brothers?


(a) LM (b) MN (c) KP (d) None of t

I TEST PAPER- 12
1. A train ran at a certain speed for one hour and then due to a technical problem, it ran at 3/5th of its original speed
and reached the destination 2 hrs late. The driver said, had the problem occurred 50 miles later, we would have
gained 40 minutes. What is the distance between the two cities that the train travelled?
(a) 200 miles (b) 500 miles (c) 400 miles (d) None of these

2. Each one wearing hat of one person and riding the bicycle of another.A is wearing B's hat and riding C's cycle. Who
is riding A's cycle?
(a) B (b) C (c) Data inadequate (d) None of these

3. A farmer is telling his wife, if we sell 75 chickens, the chicken food will last for 10 more days. But if we buy 100
chickens as we say, then the chicken food will last for 15 days less. How many chicken they have?
(a) 300 (b) 250 (c) 200 (d) None of these

4. A warehouse had a square floor with area 10,000 sq.meters. A rectangular addition was built along one entire side
of the warehouse that increased the floor by one-half as much as the original floor. How many meters did the addition
extend beyond the original buildings ?
(a)10 (b) 20 (c) 50 (d) 200 (e) 500

Directions 6-10: Each problem consists of a problem followed by two statements. Decide whether the data in the
statements are sufficient to answer the question. Select your answer according to whether:
(a) statement 1 alone is sufficient, but statement 2 alone is not sufficient to answer the question
(b) statement 2 alone is sufficient, but statement 1 alone is not sufficient to answer the question
(c) both statements taken together are sufficient to answer the question, but neither statement alone is sufficient
(d) each statement alone is sufficient
(e) statements 1 and 2 together are not sufficient, and additional data is needed to answer the question

6. Every pupil in a school was given one ticket for a concert. The school was charged a total of $6000 for these
tickets, all of which were of equal value. What was the price of one ticket?
I. If the price of each ticket had been one dollar less, the total cost would have been 1,200 less. Ans : D
II. If the price of each ticket had been $2 more, the total bill would have increased by 40%.

7. What is the ratio of male to female officers in the police force in town T?
I. The number of female officers is 250 less than half the number of male officers. Ans : B
II. The number of female officers is 1/7 the number of male officers.

8. What is the value of n?


I. 3n + 2m = 18 Ans : B
II. n – m = 2n – (4 + m)

9. How long did it take Henry to drive to work last Wednesday? (He did not stop on the way). Ans : A
I. If he had driven twice as fast he would have taken 35 minutes.
II. His average speed was 30 miles per hour.

10. What is the slope of line l which passes through the origin of a rectangular coordinate system? Ans : D

I. The line does not intersect with the line y = x + 2.


II. The line passes through the point (3,3)

Directions 11 to 15 : Read the following information carefully and answer the questions given it.
(i) P, Q, R, S, T and U are six students procuring their Master's degree in six different subjects - English, History,
Philosophy, Physics, Statistics and Mathematics.
(ii) Two of them stay in hostel, two stay as paying guest (PG) and the remaining two stay at their home
(iii) R does not stay as PG and studies Philosophy
(iv) The students studying Statistics and History do not stay as PG

Pankaj Gandhi’s Academy


rd
3 Floor, Above Hotel Woodland, Near Sharda Center, Nal Stop, Pune. Contact No. : 8600972993, 9850424051.
(v) T studies Mathematics and S studies Physics
(vi) U and S stay in hostel. T stays as PG and Q stays at home

11. Who studies English?


(a) R (b) S (c) T (d) U (e) None of these

12. Which of the following combinations of subject and place of stay is not correct?
(a) English - Hostel (b) mathematics – PG (c) Philosophy –Home (d) Physics –Hostel (e) None of these

13. Which of the following combinations of place of stay is not correct?


(a) QR (b) SR (c) US (d) Data inadequate (e) None of these

14. Which subject does Q study?


(a) History (b) Statistics (c) History or Statistics (d) Data inadequate
(e) None of these

15. Which of the following pairs of students stay at home?


(a) PQ (b) QR (c) RS (d) ST (e) None of these

16. A man is facing south. He turns 1350 in the anticlockwise direction and then 180o in the clockwise direction. Which
direction is he facing now.?
(a) North-east (b) North-West (c) South-east (d) South-west

17. A rat runs 20' towards East and turns to right, runs 10' and turns to right, runs 9' and again turns to left, runs 12'
and finally turns to left and runs 6'. Now, which direction is the rat facing?
(a) East (b) West (c) North (d) South

18. A man is facing north-west. He turns 90o in the clockwise direction, then 180o in the anticlockwise direction and
then another 90o in the same direction. Which direction is he facing now?
(a) South (b) Southwest (c) West (d) South-east

19.Radha moves towards South-east a distance of 7 km, and then she moves towards west and travels a distance of
14 m. From here, she moves towards North-West a distance of 7 m. and finally she moves a distance of 4 m towards
East and stood at that point. How far is the starting point from where she stood?
(a) 3 m (b) 4 m (c) 10 m (d) 11 m

Directions 21-26: In each question below are given two Statements followed by four Conclusions numbered (a), (b)
and (c),(d). You have to take the given Statements to be true even if they seem to be at variance from commonly
known facts. Read all the conclusions and then decide which of the given Conclusions logically follows from the given
Statements disregarding commonly known facts.
21.Statements:No apple is an orange. All bananas are oranges.
Conclusions:
(a) All apples are oranges (b) Some apples are oranges (c) No apple is a banana (d) None of the above

22.Statements: All pens are elephants. Some elephants are cats.


Conclusions:
(a) Some pens are cats (b) No pens are cats (c) All pens are cats (d) None of the above

23.Statements: All shares are debentures. No debentures are deposits.


Conclusions:
(a) All shares are deposits(b) Some shares are deposits (c) No shares are deposits (d) None of the above

24.Statements: Many fathers are brothers. All brothers are priests.


Conclusions:
(a) No father is a priest (b) Many fathers are not priests (c) Many fathers are priests (d) Both (b) and (c)

25.Statements: Some pens are pencils. All pencils are costly.


Conclusions:
(a) No pens are costly (b) Some pens are costly (c) All pens are costly (d) None of the above

26. Statements:All the harmoniums are instruments. All the instruments are flutes.
Conclusions:

Pankaj Gandhi’s Academy


rd
3 Floor, Above Hotel Woodland, Near Sharda Center, Nal Stop, Pune. Contact No. : 8600972993, 9850424051.
(a) All the flutes are instruments (b) All the harmoniums are flutes
(c) Some instruments are harmoniums (d) No flute is harmoniums

27. There is an escalator and 2 persons move down it.A takes 50 steps and B takes 75 steps while the escalator is
moving down. Given that the time taken by A to take 1 step is equal to time taken by B to take 3 steps. Find the no. of
steps in the escalator while it is stationary.
(a) 100 (b) 120 (c) 80 (d) None of these

28. If 5/2 artists make 5/2 paintings using 5/2 canvases in 5/2 days then how many artists are required to make 25
paintings using 25 canvases in 25 days?
(a) 25 (b) 5/2 (c) 20 (d) None of these

Direction 28-29. If A,B,C,D,and E are 5 members of a family.4 of them give true statements:
(i) E is my mother in law
(ii) C is my son in law's brother
(iii) B is my father's brother
(iv) A is my brother's wife
28.Who made the second statement?
(a) B (b) A (c) E (d) None of these

29.What is D’s relation with B?


(a) Nephew (b) Cousin (c) Brother (d) None of these

30. Alex speaks truth only in the morning and lies in the afternoon, whereas John speaks truth only in the afternoon. A
says that B is Alex.
Is it morning or afternoon and who is A - Alex or John ?
(a) Afternoon.A is John. (b) Afternoon.A is Alex (c) Morning.B is John (d) None of these

I TEST PAPER - 13
1. A lady's age is between 50 and 70.each of her sons has many sons as his brothers. the sum of the sons and
grandsons in the lady's age. what is her age?
(a) 64. (b) 65 (c) 70 (d) None of these

2. Two persons have 64 and 20 barrels of wine. the first one pays tax 5 barrels and 40 frames. the second one pays 2
barrel and got back 40 frames. What is the value of each barrel and the tax paid on each barrel?
(a) 8 (b) 10 (c) 30 (d) None of these

3. There are 20 balls of green color, 20 balls of red color, 20 balls of blue color,20 of balck,20 of yellow. how many u
have to pick to guarantee 2pairs of same color?
(a) 12 (b) 14 (c) 16 (d) None of these

4. There is a five digit number


1 It has 2 prime digits (1 is not a prime number)
2 The third digit is the highest
3 The second digit is lowest
4 The first digit is 1 less that third digit and greater than the addition of 4th and 5th digit.
5 The fifth digit is between first and second and is half of the fourth.
5. The sum of 4-th and 5-th digit is less that first What is that no?
(a) 61642 (b) 71842 (c) 81542 (d) 71452

5. The sum of 4-th and 5-th digit is less that first What is that no?
(a) 61642 (b) 71842 (c) 81542 (d) 71452

Directions 6-10: Each problem consists of a problem followed by two statements. Decide whether the data in the
statements are sufficient to answer the question. Select your answer according to whether:
(a) Statement 1 alone is sufficient, but Statement 2 alone is not sufficient to answer the question
(b) Statement 2 alone is sufficient, but Statement 1 alone is not sufficient to answer the question
(c) Both statements taken together are sufficient to answer the question, but neither statement alone is sufficient
(d) Each statement alone is sufficient
(e) Statements 1 and 2 together are not sufficient, and additional data is needed to answer the question

Pankaj Gandhi’s Academy


rd
3 Floor, Above Hotel Woodland, Near Sharda Center, Nal Stop, Pune. Contact No. : 8600972993, 9850424051.
6. How many ewes (female sheep) in a flock of 50 sheep are black?
I. There are 10 rams (male sheep) in the flock.
II. Forty percent of the animals are black.
Ans: (e)

7. Is the length of a side of equilateral triangle E less than the length of a side of square F?
I. The perimeter of E and the perimeter of F are equal.
II. The ratio of the height of triangle E to the diagonal of square F is 2√3: 3√2.
Ans: (d)

8. If a and b are both positive, what percent of b is a?


I. a = 3/11
II. b/a = 20
Ans: (b)

9. A wheel of radius 2 meters is turning at a constant speed. How many revolutions does it make in time T?
I.T = 20 minutes.
II.The speed at which a point on the circumference of the wheel is moving is 3 meters per minute.
Ans: (c)

10. Are the integers x, y and z consecutive?


I. The arithmetic mean (average) of x, y and z is y.
II. y-x = z-y
Ans: (e)

Directions 11-15:
Each problem consists of a problem followed by two statements. Decide whether the data in the statements are
sufficient to answer the question. Select your answer according to whether:
(A) statement 1 alone is sufficient, but statement 2 alone is not sufficient to answer the question
(B) statement 2 alone is sufficient, but statement 1 alone is not sufficient to answer the question
(C) both statements taken together are sufficient to answer the question, but neither statement alone is sufficient
(D) each statement alone is sufficient
(E) statements 1 and 2 together are not sufficient, and additional data is needed to answer the question

11. How many ewes (female sheep) in a flock of 50 sheep are black?
There are 10 rams (male sheep) in the flock.
Forty percent of the animals are black.
(a) A (b) B (c)C (d)D (e)E

12. Is the length of a side of equilateral triangle E less than the length of a side of square F?
The perimeter of E and the perimeter of F are equal.
The ratio of the height of triangle E to the diagonal of square F is 2√3 : 3√2.
(a) A (b) B (c)C (d)D (e)E

13. If a and b are both positive, what percent of b is a?


a = 3/11
b/a = 20
(a) A (b) B (c)C (d)D (e)E
.
14. A wheel of radius 2 meters is turning at a constant speed. How many revolutions does it make in time T?
T = 20 minutes.
(a) A (b) B (c)C (d)D (e)E

15. Are the integers x, y and z consecutive?


The arithmetic mean (average) of x, y and z is y.
y-x = z-y
(a) A (b) B (c)C (d)D (e)E

16. Manish goes 7 km towards South-East from his house, then he goes 14 km turning to West. After this he goes 7
km towards North West and in the end he goes 9 km towards East. How far is he from his house?
(a)14 km (b) 7 km (c) 2 km (d) 5 km (e) None of these

Pankaj Gandhi’s Academy


rd
3 Floor, Above Hotel Woodland, Near Sharda Center, Nal Stop, Pune. Contact No. : 8600972993, 9850424051.
17. Nivedita stops after going 10 km towards west from her office. Then she goes 8 km turning to her left. After this
she goes 4 km turning to her left. How far is she from her office?
(a) 18 km (b) 8 km (c) 10 km (d) 14 km (e) None of these

18. Ranju is at a fixed point, from where she goes 20 metres towards West. From there she goes 10 metres towards
North. Then she goes 35 metres towards East and after this she goes 5 metres towards South and in the end she
goes 15 metres towards West. How far is she from the fixed point?
(a) 5 km (b) 0 km (c) 10 km (d) Can not be determined (e) None of these

19. A man walks 15m towards South from a fixed point. From there he goes 12 m towards North and then 4 m
towards West. How far and in what direction is he from the fixed point?
(a) 3 m, South (b) 7 m, South-West (c) 5 m, South-West (d) 5 m, South-East (e) None of these

20. Ranjan goes 5 km towards North from a fixed point. Then he goes 3 km after turning to his right. After this he goes
5 km turning to his right. In the end he goes 4 km after turning to his left. How far and in what direction is he now from
the fixed point?
(a) 4 km, West (b) 7 km, East (c) 9 km, East (d) 7 km, West (e) None of these
.

Directions Question 21 to 24 :
Read the following information carefully and answer the questions given it.
i) Jayant, Kama, Namitha, Asha and Tanmay are five members of the family.
ii) They have their birth dates from January to May, each member in one of these months
iii) Each one likes one particular item for his/her birthday out of Bengali Sweets, Chocolates, Pastries, Ice Cream and
Dry Fruits
iv) The one who likes Pastries is born in the month which is exactly middle in the months given
v) Asha does not like Ice cream but brings Chocolates for Jayant in February.
vi) Tanmay who is fond of Bengali sweets in born in the next month immediately after Namitha
vii) Namita does not like Dry fruits or Ice cream

21. What is the choice of Asha?


(a) Pastries (b) Dry Fruits (c) Dry Fruits (d) Cannot be determined (e) None of these

22. Which combination of month and item is true for Jayant?


(a) March- Pastries (b) February – Pastries (c) February - choclates
(d) Cannot be determined (e) None of these

23. What is the choice of Kamal?


(a) Ice Cream (b) Bengali sweets (c) Dry fruits (d) Cannot be determined (e) None of these

24. In which month was Kamal born?


(a) January (b) May (c) January or May (d) Data inadequate (e) None of these

25. If in a certain code "RANGE" is coded as 12345 and "RANDOM" is coded as 123678, then the code for the word
"MANGO" would be
(a) 82357 (b) 84563 (c) 82347 (d) 82543

26. Which figure completes the series?

Solution : Only figure B follows the pattern.

Pankaj Gandhi’s Academy


rd
3 Floor, Above Hotel Woodland, Near Sharda Center, Nal Stop, Pune. Contact No. : 8600972993, 9850424051.
27. Which figure completes the series?

Solution : Only figure C follows the pattern.

28. Which figure completes the series ?

Solution : Only figure 1 follows the pattern

29. Which figure completes the series?

Solution : Only figure 4 follows the pattern

30. Which figure completes the series?

Solution : Only figure C follows the pattern

31. A theif steals half the total no of loaves of bread plus 1/2 loaf from a backery. A second thief steals half the
remaing no of loaves plus 1/2 loaf and so on. After the 5th theif has stolen there are no more loaves left in the
backery. What was the total no of loaves did the backery have at the beginning.
Solution : They are stealing in amount (x+1)/2 , (x+1)/4 , (x+1)/16 .....(x+1)/3 Subtract it from x, we will get x=31.

Pankaj Gandhi’s Academy


rd
3 Floor, Above Hotel Woodland, Near Sharda Center, Nal Stop, Pune. Contact No. : 8600972993, 9850424051.
I TEST PAPER - 14
Directions 1-5: Read the following instructions carefully and answer sthe questions given below it:
From a group of six boys M,N,O,P,Q,R and five girls G,H,I,J,K as team of six is to be selected .Some of the criteria of
selection are as follows:
(i) M and J go together
(ii)O cannot be placed with N
(iii)I cannot go with J
(iv)N goes with H
(v)P and Q have to be together
(vi)K and R go together
Unless otherwise stated, these criteria are applicable to all the following questions:
1. If the team consists of 2 girls and I is one of them , the other members are
(a) GMRPQ (b) HNOPQ (c) KOPQR (d) KRMNP

2. If the team has four boys including O and R , the members of the team other than O and R are
(a)HIPQ (b)GKPQ (c) GJPQ (d) GJMP

3. If four members are boys, which of the following cannot constitute the team?
(a) GJMOPQ (b) HJMNPQ (c) JKMNOR (d)JKMPQR

4. If both K and P are members of the team and three boys in all are included in the team, the members of the team
other than K and P are
(a) GIRQ (b) GJRM (c) HIRQ (d) IJRQ

5. If the team has three girls including J and K , the members of the team other than J and K are
(a) GHNR (b) MNOG (c) MORG (d) NHOR

Directions 6-10: Each problem consists of a problem followed by two statements. Decide whether the data in the
statements are sufficient to answer the question. Select your answer according to whether:
(a) If the question can be answered by using one of the statement alone, but cannot be answered using the other
statement alone.
(b) If the question can be answered by using either statement alone.
(c) If the question can be answered by using both statements together, but cannot be answered using either statement
alone.
(d) If the question cannot be answered even by using both statements together.

6. What is the value of (a+c):c


I. a:b=2:3
II. b:c=4:5
(a)

7. What is the average of 10 students ?


I. The average age of six youngest students is 10 years.
II. The average age of four youngest students is 5 years.
(d)
8.The distance travelled varies directly as the square of the time taken.What is the distance travelled in 10th second ?
I.Distance travelled in 5 seconds is 124 m
II.Distance travelled in 10 seconds is 567 m.
(b)
9. What is the total amount deposited in 5 accounts ?
I.The average deposit of 5 accounts is Rs 500.
II. The highest and lowest amount of money in an account is 100 and 200.
(a)
10. A vessel is 50% full and another is 60% full. Which vessel has a larger capacity.
I. Vessel A gets empty to make 5/6 of vessel B full.
II. 50% of water from vessel A accounts for 2/3 of vessel B.
(b)

Pankaj Gandhi’s Academy


rd
3 Floor, Above Hotel Woodland, Near Sharda Center, Nal Stop, Pune. Contact No. : 8600972993, 9850424051.
Directions 11-15: A cube is cut in two equal parts along a plane parallel to one of its faces. One piece is then colored
red on the two larger faces and green on remaining. While the other iscolored green on two smaller adjacent faces
and red on the remaining. Each is then cut into 32 cubes of same size and mixed up.
11. How many cubes have only one colored face each?
(a) 32 (b) 8 (c) 16 (d) 24 (e) 0

12. What is the number of cubes with at least one green face on each?
(a) 36 (b) 32 (c) 38 (d) 48 (e) 40

13. How many cubes have two red and one green face on each?
(a) 16 (b) 12 (c) 8 (d) 4 (e) 0

14. How many cubes have no colored face at all?


(a) 32 (b) 24 (c) 16 (d) 8 (e) 0

15. How many cubes have each one red and another green?
(a) 0 (b) 8 (c) 16 (d) 24 (e) 32

16. A man has only 20-paise and 25-paise coins in a bag. If he has 50 coins in all totals to Rs.11.25, then the number
of 20-paise coins is
(a) 28 (b) 27 (c) 26 (d) 25 (e) None of these

17. Working independently, Tina can do a certain job in 12 hours. Working independently, Ann can do the same job in
9 hours. If Tina works independently at the job for 8 hours and then Ann works independently, how many hours will it
take Ann to complete the remainder of the jobs?
(a) 2/3 (b) 3/4 (c)1 (d)2 (e)3

18. Jayant gets 3 marks for each right sum and loses 2 marks for each wrong sum. He attempts 30 sums and obtains
40 marks. The number of sums attempted correctly is
(a) 25 (b) 20 (c) 26 (d) 27 (e) None of these

19. A motor cyclist participant of a race says "We drove with the speed of 10 miles an hour one way, but while
returning because of less traffic we drove on the same route with 15 miles per hour." What was their average speed in
the whole journey?
(a)12 (b)12.5 (c)13 (d)13.5 (e)14

20. Three customers want haircut and a shave. In a saloon, two barbers operate at same speed. They take quarter of
an hour for the haircut and 5 mins for the shave. How quickly can they finish the haircut and shave of these three
customers?
(a)39,21,12 (b)35,21,30 (c)40,27,21 (d)45,34,23

21. A shopkeeper likes to arrange and rearrange his collection of stamps. He arranges them sometimes in pair,
sometimes in bundle of three, sometimes in bundle of fours, occasionally in bundle of fives and sixes. Every time he's
left with one stamp in hand after arrangement in bundles. But if he arranges in the bundle of seven, he's not left with
any stamp. How many stamps does a shopkeeper have?
(a)301 (b)351 (c)401 (d)451 (e)251

22. A man can row at 5 kmph in still water. If the velocity of current is 1 kmph and it takes him 1 hour to row to a place
and come back, how far is the place?
(a) 2.4 km (b) 2.5 km (c) 3 km (d)3.6 km

23. A boat running upstream takes 8 hours 48 minutes to cover a certain distance, while it takes 4 hours to cover the
same distance running downstream. What is the ratio between the speed of the boat and speed of the water current
respectively?
(a) 2 : 1 (b) 3 : 2 (c) 8 : 3 (d) Cannot be determined (e) None of these

24. The population of a town increased from 1,75,000 to 2,62,500 in a decade. The average percent increase of
population per year is:
(a) 4.37% (b) 5% (c) 6% (d)8.75% (e) None of these

25. A person walks on a plain land at 4 km/hr and up the slope at 3 km/hr and comes down at 6 km/hr and walks on
the plain land again at 4 km/hr. if he started at 3:00 pm and comes back at 9:00 pm. What is the one way distance?

Pankaj Gandhi’s Academy


rd
3 Floor, Above Hotel Woodland, Near Sharda Center, Nal Stop, Pune. Contact No. : 8600972993, 9850424051.
(a)16 (b)20 (c)24 (d)28 (e)32

26. If the ratio of production of 3 different companies A ,B & C is 4:7:5 and of overall production last year was 4lac
tones and if each company had an increase of 20% in production level this year what is the production of Company B
this year?
(a) 2.1L (b) 22.1L (c) 4.1L (d) None of these

27.If 70% of a no. is subtracted from itself it reduces to 81.what is two fifth of that no.?
(a)108 (b) 54 (c) 210 (d) None of these

28. If a certain sum of money at SI doubles itself in 5 years then what is the rate?
(a) 5% (b) 20% (c) 25% (d) 14.8%

29. A alone can do a work in 6 days B alone can do in 8 days with help of c they finished the work in 3 days.If the
agreed sum is 640 what is the share of c.
(a)18 (b)80 (c)64 (d)32 (e)81

30. Three companies are working independently and receiving the savings 20%, 30%, 40%. If the companies work
combinely, what will be their net savings?
(a) 40% (b) 30% (c) 20% (d) None of these

I TEST PAPER - 15
Directions (Questions 1-5) Each of the following questions consists of five figures marked A, B, C, D and E called
the Problem Figures followed by five other figures marked 1, 2, 3, 4 and 5 called the Answer Figures. Select a figure
from amongst the Answer Figures which will continue the same series as established by the five Problem Figures.

1. Select a figure from amongst the Answer Figures which will continue the same series as established by the five
Problem Figures.
Problem Figures : Answer Figures :

(a) (b) (c) (d) (e) (1) (2) (3) (4) (5)

(a) 1 (b) 2 (c) 3 (d) 4 (e) 5

2. Select a figure from amongst the Answer Figures which will continue the same series as established by the five
Problem Figures.
Problem Figures : Answer Figures :

(a) (b) (c) (d) (e) (1) (2) (3) (4) (5)

(a) 1 (b) 2 (c) 3 (d) 4 (e) 5

3. Select a figure from amongst the Answer Figures which will continue the same series as established by the five
Problem Figures.
Problem Figures: Answer Figures:

(a) (b) (c) (d) (e) (1) (2) (3) (4) (5)

(a) 1 (b) 2 (c) 3 (d) 4 (e) 5

4. Select a figure from amongst the Answer Figures which will continue the same series as established by the five
Problem Figures.
Problem Figures : Answer Figures:

Pankaj Gandhi’s Academy


rd
3 Floor, Above Hotel Woodland, Near Sharda Center, Nal Stop, Pune. Contact No. : 8600972993, 9850424051.
(a) (b) (c) (d) (e) (1) (2) (3) (4) (5)

(a) 1 (b) 2 (c) 3 (d) 4 (e) 5

5. Select a figure from amongst the Answer Figures which will continue the same series as established by the five
Problem Figures :
Problem Figures: Answer Figures:

(a) (b) (c) (d) (e) (1) (2) (3) (4) (5)

(a) 1 (b) 2 (c) 3 (d) 4 (e) 5

Directions (Questions 6-10) Each of the following Questions consists of five options. Choose the best option that
suits the question given.
6. Nitin was counting down from 32. Sumit was counting upwards the numbers starting from 1 and he was calling out
only the odd numbers. What common number will they call out at the same time if they were calling at the same
speed?
(a) 19 (b) 21 (c) 22 (d) They will not call out the same number (e) None of these.

7. Radha moves towards South-east a distance of 7 km, then she moves towards West and travels a distance of 14m.
From here, she moves towards North-west a distance of 7 m and finally she moves a distance of 4 m towards East
and stood at that point. How far is the starting point from where she stood?
(a) 3 m (b) 4 m (c) 8 m (d) 10 m (e) 11 m

8. In a certain office, 1/3 of the workers are women, 1/2 of the women are married and 1/3 of the married women have
children. If 3/4th of the men are married and2/3 rd of the married men have children, what part of workers are without
children?
(a) 5/18 (b) 4/9 (c) 11/18 (d) 17/18 (e) 17/36

9. A, P, R, X, S and Z are sitting in a row. S and Z are in the centre. A and P are at the ends. R is sitting to the left of
A. Who is to the right of P?
(a) P (b) A (c) X (d) S (e) z

10. Introducing a boy, a girl said, "He is the son of the daughter of the father of my uncle." How is the boy related to
the girl?
(a) Cousin (b) Nephew (c) Uncle (d) Son in law (e) Grandfather

Directions (Questions 11-12) : Each Question Given Below has a problem and two statements numbered I and II
giving certain Information. You have to decide if the information given in the statements is sufficient for answering the
problem. Indicate your answer as
(i) if data in statement I alone are sufficient to answer the question;
(ii) if data in statement II alone are sufficient to answer the question;
(iii) if data either in I or II alone are sufficient to answer the question;
(iv) if the data even in both the statements together are not sufficient to answer the question;
(v) if the data in both the statements are needed.

11. Is Arun taller than Sachin?


I. Dinesh is of the same height as Arun and Sachin. II. Sachin is not shorter than Dinesh.
(a) i (b) iii (c) ii (d) v (e) iv

12. In a certain code language, ‘13’ means ‘stop smoking’ and ‘59’ means ‘injurious habit’. What is the meaning of ‘9’
and ‘5’ respectively in that code?
I. ‘157’ means ‘stop bad habit’ II. ‘839’ means ‘smoking is injurious’.
(a) ii (b) iii (c) v (d) iv (e) i

Pankaj Gandhi’s Academy


rd
3 Floor, Above Hotel Woodland, Near Sharda Center, Nal Stop, Pune. Contact No. : 8600972993, 9850424051.
Directions (13- 15) : In the following problem, there is one question and three statements I, II and III below the
question. You have to decide whether the data given in the statements is sufficient to answer the question. Read all
the statements carefully and find out the probable pair which can be sufficient to answer the question.
13. Five persons --- A, B, C, D and E are sitting in a row. Who is sitting in the middle?
I. B is in between E and C. II. B is to the right of E. III. D is in between A and E.
(a) I and II together (b) II and III together (c) I and III together (d) I, II and III together (e) Data insufficient.

14. Four Subjects - Physics, Chemistry, Mathematics and Biology were taught in four consecutive periods of one hour
each starting from 8.00 a.m. At what time was the Chemistry period scheduled?
I. Mathematics period ended at 10.00 am which was preceded by Biology.
II. Physics was scheduled in the last period. III. Mathematics period was immediately followed by Chemistry.
(a) Only I (b) Only I or II (c) Only II (d) II and III together (e) I and II together or I and III together

15. How many sons does Sharma have?


I. Saurav and Aditya are brothers of Sonali. II. Ayesha is sister of Sharmila and Aditya.
III. Ayesha and Sonali are daughters of Sharma.
(a) I and II only (b) II and III together (c) I, II and III together
(d) I, II, III together are not sufficient (e) I and III together

Directions (Questions 16-20) : The following table shows the number of new employees added to different
categories of employees in a company and also the no of employees from these categories who left the company ever
since the foundation of the company in 1995.

Year Managers Technicians Operators Accountants Peons


New Left New Left New Left New Left New Left
1995 760 - 1200 - 880 - 1160 - 820 -
1996 280 120 272 120 256 104 200 100 184 96
1997 179 92 240 128 240 120 224 104 152 88
1998 148 88 236 96 208 100 248 96 196 80
1999 160 72 256 100 192 112 272 88 224 120
2000 193 96 288 112 248 144 260 92 200 104
16. During the period of 1995 and 2000, the total no of operators who left the company is what percent of the total
number of Operators who joined the company?
(a) 19% (b) 21% (c) 27% (d) 29% (e) 32%

17. For which of the following categories the percentage increase in the number of employees working in the company
from 1996 to 2000 was maximum?
(a) Managers (b) Technicians (c) Operators (d) Accountants (e) Peons

18. What is the difference between total number of Technicians added to the company and total number of
Accountants added to the company during the year 1996 to 2000 at the maximum?
(a) 128 (b) 112 (c) 96 (d) 88 (e) 72

19. What was the total no. of peons working in the company in the year 1999?
(a) 1312 (b) 1192 (c) 1088 (d) 968 (e) 612

20. What is the pooled average of all employees in the year 1997?
(a) 1325 (b) 1285 (c) 1265 (d) 1235 (e) 1195

Directions (Questions 21-25) : The following questions are based on the information given. Choose the appropriate
answer that suits the question based on the information.
A cube is cut in two equal parts along a plane parallel to one of its faces. One piece is then colored red on the two
larger faces and green on remaining. While the other is colored green on two smaller adjacent faces and red on the
remaining. Each is then cut into 32 cubes of same size and mixed up.

Pankaj Gandhi’s Academy


rd
3 Floor, Above Hotel Woodland, Near Sharda Center, Nal Stop, Pune. Contact No. : 8600972993, 9850424051.
21. How many cubes have only one colored face each?
(a) 32 (b) 8 (c) 16 (d) 24 (e) 0

22. What is the number of cubes with at least one green face on each?
(a) 36 (b) 32 (c) 38 (d) 48 (e) 40

23. How many cubes have two red and one green face on each?
(a) 16 (b) 12 (c) 8 (d) 4 (e) 0

24. How many cubes have no colored face at all?


(a) 32 (b) 24 (c) 16 (d) 8 (e) 0

25. How many cubes have each one red and another green?
(a) 0 (b) 8 (c) 16 (d) 24 (e) 32

Directions (Questions 26-30): In each of the following questions two statements are given and these statements are
followed by two conclusions numbered (1) and (2). You have to take the given two statements to be true even if they
seem to be at variance from commonly known facts. Read the conclusions and then decide which of the given
conclusions logically follows from the two given statements, disregarding commonly known facts.

26. Statements: Some papers are pens. All the pencils are pens.
Conclusions : 1. Some pens are pencils. 2. Some pens are papers.
(a) Only (1) conclusion follows (b) Only (2) conclusion follows (c) Both (1) and (2) follow
(d) Either (1) or (2) follows (e) Neither (1) nor (2) follows

27. Statements: Some dogs are bats. Some bats are cats.
Conclusions: 1. Some dogs are cats. 2. Some cats are dogs
(a) Only (2) conclusion follows (b) Both (1) and (2) follow (c) Either (1) or (2) follows
(d) Only (1) conclusion follows (e) Neither (1) nor (2) follows

28. Statements: All the windows are doors. No door is a wall.


Conclusions: 1. Some windows are walls. 2. No wall is a door.
(a) Either (1) or (2) follows (b) Only (2) conclusion follows (c) Both (1) and (2) follow
(d) Neither (1) nor (2) follows (e) Only (1) conclusion follows

29. Some actors are singers. All the singers are dancers.
Conclusions : 1. Some actors are dancers. 2. No singer is actor.
(a) Only (1) conclusion follows (b) Only (2) conclusion follows (c) Either (1) or (2) follows
(d) Neither (1) nor (2) follows (e) Both (1) and (2) follow

30. Statements: All the pencils are pens. All the pens are inks.
Conclusions: 1. All the pencils are inks. 2. Some inks are pencils.
(a) Only (2) conclusion follows (b) Only (1) conclusion follows (c) Either (1) or (2) follows
(d) Both (1) and (2) follow (e) Neither (1) nor (2) follows

Pankaj Gandhi’s Academy


rd
3 Floor, Above Hotel Woodland, Near Sharda Center, Nal Stop, Pune. Contact No. : 8600972993, 9850424051.
I Sample Paper - 16
1. A train ran at a certain speed for one hour and then due to a technical problem, it ran at 3/5th of its original speed
and reached the destination 2 hrs late. The driver said, had the problem occurred 50 miles later, we would have
gained 40 minutes. What is the distance between the two cities that the train travelled?
a) 100 miles b) 125 miles c) 150 miles/hr d) 280 miles/hr

2. Each one wearing hat of one person and riding the bicycle of another. A is wearing B's hat and riding C's cycle.
Who is riding A's cycle?
a) a b) b c) c d) none of this

3. A ship went on a voyage after 180 miles a plane started with 10 times speed that of the ship. Find the distance
when they meet from starting point.
a)215 b)180 c) 200 d) none of this

5. My husband's watch gains 2 minutes every watch and my watch loses 1 minute for each hour. One day, we were
late to marriage because the difference between the time in the two watches was 1 hr and we looked at the slow
watch. When did we last set our watches to the same time?
a)20 hr b)180 c) 200 d) none of this

6. Salay walked 10 m towards West from his house. Then he walked 5 m turning to his left. After this he walked 10 m
turning to his left and in the end he walked 10 m turning to his left. In what direction is he now from his starting point?
(a) South (b) North (c) East (d) West (e) None of these

7. Laxman went 15 kms south from my house, then turned left and walked 20 kms. He then turned east and walked 25
kms and finally turning left covered 20kms. How far was he from his house.
(a) 5 kms (b) 10 kms (c) 40 kms (d) 80 kms (e) None of these

8. The door of Aditya's house faces the east. From the back side of his house, he walks straight 50 metres, then turns
to the right and walks 50 metres, then turns towards left and stops after walking 25 metres . Now Aditya is in which
direction from the starting point?
(a) South-East (b) North-East (c) South- West (d) North-West (e) None of these

9. P, Q, R and S are playing a game of carom. P, R, and S, Q are partners and sits opposite to each other. S is to the
right of R who is facing west. Then Q is facing?
(a) North (b) South (c) East (d) West (e) None of these

10. A man is facing south. He turns 1350 in the anticlockwise direction and then 180o in the clockwise direction. Which
direction is he facing now.?
(a) North-east (b) North-West (c) South-east (d) South-west

Directions 11-15: Each problem consists of a problem followed by two statements. Decide whether the data in the
statements are sufficient to answer the question. Select your answer according to whether:
(a) statement 1 alone is sufficient, but statement 2 alone is not sufficient to answer the question
(b) statement 2 alone is sufficient, but statement 1 alone is not sufficient to answer the question
(c) both statements taken together are sufficient to answer the question, but neither statement alone is sufficient
(d) each statement alone is sufficient
(e) statements 1 and 2 together are not sufficient, and additional data is needed to answer the question

11. Every pupil in a school was given one ticket for a concert. The school was charged a total of $6000 for these
tickets, all of which were of equal value. What was the price of one ticket?
1. If the price of each ticket had been one dollar less, the total cost would have been 1,200 less.
2. If the price of each ticket had been $2 more, the total bill would have increased by 40%. ANS: (d)

12. What is the ratio of male to female officers in the police force in town T?
1. The number of female officers is 250 less than half the number of male officers.
2. The number of female officers is 1/7 the number of male officers. ANS: (b)

13. What is the value of n?


1. 3n + 2m = 18
2. n – m = 2n – (4 + m) ANS: (b)

Pankaj Gandhi’s Academy


rd
3 Floor, Above Hotel Woodland, Near Sharda Center, Nal Stop, Pune. Contact No. : 8600972993, 9850424051.
14. How long did it take Henry to drive to work last Wednesday? (He did not stop on the way).
1. If he had driven twice as fast he would have taken 35 minutes.
2. His average speed was 30 miles per hour. ) ANS: (a)

15. What is the slope of line l which passes through the origin of a rectangular coordinate system?
1. The line does not intersect with the line y = x + 2.
2. The line passes through the point (3,3) ) ANS: (d)

Directions 16 to20 : Read the following information carefully and answer the questions given it.
i) P, Q, R, S, T and U are six students procuring their Master's degree in six different subjects - English, History,
Philosophy, Physics, Statistics and Mathematics.
ii) Two of them stay in hostel, two stay as paying guest (PG) and the remaining two stay at their home
iii) R does not stay as PG and studies Philosophy
iv) The students studying Statistics and History do not stay as PG or hostel
v) T studies Mathematics and S studies Physics
vi) U and S stay in hostel. T stays as PG and Q stays at home

16. Who studies English?


(a) R (b) S (c) T (d) U (e) None of these

17. Which of the following combinations of subject and place of stay is not correct?
(a) English - Hostel (b) mathematics – PG (c) Philosophy –Home (d) Physics –Hostel (e) None of these

18. Which of the following combinations of subject and place of stay is not correct?
(a) P,PG, english (b) S,home,physics (c) R, home , philosophy (d) T, PG,mathematics (e) None of these

19. Which subject does Q study?


(a) History (b) Statistics (c) History or Statistics (d) Data inadequate (e) None of these

20. Which of the following pairs of students stay at home?


(a) PQ (b) QR (c) RS (d) ST (e) None of these

21. A man is facing north-west. He turns 90o in the clockwise direction and then 135o in the anticlockwise direction.
Which direction is he facing now?
(a) East (b) West (c) North (d) South

22. A man is facing north-west. He turns 90o in the clockwise direction, then 180o in the anticlockwise direction and
then another 90o in the same direction. Which direction is he facing now?
(a) South (b) Southwest (c) West (d) South-east

23. Radha moves towards South-east a distance of 7 km, and then she moves towards west and travels a distance of
14 m. From here, she moves towards North-West a distance of 7 m. and finally she moves a distance of 4 m towards
East and stood at that point. How far is the starting point from where she stood?
(a) 3 m (b) 4 m (c) 10 m (d) 11 m

24. A rat runs 20' towards East and turns to right, runs 10' and turns to right, runs 9' and again turns to left, runs 12'
and finally turns to left and runs 6'. Now, which direction is the rat facing?
(a) East (b) West (c) North (d) South

Directions 25-29: In each question below are given three Statements followed by four Conclusions numbered I, II and
III, IV. You have to take the given Statements to be true even if they seem to be at variance from commonly known
facts. Read all the conclusions and then decide which of the given Conclusions logically follows from the given
Statements disregarding commonly known facts.

25. Statements: No apple is an orange. All bananas are oranges.


Conclusions:
(I) All apples are oranges (II) Some apples are oranges (III) No apple is a banana
(a) all three (b) I and II (c) only III (d) none

26. Statements: All pens are elephants. Some elephants are cats.
Conclusions:
(I) Some pens are cats (II) No pens are cats (III) All pens are cats

Pankaj Gandhi’s Academy


rd
3 Floor, Above Hotel Woodland, Near Sharda Center, Nal Stop, Pune. Contact No. : 8600972993, 9850424051.
(a) none (b) I and II (c) I or II (d) only III

27. Statements: All shares are debentures. No debentures are deposits.


Conclusions:
(I) All shares are deposits (II) Some shares are deposits (III) No shares are deposits
(a) all three (b) I and II (c) only III (d) none
28. Statements: Many fathers are brothers. All brothers are priests.
Conclusions:
(I) No father is a priest (II) Many fathers are not priests (III) Many fathers are priests
(a) only I (b) only I and II (c) only II (d) only III

29. Statements: Some pens are pencils. All pencils are costly.
Conclusions:
(I) No pens are costly (II) Some pens are costly (III) All pens are costly

(a) only II (b) I or II (c) only III (d) none

30. Statements: All the harmoniums are instruments. All the instruments are flutes.
Conclusions:
(I) All the flutes are instruments (II) All the harmoniums are flutes (III)Some instruments are
harmoniums
(a) only I (b) II and II (c) only II (d) all three

IMP Questions
1. Find the 8th term 0,0,6,6,20,20,42,_,_ Ans: 72,72

2. There are five same model cars parked in the same direction in parking. If 3 cars are red, 1 is black and 1 is green,
in how many ways they can be arranged in the parking? 5! / 3! = 20 ways

3. You meet three people, A, B and C in an island (people are either liars or truth-tellers).
You ask A: - How many truth-tellers are among you guys?
The volcano makes a loud noise when A answers and you can't hear the answer.
So you ask B: What did A say? -
and B answers: - A said that one of us is a truth-teller and two of us are liars.
and then C adds - Don't believe what B said, he's lying.
What class of people are B and C? Ans: B= Lier, C speaks truth

4. A person says that their speed while going to a city was 10mph however while returning as there is no much traffic
they came with a speed of 15mph. What is their average speed? Ans: 12 mph

5. Sum of all 120 numbers formed by using 1, 2, 3, 3, 3, and 4 is? Ans: 33399300

6. There are 12 bottles in a refrigerator out of which there are 7 identical blue bottles and 5 identical red bottles. In
how many ways can we take out bottles from the fridge such that at least one blue and one red bottle is in the fridge.
Ans. 4×6 =24

7. 2,6,30,60,130,210 what is the 8th number in series? Ans: 350

8. Reversing the digits of father`s age we get son`s age. One year ago father was twice in age of that of his son? find
their current ages?
Ans: F = 73, S= 37. 10x + y - 1 =2(10y + x - 1)

9. A father and boy rowing in river upstream when they move one mile, boy noticed that his father hat was flown with
current of river. After 5 min he tells his father that his hat has fallen. Then they return back downstream and took the
hat from the point where they began boating in 5 min. What is the speed of the water current? Ans. 6 miles/hr

10. One neighbour said to other “This year she had squarer plot of land to cultivate cabbage if she could cultivate 599
more cabbage this year, what is the size of the plot this year? Ans: 90000

Pankaj Gandhi’s Academy


rd
3 Floor, Above Hotel Woodland, Near Sharda Center, Nal Stop, Pune. Contact No. : 8600972993, 9850424051.
th
11. The ball has thrown from 180 feet height, every time it jumps 1/10 of its height. How much distance it will travel?
Ans: 220

12. Preeti has some sons, every son has sons as many brothers. Her age is between 50 to 70. What is the age of
Preeti? Ans: 64

13. 161?85?65?89=100
then use + or - in place of ? and take + as m,- as n then find value of m-n
1)3 2)-1 3)1 4)-3
Ans. 161 – 85 – 65 + 89=100
m=1 n= -2
m – n = 1 – (- 2)= 3

14. The width of a river is 4100 inches and there is a bridge of which, one eight of the area is one side of the river and
1/7th is another side. Then what is length of bridge?
14. + 4100 + = l = 5600

15. There is some number of stations. For every station the tickets will available from that station. In that route some
new stations are opened then 34 tickets are sold out extra.
(a) How many stations before (b) How many stations new
(a) 8 (b) 8+2 = 10

16. One man is coming down from the top of the stairs and he downed the 7 steps. After it another person stated from
the bottom of stairs. When second one reaches top, first person has to travel four more steps to reach bottom. How
many steps are there? 1st one takes 2 steps for second one.

17. There are 7 small bags and 18 large bags with marbles. In travelling those two bags are broken. That's why
marbles are taken out individually. The number of marbles are 233. Then how many marbles in each bag?
Small bag= 35, Large bag = 11x18= 198

18. If it takes 7 cats to kill 7 rats in 7 different ways in 7 days then how many days will it take for 10 cats to kill 10 rats
in 10 different ways? Ans: 7

19. Find the total number of rectangles (include squares also as rectangles) in a 8 * 8 standard chessboard?

20. Find the minimum number of tyres required for a standard old modelled four-wheeled car to travel 48000 miles, if
24000 miles per each tyre? Ans: 8

21. Two weather reporting resources DD-I and BBC gave a report for the rain coming in Bangalore city. The
guarantee of rain not to come given by DD-I is 2/3, while the guarantee of rain to fall given by BBC is 3/4. The chance
of rain to fall is more. If rain falls, what is the guarantee to fall?

22. What is the next term in the below series.


1/5 , 1/2 , 1/3 , 2/5 ,1, 2/3, 3/5, 1 1/2 ,1, 4/5,? Ans: 2

23. 8 : 18 : : 24 : ?
(a) 38 (b) 32 (c) 44 (d) 43
(a) 38 1×8 = 8 similarly 3×8 = 24 so, 38

24. We have 3 weights one of 2 kg, one of 3 kg, one of 5 kg. By using only two weight, how many weights you can
weight? Ans. 3C2× 2 = 6

25. How many 4 digit number are their which have exactly 3 distinct digits? Ans: 3672

26. Find the total number of four digit numbers that can be formed using the digits 1,2,5,6. Ans. 256

27. The digital sum of the year 2007 is 2 + 0 + 0 + 7 = 9, which is a square number. How many years during the
twenty-first century have a square digital sum? Ans: 16

Pankaj Gandhi’s Academy


rd
3 Floor, Above Hotel Woodland, Near Sharda Center, Nal Stop, Pune. Contact No. : 8600972993, 9850424051.
28. There are 7 men and 5 women sitting in a circle, find all the possible way so that 2 women will not sit together.
8
Ans: C5 x 5! X 7!

29. Find four digit number formed given digits 1,2,3,4,5,5,6,6 repetition of the digits not allowed.
Ans. × × × = 360

30. Find the 8th term 3,11,31,69,131, 223, 351, ? = 521 difference of difference

31. How many 4 digit number can be formed if


1) the number will be an even number 2) they must be divisible by 4 3) repetition is not allowed
Ans: 1) 9x10x10x5 2) 9x10x25 3) 9x9x8x7

32. A cube of volume 1000 cm3 is divided into small cubes of 1cm3 and all the outer surfaces are painted. How many
cubes will be there with at least one side painted?
Ans: zero side coloured = (10 – 2)3 = 512
At least one side coloured = 1000 – 512 = 488

33. Five persons Muckerjee, Misra, Iyer, Patil and Sharma, all take then first or middle names in the full names. There
are 4 persons having first or middle name of Kumar, 3 persons with Mohan, 2 persons with Dev and 1 Anil.
-- Either Mukherjee and Patil have a first or middle name of Dev or Misra and Iyer have their first or middle name of
Dev.
-- Of Mukherkjee and Misra, either both of them have a first or middle name of Mohan or neither have a first or middle
name of Mohan.
-- Either Iyer of Sharma has a first or middle name of Kumar but not both.
Who has the first or middle name of Anil?
Ans: Muckerjee

34. Find m+n?


if a. (x+m)(x+n) = x2 + 5x + mn
b. mn = 4 or 6
1. a is enough to derive the answer 2. b is enough, 3. Both are needed 4.can't find using two

35. x2 - y2 = 16 and 2. xy = 4. Find x+y=? both statement are required to find out the value of x+y

36. Find the 8th term of this series 4, 8, 14, 22, 32, ? 44, 58, ? = 74

37. What is the next number of the following sequence 1,3,1,2.5,5,1,4,5,4,8,7,? Ans: 9.5

38. Students are standing in circular track. If 12th and 30th students are opposite to each other. Find total number of
students. Ans: 36

39. How many arrangements can be made if there are 5 balls from which 3 are red, 1 is green, 1 is yellow. Find the
probability of getting all 3? Ans: 20 ways

40. P, Q, R and S are four persons who were seated in a room in the directions North-East, South-East, South-West
and North-West respectively. People at one end of the diagonals move to the other end of the diagonal. Based on the
following information solve the questions:
P moves one and half side in the clockwise direction where as S moves the same amount in the anti-clockwise
direction. What is the current direction of the person P? East

41. What is the next number of the following sequence: 5,6,6,5,6.5,8,7,9.5,?

42. What is the next number of the following sequence: 1,3,1,2.5,5,1,4,5,4,8,7,? Ans: 9.5

43. Raju has 6 balls. Out of which 4 green 2 blue. He'll pick 2 balls. What’s probability that it contain 1 blue and 1
green? 4/15

44. Two pipes A and B separately fill in 15/2 min and 5 min respectively and third pipe C can empty of 14 l/m if all the
pipes are open when the cistern is full, it is emptied in 1 hr. how many litres does the cistern hold? Ans: 40 lit

45. There are 6 balls in bag out of them 4 green and 2 red, randomly picked two balls one by one, so find the
probabilty atleast one picked ball will be red? Ans: 3/5

Pankaj Gandhi’s Academy


rd
3 Floor, Above Hotel Woodland, Near Sharda Center, Nal Stop, Pune. Contact No. : 8600972993, 9850424051.
th th
46. All the students of class are told to sit in circle shape. Here the boy at the 6 position is exactly opposite to 16
boy. What is the total number of boys in the class? Ans: 20

47. Venkat has 1 boy & 2 daughters. The product of these children age is 72. The sum of their ages give the door
number of Venkat. Boy is elder of three. Can you tell the ages of all the three.
Ans: 3,3,8

48. Mark (a) if Statement (1) ALONE is sufficient, but statement (2) is not sufficient
(b) if Statement (2) ALONE is sufficient, but statement (1) is not sufficient
(c)if BOTH statements TOGETHER are sufficient, but NEITHER statement ALONE
(d)if EACH statement ALONE is sufficient
(e)if Statements (1) and (2) TOGETHER are NOT sufficient
(1) Is b a positive number?
(i) 1,452*b > 0
(ii) -b < 0 Ans: Alone 2

(2) Find the area of below plane. All the sides intersect each other at right angle.

(i) QR = 4m
(ii) PQ = 3m
Ans: Statements (1) and (2) TOGETHER are NOT sufficient

(3) Following square area is to be filled by X and O tiles. Find how many X tiles are required if the number of O tiles is
greater than ½ the X tiles.

(i) If the X files are arranged at corners of the square


(ii) X is filled at the centre
Ans: Statements (1) and (2) TOGETHER are sufficient

49. The difference between the compound interest payable half yearly and the simple interest on a certain sum lent
out at 10% per annum for 1 year is Rs 25. What is the sum? Ans. 10,000

50. 5 cats can catch 5 mice in 5 minutes. How many cats does it take to catch 100 mice in 100 minutes? Ans: 5 min

51. If in coded language


23 = 10
72 = 63
65 = 66
84 = 96
Then
97 = ? Ans: 144

52. What number comes next 2, 2, 4, 12, 48, __? 240

53. A card board of 34 * 14 has to be attached to a wooden box and a total of 35 pins are to be used on the each side
of the card box. Find the total number of pins used. Ans: 136 pins

54. If a shopkeeper has two type of rice. One at Rs. 50 and other at Rs.80. He mixed rice in 2:1 ratio. Find the
average price of the mixture. Ans. 60

55. If j/k=1/2, k/l=4, m/n=5/3, n/o=4/5, k/n = 5/3. then find jkl/mno? Ans: 5/18

Pankaj Gandhi’s Academy


rd
3 Floor, Above Hotel Woodland, Near Sharda Center, Nal Stop, Pune. Contact No. : 8600972993, 9850424051.
56. {1,2,3,4,5,5,6,6} How many four digited numbers can be formed without repeating the numbers?
Ans: 6x5x4x3 = 360

57. A wall clock looses 10 mins in 1 hr. For one hour of wall clock table clock gains 10mins.
for 1 hour of table clock alarm clock looses 5 mins for 1hr of alarm clock, wrist watch gains 5min.
they were set correctly at 12 pm. What was the time of wrist watch at 6 pm(of correct time clock)?
Ans : × × × × ≅ so, time –-> 5:50 pm

58. There is a two sided weighing pan with sides of unequal length. Four oranges to right side weigh equal to 1 kg to
the left side. But when 1 kg weight is kept at the right side it needs 16 oranges to the left side to balance it. Find the
weight of each orange
Ans: 1/8 kg

59. "How old are you, Alchemerion?" asked one of the wizards appearances the wizards answer with a riddle, "I am
still Very young as wizards go.I am only three times my son's age. My father is 40 year more than twice of my age.
Together the three of us are a 1240 year old". How old is Alchemerion? Ans: 120 years

60. Cynthia and Reuben go to town to shop. Reuben purchases a hat and a suite, which costs him 15$. Cynthia
purchases a hat, the cost of which is equal to Reuben’s suite. The rest of the money Cynthia spends in buying a new
dress. On the way to home Cynthia called Reuben attention to the fact that his hat costs $1 more than her dress. Then
the Uncle said if we had divided our hat money such that we bought a different hats mine costing 1.5 times cost of
yours then we would have spent the equal amount of money.
a) what is the cost of Cynthia’s hat in that case? 5.6$
b) How much money altogether they spent? 29$

61. Mr. Sekhar and Ms. Sekhar are having conversation like this:
Mr. Sekhar said to Ms. Sekhar: "Give me three quarters of what you have, I will add it with what I have, totally to buy a
house at indiranagar for 5000/-, remaining at hand with you can be bought the running stream and groove". Ms.
Sekhar, said ,"No, No, give me two thirds of what you have , I will add it with what I have, totally we can buy the house
, and the groove and running stream we can buy with the remaining money we have". Can you find the money
required to buy the groove and stream? Ans: Rs. 333.33

62. A supportive young hare and tortoise raced in opposite directions around a circular track that was 100 yards in
diameter. They started at the same spot, but the hare did not move until the tortoise had a start of one eighth of the
distance (that is, the circumference of the circle). The hare held such a poor opinion of the other’s racing ability that he
sauntered along, nibbling the grass until he met the tortoise. At this point the hare had gone one sixth of the distance.
How many times faster than he went before must the hare now run in order to win the race?
Ans: 81 times faster

63. A shopkeeper purchased an article at 20% discount on list price, he marked up his article in such a way that after
selling the article at 20% discount, he gained 20% on SP. what % is SP of the list price?
Ans: 100%. List price is same as S.P.

64.1.All pens are pencils.


2.All pencils are ball.
3.No pencils are ball.
4.Some pens are pencils.
5.All Pencils are Box.
6.All boxes are pens.
A) 654 B) 235 C) 214 D) 621 E) 125

65. Eight persons named Jate, Late, Mate, Nate, Qate, Quote, Kate, Pate stay in different floors in a building. There
are 5 floors in a building. There may be either one or two room in each floor.
Only One person live in second floor.
Mate and Nate always stay together.
Kate is directly next floor to Pate.
Late is only one stays in second floor.
Quote is only in the floor as Qate.
Jate stays in floor with two rooms.

Pankaj Gandhi’s Academy


rd
3 Floor, Above Hotel Woodland, Near Sharda Center, Nal Stop, Pune. Contact No. : 8600972993, 9850424051.
i) Which of the following is correct?
(a)Late stays in third floor.
(b)Late stays in fifth floor.
(c)Mate stays in third floor and Nate in second floor. (d)Late stays in second floor.
(e)Jate and Late stays in second floor.

ii) If Late is in third floor, which is true?


(a) Kate stays in first floor. (b) Pate stays in fourth floor.
(c) Mate stays in first floor. (d) Quote stays in fifth floor.
(e) Jate stays in second floor.

66. A recent murder case centered around the six men, clam, flip, gront, herm, mast, and Walt. In one order or
another man were the victim, the murderer, the witness, the police, the judge, and the hangman. The facts of the case
were simple. The victim had died instantly from the effect of gunshot wound inflicted a shot. After a lengthy trial the
murderer was convicted, sentenced to death, and hanged. Mast knew both the victim and the murderer. In court the
judge asked clam his account of the shooting. Walt was the last of the six to see flip alive. The police testified that he
picked up gront near the place where the body was found. Herm and Walt never met. What role did each of the play in
this melodrama?
Ans: victim - Herm, murderer- Flip, Witness- Gront, Police - Clam, Judge- Mast, Hangman- Walt.

67. "How old are you, Alchemerion?" asked one of the wizards appearances the wizards answer with a riddle, "I am
still Very young as wizards go.I am only three times my son's age. My father is 40 year more than twice of my age.
Together the three of us are a 1240 year old". How old is Alchemerion?
Ans: 120 years

68. players were playing a card game. Cards had different colours on both sides. Neither of cards had same colour on
both sides. Colours were 2 Red, 2 Blue, 2 Green, 2 Yellow. Cards were lying in front of each player. Now, each player
knew the colour on other side of his card. They are required to tell their colour.
Statement given by each of them was :

Annie : Blue or Green


Bobby : Neither Blue nor Green
Cindy : Blue or Yellow
Danny : Blue or Yellow
colours of cards that are visible to all were Red, Blue, Green, Blue in order of their names. Exactly two of them are
telling truth and exactly two of them are lieing. Can you tell the colour on other face of card for each player ?

Ans: Annie : Yellow (Lie)


Bobby : red (Telling truth)
Cindy : yellow (Telling truth)
Danny : Green (Lie)

Q. There are 1000 junior and 800 senior students in a class. And there are 60 sibling pairs where each pair has 1
junior and 1 senior.1 student is chosen from senior and 1 from junior randomly. What is the probability that the two
selected students are from a sibling pair?
Ans. × =
,

Q: 4 persons are there called John, Jacob, Peter, and Williams and four languages English, Italian, German and
French.
There is no common language for all four
Except one language, no language is spoken by more than two
One can either German or French but not both
John can’t speak English but John can act as interpreter between Jacob and Peter
Jacob knows German but he can talk with William who does not know a word of German
No common language between John, Peter and Williams
Which two languages does each person speak?
Ans:
John – German, Italian
Jacob – German, English
Peter – French, Italian
William – English, French

Pankaj Gandhi’s Academy


rd
3 Floor, Above Hotel Woodland, Near Sharda Center, Nal Stop, Pune. Contact No. : 8600972993, 9850424051.
MATHEMATICAL REASONING
(Encrypting Algorithm)
1. EVER+SINCE= DARWIN 5653 + 97825 = 103478

2. WORLD + TRADE = CENTER 53684 + 76042 = 129726

3. CALL + THEIR = OFFICE 7633 + 92845 = 100478

4. DAYS + TOO = SHORT 9871 + 655 = 10526

5. TWO + DAY = MORE 893 + 472 = 1365

6. AFTER + TWO = YEARS 49703 + 728 = 50431

7. CALL + LETTER = OFFICE 1522 + 298897 = 300419

8. ZERO + POINT = ENERGY 3168 + 98504 = 101672

9. USA + USSR =PEACE 932 + 9338 = 10270

10. EAT + THAT = APPLE 819 +9219 = 10038

11. CROSS + ROADS = DANGER 96233 + 62513 =158746

12. SCOOBY + DOOO = BURIED 695573 + 8555 =704128

13. SCOOBY + DOOO = BUSTED 194423 + 7444 = 201867

14.APPLE+GRAPE=CHERRY 63374+90634=154008

15.APPLE+LEMON=BANANA 67794+94832=162626

16.APPLE+LITCHI=CHERRY 31176+794829=826005

17.APPLE+PAPAYA=LITCHI 85569+585838=671407

18.APPLE+PEAR=CHERRY 97745+7593=105338

19.APPLE+PEAR=GRAPE 85572+5280=90852

20.BANANA+GUAVA=ORANGE 249494+65474=314968

21.BANANA+PEACH=LITCHI 416161+87129=503290

22.CHERRY+PEACH=LITCHI 579113+29457=608570

23.GRAPE+PEACH=LITCHI 25384+84376=109760

24.GUAVA+LEMON=ORANGE 95676+30814=126490

25.GUAVA+MELON=ORANGE 95676+30814=126490

26.LEMON+MANGO=BANANA 42673+60357=103030

27.LEMON+MELON=LITCHI 16835+86135=102970

28.LEMON+PLUM=ORANGE 95312+8963=104275

Pankaj Gandhi’s Academy


rd
3 Floor, Above Hotel Woodland, Near Sharda Center, Nal Stop, Pune. Contact No. : 8600972993, 9850424051.
29. MELON+OLIVE=PAPAYA 59274+72089=131363

30.MELON+PLUM=ORANGE 95816+7829=103645

31.ORANGE+PAPAYA=CHERRY 291835+414161=705996

32.PAPAYA+PLUM=CHERRY 595929+5843=601772

33.BLACK+GREEN=ORANGE 79208+53446=132654

34.BROWN+PINK=IVORY 83257+6974=90231

35.BROWN+YELLOW=PURPLE 52813+649981=702794

36.GREEN+IVORY=ORANGE 62334+58129=120463

37.IVORY+PURPLE=VIOLET 28346+794705=823051

38.SILVER+VIOLET=PURPLE 657139+152734=809873

39.BASIL+CARROT=POTATO ` 91572+316608=408180

40.CABBAGE+LETTUCE=EGGPLANT 6355341+8122961=14478302

41.CARROT+CELERY=PARSLEY 628870+654589=1283459

42.CARROT+CELERY=POTATO 486607+421263=907870

42.CARROT+MINT=POTATO 298805+7145=305950

43.CARROT+ONION=TOMATO 493325+27627=520952

44.CARROT+POTATO=LETTUCE 520068+968286=1488354

45.CELERY+GARLIC=OREGANO 782834+603257=1386091

46.CELERY+ONION=GARLIC 189837+64064=253901

47.CELERY+PEPPER=GARLIC 415186+313318=728504

48.CELERY+PEPPER=TOMATO 142435+646643=789078

49.CELERY+TOMATO=LETTUCE 751592+836483=1588075

50.CORN+GARLIC=POTATO 6074+197856=203930

51.GARLIC+ONION=CELERY 594736+12312=607048

52.GARLIC+ONION=PEPPER 623519+84184=707703

53.OKRA+TOMATO=PEPPER 9234+698469=707703

54.ONION+RADISH=TOMATO 57457+193468=250925

55.PARSLEY+PEPPER=LETTUCE 6931752+656653=7588405

56.PEPPER+POTATO=CELERY 363365+398289=761654

57.PEPPER+THYME=CELERY 727723+96512=824235

58.PEPPER+THYME=TURNIP 585587+62318=647905

Pankaj Gandhi’s Academy


rd
3 Floor, Above Hotel Woodland, Near Sharda Center, Nal Stop, Pune. Contact No. : 8600972993, 9850424051.
59.PEPPER+TOMATO=CELERY 252257+198319=450576

60.PEPPER+TURNIP=CELERY 434432+102894=537326

61.POTATO+TOMATO=LETTUCE 863536+369536=1233072

62.POTATO+TOMATO=PUMPKIN 168486+863486=1031972

63.THYME+TOMATO=CARROT 64091+659265=723356

64.AT * AT =PEA 19 * 19 = 361

Pankaj Gandhi’s Academy


rd
3 Floor, Above Hotel Woodland, Near Sharda Center, Nal Stop, Pune. Contact No. : 8600972993, 9850424051.
1. FHW 5. INN 9. KNV
*EBF *AT AMN
---------- -------- -----------
EWHH AORT AVEE
CBEC* BALT RMEW
BLCG** ------ WENF
------------ BOOST
WCAGGH ------------
Ans: 784 Ans: 466 ANNVGE
*537 38
-------------- ---- Ans: 947
5488 3728 364
2352 1398 ---------
3920 ----- 3788
-------- 17708 5682
421008 2841
6. RED ----------
2. WHY*NUT *AS 344708
-------- ----- 10. VNK
OONP ARCS NMA
OYPY RED -----------
OUHA ------ WWNV
--------- CDTS NNKN
ONEPOP Ans: 247 WKKM
Ans: 234 *15 -----------
*765 ---------- ANMVEV
--------- 1235 Find the value of N+N+K+N=?
1170 247
1404 ---------- Find the value of W+K+K+M=?
1638 3705 Which of the following value
--------- comes at least one in
179010 7. H A T multiplication?
*CUP (a) 0 (b) 1
3. P O D --------------------- (c) 5 (d)8
SPY EIUI Ans: 749
_______ EART- 463
ESPA EUPI-- ----------
RSQ* --------------------- 2247
OSYS** HIEEEI 4494*
----------- Ans: 345 2996**
ODISHA *876 ---------
---------- 346787
3. 317 2070 V=7, N=4,
538 2415 M=6,K=9,A=3,W=2,E=8
_______ 2760
2536 ---------- 11. PAS
951* 302220 *RBQ
1585** ----------------
----------- 8. SAY SBKW
170546 *MY ASAA
-------------- SEPB
4. DOG NAME ---------------
* GO AMNE SQSKAW
------ ---------
BOOO STYLE Ans: 543
APIG 687
------ Ans: 3 2 4 -------
BBUD *94 3801
Ans: 486 ----------- 4344
68 1296 3258
_____ 2916 -------
3888 ------------ 373041
30456
2916
_____
33048

Pankaj Gandhi’s Academy


rd
3 Floor, Above Hotel Woodland, Near Sharda Center, Nal Stop, Pune. Contact No. : 8600972993, 9850424051.
12. H M K 15. PDA 19. GAS
AVE *HAD *FBI
----------- ----------- ------
ANXX NHHZ FTBI
XAVH DNZE SSTB
MXVW DIED SASF
------------------ --------- --------
MAMVW X ANDAAZ What is 5(H-A)? SRISTI
Ans: 723
Ans: 846 *432 Ans: 571
*537 --------- 326
------- 1446 --------
5922 2169 3426
2538* 2892 1142
4230* -------- 1713
---------
---------- 312336
186146
454302 ---------
13. AGE so, H+A+N+D=10 20. A P R
*OAT and 5(H-A)= 5(4-3)=5. *O C T
---------- -------------
SOAR 16. D-M-H PURA
HOG * A-N-P ROJR
__________________
GOTO RECU
HMDD
----------- -------------
HDZS
GECOIR RAAJAA
BMZP
Ans: 317 ___________________ What is the value of E?
*538 HHDBZD Ans: 432
------ 1)Find 2A+1 ? *567
2536 (a)7 (b)11 ---------
951 (c)13 (d)15 3024
1585 2592
---------- Ans: 493 2160
170546 *678 ----------
------------ 244944
14. A multiplication is given below 3944
where each letter represents a 3451 21. CGD
single digit and no two no.s are 2958 *BQS
represented by the same letter: ------------ ---------
WHY 334254 so 2A+1=13 ans is c OQSC
NUT GOSR
---------------- 17. (B E) (B E) = M O B QPOO
OONP Ans: 19 x 19 = 361 ---------
DSBRSC
OYPY
18. OUM Ans: 846
OUHA
x WUL 753
----------------- ----------
ONEPOP ----------
WWSO 2538
i)Which of these is not divisible by 4230
2? ILOI*
UUES** 5922
a)W b)Y c)U d)N ----------
-----------
637038
ii)One of those digits appear only USIITO
once in the multiplication except:
a)2 b)5 c)6 d)8 22. F Y H
Ans: 743
EWT
iii)Which of these can be sides of 649 ___________

a triangle? ------------ ZETF


a)NYT b)OWY c)EUO d)APW 6687 EKZZ
Ans: 234 2972* YZTX
*765 4458** ___________

------------ YFYWEF
--------- Ans: 846
1170 482207
573*
1404 ---------
1638 2538
---------- 5922*
179010 4230**
---------
484758

Pankaj Gandhi’s Academy


rd
3 Floor, Above Hotel Woodland, Near Sharda Center, Nal Stop, Pune. Contact No. : 8600972993, 9850424051.
23. B C H 27. B C H Ans: 846*537
*GEI *GEI -----------------
_____________ ___________ 5922
BADB BADB 2538
AAIJ AAIJ 4230
AFFF AFFF
______________ --------------
___________
AHJFDB 454302
AHJFDB
Ans: 2 3 8 * 7 5 9 32. HAT
-------------- *CUP
Ans: . 2 3 8
2142 ------
*759
1190 ___________ EIUI
1666 2142 EART-
--------------- 11 9 0 * EUPI--
180642 16 6 6 * * ----------
___________ HIEEEI
24. HE 180642
*IS Ans: 345
----- 28. ZINC+BCOZ=ANGRY 876
HUB A+N+G+R+Y=? ---------
OF 2070
----- Ans: 4532 2415
LAB 9274 2760
Ans: 14 -------- ---------
*59 13806 302220
--- (1+3+8+0+6) = 18
33. COW
126
29. RED *ONE
70
AS -----------
-----
826 -------- NNRC
ARCS OOVR
25. CGD RED DECD
*BQS -------- ------------
---------- CDTS WORONC
OQSC Ans: 7 4 3
GOSR Ans: 247
15 *4 6 9
QPOO ----------
--------
------------ 6687
1235
DSBRSC 4458
247
-------- 2972
Ans: 846*753
3705 -------------
-------------
348467
2538 30. WPD
4230 *GKI 34. ALE
5922 ------ x RUM
---------- KFPP ----------
637038 GGZM WINE
FGFI W UW L
------------ EWWE
26. F Y H GDWDFP
EWT -------------
___________ ERMPNE
Ans: 943
ZETF Ans:
678
EKZZ --------
605
YZTX 7544 987
___________
6601 ------------
YFYWEF 4235
Ans: 846 5658
-------- 4840*
573* 5445**
639354
--------- ----------------
2538 31. HMK*AVE
597135
5922* -----------------
4230** ANXX 35. B O W
--------- XAVH MEN
484758 MXVW -------------------
-------------- ESDD
MAMVWX ODMA
DEMB

Pankaj Gandhi’s Academy


rd
3 Floor, Above Hotel Woodland, Near Sharda Center, Nal Stop, Pune. Contact No. : 8600972993, 9850424051.
---------------- Ans: 432 Ans: 125
MADADD *567 x 37
Ans: 846 --------- ----------
357 3024 875
-------- 2592 375*
5922 2160 ----------
4230 ---------- 4625
-2538 244944
--------- ----------
302022 39. PLAYS
+WELL
36. W H Y --------
NUT BETTER
--------------
OONP Ans: 97426
OYPY +8077
OUHA ----------
---------------- 105503
ONEPOP
1) Which of these is not divisible
by 2 40. A G E
A) W B) Y C) U D) N *O A T
2) which of the fallowing can be ---------------
sides of a triangle SOAR
A) NYH B) OWY HOG
C) EUO D) APW GOTO
----------------
GECOIR
Ans: 234 Ans: 3 1 7
*765 *5 3 8
---------- ---------------
1170 2536
1404 951
1638 1585
----------- ---------------
179010 170546
37. D S P 42. SEND
LIE +MORE
------------
----------------
SPSS
PET A MONEY
Ans: 9567 + 1085 = 10652
PODE
------------
PNEADS 43. EAT
+ THAT
Ans: 543 ----------------
678 APPLE
---------- Ans: 819 + 9219 = 10038
4344
3801 44. NO
3258 GUN
---------- + NO
368154 ----------
HUNT
38. OTC Ans: 87 + 908 + 87 = 1082
. *HAS
--------------- 45. ABC
TICO x DE
CHKC. ----------
CLAI.. FEC
--------------- DEC
COOKOO ----------
HGBC

Pankaj Gandhi’s Academy


rd
3 Floor, Above Hotel Woodland, Near Sharda Center, Nal Stop, Pune. Contact No. : 8600972993, 9850424051.
English Sample Paper 1
Directions(Question 1-8): Identify the correct sentences.
1. (a) Clinical Research is fast emerging as a rewarding career in India.
(b) Clinical Research is emerging fastly as a rewarding career of India.
(c) Clinical Research will be fast emerging as a rewarding career for India.
(d) Clinical Research is emerging faster as a rewarding career with India.

2. (a) Beauty products and luxury items is always been endorsed by women.
(b) Beauty products and luxury items is always being endorsed by women.
(c) Beauty products and luxury items have always been endorsed by women.
(d) Beauty products and luxury items has always been endorsed by women.

3. (a) This world is being aflamed with anger that leads to conflict, crime, violence and bloodshed.
(b) This world is aflame with anger that leads to conflict, crime, violence and bloodshed.
(c) This world is aflame with anger that had led to conflict, crime, violence and bloodshed.
(d) This world will be aflamed with anger which is leading to conflict, crime, violence and bloodshed.

4. (a) The food guarantee act will be a powerful tool in achieving the goal of a hunger –free India.
(b) The food guarantee act will be a powerful tool to achieve the goal of a hungry –free India.
(c) The food guarantee act will be a powerful tool in achievement of the goal of a hunger –free India.
(d) The food guarantee act will be a powerful tool for achieving the goal in a hunger –free India.

5. (a) Since its inception digital photography was gained a lot of popularity.
(b) Since its inception, digital photography has gained immense popularity.
(c) Since its inception digital photography has gained abundance in popularity.
(d) Since its inception digital photography has gained humongous popularity.

6. (a)Experts maintain that an anti- wrinkle cream merely hides ugly wrinkles and moisturizes the
skin.
(b)Experts are maintaining that an anti- wrinkle cream nearly hides ugly wrinkles and will moisturize
the skin.
(c) Experts should maintain that an anti- wrinkle cream merely is hiding ugly wrinkles and moisturizes
the skin.
(d) Experts maintain that an anti- wrinkle cream merely is hiding ugly wrinkles and moisturizes the
skin.

7. (a) Education will be imparted through lecturing, demonstrating and field visits to disaster –affected
areas.
(b) Education will be imparted through lectures, demonstrations and field visits to disaster –affected
areas.
(c) Education will be imparting by lectures, demonstrations and field visits to disaster –
affected areas.
(d) Education will be imparted with lectures, demonstrations and field visiting to disaster –affected
areas.

8. (a) Plants are going a long way in providing visual stimulation and comfort.
(b) Plants go a long way with providing visual stimulating comfort.
(c) Plants go a long way in providing visual stimulation and comfort.
(d) Plants will be going a longer way in providing visual stimulation and comfort.

Directions(Questions 9-16): In each of the following sentences, some part of the sentence or the entire
sentence is underlined. Beneath each sentence you will find four ways of phrasing the underlined part.

Pankaj Gandhi’s Academy


rd
3 Floor, Above Hotel Woodland, Near Sharda Center, Nal Stop, Pune. Contact No. : 8600972993, 9850424051.
This is a test of correctness and effectiveness of expression. In choosing answers, follow the
requirements of standard written English, that is pay attention to grammar, choice of words, and
sentence construction. Choose the answer that expresses most effectively what is presented in the
original sentence; this answer should be clear and exact, without awkwardness, ambiguity or
redundancy.

9. The cut throat competition in the global skies as well as the continued spiral in oil prices have finally
taken a toll on India’s largest airline.
(a) have taken finally a toll of India’s (c) have finally taken a toll in India
(b) has finally taken a toll on India’s (d) have finally taken a toll to Indian

10. Prospects of a bumper Kharif food grain production has brightened following heavy rains in the
country.
(a) have brightened , following (b) have followed brightly
(c) has a brightened following (d) follows brightly

11. On July 11, just a little while after attacks rocked Srinagar, serial blasts killed round 200 persons in
Mumbai.
(a) attacks was rocking Srinagar, serial blasts killed around
(b) attacks rocked Srinagar, serial blasts had killed around
(c) attacks had rocked Srinagar, serial blasts killed around
(d) attacks had rocked Srinagar, serial blasts had killed around

12. Already famous for angry labor strikes and philosophical debates in smoke- filled cafes, passions
have now been brought online by the French.
(a) these passions are now being brought online by the French.
(b) these passions will now be brought online by the French.
(c) these passions should now be brought online by the French.
(d) the French have now brought these passions online.

13. Similar to an iceberg, nine-tenths of our malnutrition, the most dangerous part, are below the surface.
(a) are under the surface. (b) Is below the surface.
(c) happen under the surface. (d) Will be below the surfaces.

14. Response to some questions generally come from prescriptions of moral observance of the laws.
(a) come from prescriptions on (b) comes from prescribing of
(c) comes from prescriptions in (d) comes from prescriptions of

15. Stanford Business School had a spirit of ethos that I never do figure out.
(a) that I never did figure out. (b) that I had never figured on.
(c) I have never figured. (d) that I never do figured out.

16. Placement opportunities were greater for students of the Alpha Business School than Merit business
school.
(a) than that of merit Business school. (b) than for those of Merit Business School.
(c) than for those in Merit Business School. (d) than those in Merit Business School.

Directions(Questions 17-24): Fill in the blank with the correct option that fits in grammatically and
logically.
17. Innovation becomes a crucial success factor that has a remarkable effect on a company’s growth,
profitability and _________.
(a) valuation (b) Manipulation (c) verification (d) Restriction

18. Macromedia Flash Player’s ability to run on all platforms makes it the ________ of choice when it
comes to presenting information.
(a) backlash (b) front end (c) right end (d) left end

Pankaj Gandhi’s Academy


rd
3 Floor, Above Hotel Woodland, Near Sharda Center, Nal Stop, Pune. Contact No. : 8600972993, 9850424051.
19. With the prevailing problem of ever tight budgets, it is an enormous advantage if a hospital can free
its staff from as many administrative and _________ tasks as possible so that they can concentrate on
caring for patients.
(a) Logical (b) Logistical (c) Experimental (d) Coordinative

20. Given the implications and the signals that the uncertainty of this litigation can send, it is vital for the
government to take a ___________ and bring an end to this confusion.
(a) Pessimistic view (b) Skeptical view (c) Consolidated view (d) Pragmatic view

21. As marriages, jobs, homes and even friends appear at the click of a mouse, there is an ________ in
newspaper companies.
(a) unauthentic frustration (b) unavoidable circumstance (c) uncomfortable rustling
(d) unusual spending

22. If manufacturing of liquor is legally allowed, then _________ of liquor should also be made legal.
(a) promotion (b) production (c) procurement (d) enticement

23. Since the entire team was able to meet its targets much in advance, the manager announced a 20%
increase in the compensation __________ for all the team-mates.
(a) Along the board (b) Against the board (c) Across the board (d) Above board

24. Every time there is a severe thunderstorm, low-lying streets ________.


(a) are flooded by (b) are flooding with (c) flood with (d) are flooded with

Directions(Questions 25-30): For each question in this section, select the best of the answer choices
given.
25. It is observed that people living in the desert prefer to live in while tents and wear white clothes.
Arabians mostly wear long, loose, white robes. Even women wear white veils rather than coloured ones.
Black is rarely used in Arabia.
Which of the following statements can be deduced from the above?
(a) Black is considered evil and hence taboo in Arabia. (b) The colour white brings peace and
prosperity.
(c) Arabians are known to dress comfortably. (d) White is preferred as it reflects heat.

26. Direction, an NGO formed to rehabilitate street children has been doing a lot good work. Children
who were once beggars and drug addicts on the streets are picked up, given a home and trained to
become economically independent and socially useful. Some children learnt carpentry, other learnt
plumbing, but most of them took up painting as a profession.
Which of the following statements could be true?
(a) Painting provides a psychological release for the children.
(b) It was found that painting pays maximum money.
(c) Paint contains toluene and most of the children were once addicted to inhaling toluene.
(d) Most of the children are artistic.

27. The humble fluorescent tube has come a long way indeed. It replaced the ordinary light bulb
because fluorescent lams emit such a bright glow without getting scalding hot like an ordinary light bulb.
The fluorescent lamp is also recommended by energy economists.
Which of the following statements is true?
(a) The humble fluorescent tube is the best lighting available.
(b) Edison’s light bulb has failed miserably.
(c) The fluorescent tube consumes a lot less power.
(d )Most burns cases are due to the scalding hot light bulb.

Pankaj Gandhi’s Academy


rd
3 Floor, Above Hotel Woodland, Near Sharda Center, Nal Stop, Pune. Contact No. : 8600972993, 9850424051.
28. The entire Raymond family fell sick and was diagnosed with chemical poisoning. The doctors treating
them tried to record the food consumed by them before falling sick. The Raymond’s said that they did not
consume anything unusual except a condensed milk cake.
Which of the following statements is true?
(a) The condensed milk was poisoned.
(b) The Raymond’s were lying.
(c) The Raymond’s did not have chemical poisoning but food poisoning.
(d) The condensed milk was stored in a tin that caused the poisoning.

29. Silk is a natural protein fiber that can be woven into textiles. It is obtained from the cocoon of
silkworm larvae reared in captivity. In addition to clothing manufacture and other handicrafts, silk is also
used for items like parachutes, bicycle tires, comforter filling and artillery gunpowder bags. Early
bulletproof vests were also made from silk in the era of black powder weapons until roughly World War I.
Silk undergoes a special manufacturing process to make it adequate for its use in surgery as non-
absorbable sutures. Chinese doctors have also used it to make prosthetic arteries. Silk cloth is also used
as a material to write on. Silk has recently come under criticism from some animal rights activists.
Which of the following statements is true?
(a) Animal rights activists are becoming a nuisance.
(b) The silkworms are boiled alive in their cocoons to spin silk.
(c) Animal rights activists do not appreciate the splendour of silk.
(d) Animal rights activists have nothing better to do than protest against everything.

30. Neurologists across Bangalore city are alarmed at the increasing incidence of epileptic fits. They
have advised most of their patients to stay away from leafy vegetables like cabbage, spinach etc and
vegetables like cauliflower.
Which of the following statements is true?
(a) Vegetables cause giddiness and nausea. (b) Bangaloreans are under a lot of tension.
(c) Leafy vegetables harbor a worm that causes a shadow in the brain. (d) Neurologists have a
habit of exaggerating.

Directions(Questions 31-35): Read the following passage below and answer the questions that follow
on the basis of what is stated / implied in that passage.
Green coffee beans have supplied a new player in the antioxidant arena. An extract of green
coffee beans has been found to have a stronger antioxidant effect than established antioxidants like
green tea and grape seed extract.
The active constituent in coffee that is responsible for its many health benefits is a compound
called chlorogenic acid. It neutralizes free radicals, and addresses the problem of hydroxyl radicals, both
of which can lead to cellular degeneration if left unchecked. Chlorogenic acid also helps regulate
metabolism. Compared to green tea and grape seed extract, green coffee bean extract coffee bean
extract is twice as effective in absorbing oxygen free radicals. One of the advantages of using the green
coffee bean extract is that the negative effects of coffee are avoided. The chlorogenic acid is thought to
boost metabolism by changing the way glucose is taken up by the body. And it does contain caffeic
acids, which give a boost to energy levels like regular coffee does. But unlike boiled coffee, green coffee
bean extract contains no cafestol, which is a diterpene. Along with its diterpene relative kahweol, cafestol
increases concentrations of the ‘bad’ cholesterol, LDL, to levels that over a lifetime might increase the
risk of coronary heart disease by as much as 20%. These diterpenes also had an effect on the levels of
liver enzymes measured. When these are elevated it is an indicator of stress on the liver. However the
study that measured this found this was a transient effect, and also that the levels of liver enzymes were
much lower than those with liver disease.
As a side note on the health effect of the diterpenes found in regular coffee, it was found that by
simply drinking filter coffee, none of these effects on cholesterol levels or the liver took place. The coffee
filter removed the offending diterpenes. And levels of these diterpenes in instant coffee are low.
Other benefits of green coffee bean extract include an increase in the effectiveness of pain
killers, especially for migraine medications; a reduction in the risk of diabetes; and assisting the body
burn a higher proportion of lipids (fats) compared to carbohydrates, which could help with muscle fatigue
for athletes and bodybuilders.

Pankaj Gandhi’s Academy


rd
3 Floor, Above Hotel Woodland, Near Sharda Center, Nal Stop, Pune. Contact No. : 8600972993, 9850424051.
Interestingly, on the subject of caffeine and liver disease, further studies have indicated it may in
fact support liver health for some people. Those who were at high risk of developing liver disease due to
drinking too much alcohol were found less likely to suffer liver damage if they drank more than two cups
of coffee or tea a day. This was a population based study, not a clinical trial, and so is not conclusive on
the subject. But it does offer some promising information. Those drinking in excess of two cups or more
a day were half as likely to develop liver disease compound to those drinking less than one cup a day.
Researchers do not know what caused this protective effect.
One of the criticisms of coffee in regards to health is that it leaches calcium from the bones. But
this effect has been found to be overemphasized, at least in children. And adults who consume a diet
with sufficient levels of calcium will be protected from the small amount of calcium that is lost due to
coffee consumption.
So the old axiom that caffeine can stunt a child’s growth is a myth. It was based on the fact that in
order studies, caffeine was associated with low bone mass because those studies were done on elderly
people who drank a lot of coffee and had diets that were low in calcium. Recent studies in the US
followed 80 teenagers over 6 years, and found no difference in the bone density of those with a high
level of caffeine in their diet, compared to those teenagers who had little caffeine. Other studies
determined that the amount of calcium lost from bones is small and can be balanced by having sufficient
calcium in your diet.

31. Which of the following benefits can be attributed to green coffee beans?
(a) Green coffee beans can alleviate pain. (b) Green coffee beans can help alcoholics give up the
alcohol addiction.
(c) Green coffee beans increases insulin efficiency. (d) Green coffee beans help absorb harmful
free radicals.

32. How does the passage debunk the old axiom that caffeine can stunt a child’s growth?
(a) It reverses the axiom by emphasizing that caffeine in fact prevents osteoporosis.
(b) Older studies were done on elderly people who had low calcium diets.
(c) Coffee actually adds to the calcium content in the body.
(a) a only (b) b only (c) a and c (d) b and c

33. What is the commonly over-hyped complaint about the effects of coffee?
(a) Coffee decalcifies the bones. (b) Coffee causes fluctuating blood glucose levels.
(c) Caffeine causes liver cancer. (d) Coffee increases the risk of
cancer.

34. ‘Green coffee beans have supplied a new player in the antioxidant arena.’ What is implied in this
statement?
(a) Green coffee beans are just another drop in the ocean of antioxidants.
(b) It has just been discovered that green coffee beans also contain beneficial antioxidants.
(c) Researchers are trying to put together a hierarchical list of beneficial antioxidants.
(d) All of the above.

35. How do the components of boiled coffee beans impact the body?
(A) The chlorogenic acid leads to low energy levels.
(B) Cafestol increases the concentration of bad cholesterol in the body.
(C) Diterpenes increase the liver enzymes level that leads to liver stress.
(a) A only (b)B only (c) A and C (d) B and C

Directions(Questions 36-40): Read the following passage below and answer the questions that follow
on the basis of what is stated / implied in that passage.

What was the secret of Gandhi’s popularity? Indian politics in the year 1915 was at a standstill.
Neither the moderates nor the extremists were making any real impact on the people. But at the same
time, as a result of the Swadeshi movement of 1905 and Home Rule League activities later, politics was

Pankaj Gandhi’s Academy


rd
3 Floor, Above Hotel Woodland, Near Sharda Center, Nal Stop, Pune. Contact No. : 8600972993, 9850424051.
moving into ever widening outer rings. Newspapers and pamphlets were carrying the nationalist
message beyond the metropolitan cities and active groups were being formed in the districts. Gandhi
himself utilized the network laid out by the Home Rule League. However, outside the closed ring of
gentlemen’s politics, there was a large section of half-educated and illiterate people with no one to lead
them. Various groups in the backward regions felt threatened by the changes occurring under Western
impact. Gandhi stepped in to bridge the gap between the English-educated politicians and the less
educated mass of common men and women. The people of the outer orbit were attracted to Gandhi
because he was so different from the English-educated leaders. His simple attire, preference for
swadeshi goods and his overt religiosity made him more acceptable to the common people. But the
leaders found it difficult to accept him. Why then did the established leaders despite their earlier
misgivings about him accept Gandhi in 1921?
As already seen, Gandhi first began his experiments in local areas like Champaran, Kheda and
Ahmedabad. Those activities brought him countrywide fame as a powerful leader. It was through the
Rowlatt Satyagraha that he made his entry into national politics. Gandhi effectively utilized the Khilafat
issue and the Punjab “wrong” to advance his political standing.
The acceptance of Gandhi into mainstream politics was not complete and unquestioned. The
national leaders were keenly aware of the fact that Non-cooperation would jeopardize council entry. Yet
they finally yielded to Gandhi and accepted his plan of action because circumstances forced their hands.
Gandhi’s intervention in Indian politics since the Rowlatt Satyagraha emerged as the all-important factor.
Since then it seemed to determine the course of mainstream politics. How does one explain the events
of 1919-22? Perhaps it was because of expectation. There was expectation in the air. The public
expected from him a miracle. In 1920-21, all eyes were fixed on Gandhi. The promise of swaraj in one
year cast a spell. Even the revolutionaries were willing to suspend activity for a year. Gandhi’s immense
popularity had made him almost irresistible. The government did not know what to do with him. At the
same time he was an embarrassment to some of the Indian leaders. He was not planning to capture the
Congress and turn it to his will but his attitude changed sharply after the release of the Hunter
Commission report in May 1920 on the Punjab incidents. This was something, which Gandhi could not
bear. He came out openly against council entry. The die was cast. Gandhi made up his mind to launch
Satyagraha. Gandhi knew that he had to take the Congress along with him. To get the Congress to pass
the right resolutions was a practical problem. Gandhi overcame this by using the Khilafat plank and the
Punjab atrocities. He thus became the supreme leader.
But at the same time it must be admitted that Gandhi’s calculations went wrong. Satyagraha did
not bring about swaraj in one year. People did not remain non-violent. The Hindu-Muslim entente did not
take place. When violence erupted, Gandhi suspended the movement and finally called it off to the great
dismay of his followers and onlookers. Gandhi’s action came in for attack from all sides. It seemed that
the Gandhian style of politics was discredited though not altogether discarded. After the collapse of the
Non-cooperation movement, the unity of mainstream politics was broken. Non-cooperation failed, but it
broke the spell of fear of British authority among the common people. The British in India had lost the
mandate of history.

36. When the Indian Politics was at a standstill in the year 1915, how was it moving into ever widening
outer rings?
(a) As the moderates and extremists were not making any impact on the people
(b) As a result of the swadeshi movement and the Home rule league activities
(c) As a result of the groups being formed in the districts
(d) Newspapers and pamphlets carried the Nationalist message beyond the metropolitan cities

37. How did Gandhi enter into National politics?


(a) he utilized the network laid out by the home rule league
(b) he bridged the gap between the English – educated politicians and the common man
(c) because of his country wide fame as a powerful leader
(d) through the Rowlatt Satyagraha

38. Why did Gandhi make up his mind to launch Satyagraha?


(a) he could not bear the release of the Hunter commission report
(b) his immense popularity made him almost irresistible

Pankaj Gandhi’s Academy


rd
3 Floor, Above Hotel Woodland, Near Sharda Center, Nal Stop, Pune. Contact No. : 8600972993, 9850424051.
(c) the public expected a miracle from him
(d) he knew that he had to take the congress along with him

39. How did Gandhi become the Supreme leader?


(a) as he had promised swaraj in one year
(b) by getting the congress to pass the right resolutions using the Khilafat plank and the Punjab atrocities
(c) because of his intervention in Indian Politics after the Rowlatt Satyagraha emerged
(d) As he effectively utilized the Khilafat issue and the Punjab atrocities

40. Gandhi’s followers and onlookers were dismayed for which of the following reasons?
(a) his suspension of Satyagraha, as it did not bring about swaraj in on year
(b) when he became a supreme leader
(c) the promise of swaraj in one year
(d) as he broke the spell of fear of British authority among the common people

English Sample Paper 2


Directions(Questions 1-5): Read the passage and answer the questions that follow on the basis of the
information provided in the passage
A snow leopard roars in the high mountains of Asia. A black rhinoceros gallops across the plains of Africa.
A grizzly bear hunts for fish in a North American river. A mother blue whale and her calf glide through the deep
waters of the ocean. All of these animals share the Earth with us. They fascinate us with their beauty, their grace,
and their speed. We love observing their behaviour, and learning more about their habits. But just loving them is
not enough. All of these animals are endangered. Many of them have died, and without special care, they may
someday disappear from the Earth.
Why is it important to care for animals like these? One reason is to protect the balance of life on Earth.
Another reason is the beauty of the animals themselves. Each species of animal is special. Once it is gone, it is
gone forever.
Africa was once filled with an abundance of wild animals. But that is changing fast. One of these animals,
the black rhinoceros, lives on the plains of Africa. It has very poor eyesight and a very bad temper! Even though the
black rhino is powerful, and can be dangerous, its strength can’t always help it to escape hunters. Some people
think that the rhino’s horn has magical powers, and many hunters kill rhinos for their valuable horns. This has
caused the black rhino to be placed on the endangered species list.
The elephant seems to represent all that is strong and wild in Africa. It once had no natural enemies, but is
now endangered-killed for its ivory tusks.
The fastest land animal, the cheetah, also lives in Africa. It, too, is becoming extinct as people take over
more and more of the land that is the cheetah's natural habitat.
Imagine Africa without the powerful rhino, the gentle, intelligent elephant, or the lightning quick cheetah.
Once they are gone, they are gone forever.
Wherever people are careless about the land, there are endangered species. Grizzly bears like to wander
great distances. Each bears needs up to 1,500 square miles of territory to call its homeland. Today, because
forests have been cleared to make room for people, the grizzly’s habitat is shrinking and the grizzly is disappearing.
It joins other endangered North American animals, such as the red wolf and the American crocodile.
In South America, destruction of the rain forest threatens many animals. Unusual mammals, such as the
howler monkey and the three-toed sloth, are endangered. Beautiful birds like the great green macaw and the
golden parakeet are also becoming extinct. They’re losing their homes in the rain forest, and thousands die when
they are caught and shipped off to be sold as exotic pets.
The giant panda of Asia is a fascinating and unique animal. Yet there are only about 1,000 still living in the
wild. The giant panda’s diet consists mainly of the bamboo plant, so when the bamboo forests die, so does the
panda. China is now making an effort to protect these special creatures from becoming extinct.
Asia’s big cats are also in trouble. The exotic snow leopard lives high in the mountains. Even there, it faces
the loss of its natural habitat, and hunters who kill it for its fur. The tiger, the largest of all the big cats, is hunted
merely for sport.
Ocean-dwelling animals are in danger of extinction as well. The blue whale is the largest animal in the
world. It weighs up to 390,000 pounds. Whale hunting and pollution are this species’ greatest enemies.
Unfortunately, it is people who cause many of the problems that animals face. We alter and pollute their habitats.
We hunt them for skins, tusks, furs, and horns. We destroy animals that get in the way of farming or building. And
we remove them from their natural habitats and take them home as pets.

Pankaj Gandhi’s Academy


rd
3 Floor, Above Hotel Woodland, Near Sharda Center, Nal Stop, Pune. Contact No. : 8600972993, 9850424051.
What can you do to help endangered animals? Learn as much as you can about them. The more you
know, the more you can help. Make an effort to support zoos and wildlife groups. Many zoos breed endangered
animals, helping to ensure that they will continue to live on. Contribute to groups, such as the National Wildlife
Federation and the Sierra Club, that work hard to protect animals. You can also be a smart shopper and never buy
a pet that has been raised in the wilderness.
The world is made up of many living things, and each thing is dependent on the others to survive. If we
allow even one species on Earth to become extinct, it has an impact on other living things and changes our world.
When we mention any endangered wild animals, let’s hope that we never again have to say, "Gone forever."
1. Which of the following is NOT an opinion?
(a) It is important for us to take care of endangered animals.
(b) The black rhinoceros is the most frightening animal in Africa.
(c) The fastest land animal, the cheetah, also lives in Africa.
(d) If we don’t protect endangered animals now, we will regret it in the future.

2. Read this sentence from the article. Today, because forests have been cleared to make room for people, the
grizzly’s habitat is shrinking and the grizzly is disappearing.
What does habitat mean?
(a) an animal’s natural environment (b) a bear’s thick wool coat (c) a bear’s den (d) an animal’s
vision

3. Which of the following does NOT support the main idea of the article?
(a) Ocean-dwelling animals are in danger of extinction as well.
(b) Beautiful birds like the great green macaw are also becoming extinct.
(c) Wherever people are careless about the land, there are endangered species.
(d) The elephant seems to represent all that is strong and wild in Africa.

4. Read this sentence from the article. They fascinate us with their beauty, their grace, and their speed.
What is an antonym for the word fascinated?
(a) thrill (b) bore (c) scare (d) humor

5. What will happen to some animal species if current trends continue?


(a) Their numbers will increase. (b) There will be no significant change.
(c) They will become extinct. (d) They will move to other habitats

Directions(Questions 6-10): Read the passage and answer the questions that follow on the basis of the
information provided in the passage
The last half of my life has been lived in one of those painful epochs of human history during which the
world is getting worse, and past victories which had seemed to be definitive have turned out to be only temporary.
When I was young, Victorian optimism was taken for granted. It was thought that freedom and prosperity would
speed gradually throughout the world by an orderly process, and it was hoped that cruelty, tyranny, and injustice
would continually diminish. Hardly anyone was haunted by the fear of great wars. Hardly anyone thought of then in
eighteenth century as a brief interlude between past and future barbarism.
6. The author feels and about the later part of his life because
(a) The world had not become prosperous (b) He was nostalgic about his childhood.
(c) The world was painfully disturbed during that period of time (d) The author had not won any further
victories.

7. The victories of the past


(a) Filled men with a sense of pessimism (b) Proved to be temporary events
(c) ended, cruelty, tyranny, and injustice (d) Brought permanent peace and security

8. The world 'definitive' as used in the passage means


(a) Incomplete (b) defined (c) temporary (d) final

9. During the Victorian age people believed that


(a) There would be unlimited freedom (b) Strife would increase
(c) Peace would prevail and happiness would engulf the whole world. (d) Wars would be fought on a
bigger scale

10. A brief interlude between past and future barbarism' can be interpreted as
(a) A dramatic performance during wars (b) An interval between cruel wars

Pankaj Gandhi’s Academy


rd
3 Floor, Above Hotel Woodland, Near Sharda Center, Nal Stop, Pune. Contact No. : 8600972993, 9850424051.
(c) A short space of time between two great events
(d) A short period of time between past and future acts of savagery.

Directions(Questions 11-18): Pick out the most effective word from the given words to fill in the blank to make the
sentence meaningfully complete.
11. The petition before the Court prayed for .....................the appointment orders issued by the management
(a) Removing (b) granting (c) posting (d) quashing (e) dismissing

12. I have a profound respect for his political..................


(a) Personality (b) ambition (c) temperament (d) sagacity (e) involvement

13. The truck was.............. The traffic and the policemen asked the driver to move off.
(a) Failing (b) obstructing (c) obviating (d) hiding (e) disturbing

14. The paternalistic attitude is so ingrained in the managements that they have to.....................try to change it
(a) Casually (b) slowly (c) subtly (d) inadvertently (e)
conscientiously

15. For a few seconds, Madan was.............blinded by the powerful lights of the oncoming car
(a) Heavily (b) largely (c) greatly (d) powerfully (e) totally

16. Sachin was to reach that afternoon but was ...... up at Delhi for some personal work.
(a) delayed (b) kept (c) held (d) stayed (e) detained

17. He..... his shoes till they shone.


(a) brushed (b) scrubbed (c) wiped (d) polished (e) shined

18. Let us take him upon his word. I see no room for any ....... in the matter
(a) Deviation (b) duplicity (c) difference (d) cynicism

Direction(Questions 19-26): In each question below is given a passage followed by several inferences. You have
to examine each inference separately in the context of the passage and decide upon its degree of truth or falsity.
Mark your answer as:
(a) if the inference is ' definitely true' i.e., it directly follows from the facts given in the passage
(b) if the inference is ' probably true' though not definitely true in the light of the facts given
(c) if you think the data are in adequate i.e., from the facts given you cannot say whether the inference is likely to
be true or false
(d) if you think the inference is ' probably false' though not definitely false in the light of the facts given; and
(e) if you think inference is ' definitely false' i.e., it contradicts the given facts

Passage I
More than a decade of erosion in budgetary support from the Union Government, has seriously affected Indian
Railway's capacity to finance its plan expenditures. The situation has come to a pass where the railways must now
think of innovative ways to get longer mileage from its investments. Significantly the resource crunch has had
grievous impact on the railways. As a result, it will not be in a position to acquire necessary equipments and this will
seriously affect the railway's capacity to serve the needs of the economy in future.
19. Railways had so far believed in traditional ways in generating income. Ans B
20. Government has shifted its priority from railways to other areas. Ans: B
21.The union government has reduced drastically the budgetary support to railways during the last decade. Ans: B
22. The fiscal position of railways in the earlier plan period was better than the current plan period Ans: A
23. During the current plan period, the railways will not be able to expand its network Ans: A

Passage II
The smaller pesticide formulation units in India operate under heavy constraints such as obsolete technology, small
scale of operation and dependence on large units for raw materials. In view of the loss of expensive material by the
smaller units it is important to either eliminate or reduce losses to the extent possible through innovative and
sustainable waste minimization techniques. Operating profit margins of the units are very low and small adverse
conditions land these companies in trouble. Maximum losses suffered by these units are through poor house
keeping, sub- optional operating practices, and lack of proper opportunities for recycling waste.
24. Smaller units should be operationally self sufficient so as to minimize loss. Ans: A
25. Waste management process in India need modernization Ans: B

Pankaj Gandhi’s Academy


rd
3 Floor, Above Hotel Woodland, Near Sharda Center, Nal Stop, Pune. Contact No. : 8600972993, 9850424051.
26. Lack of funds compels smaller units to ignore house keeping. Ans: B

Direction(Questions 27-32): In each of the following questions, find out which part of the sentence has an error. If
there is no mistake the answer is 'no error'
27. Our is the only / country in the world / than can boast of / unity in diversity / no error
a b c d e

28. May I know/who you want / to see please / No error


a b c d

29. Due to me being a newcomer / I was unable to get a house / suitable for my wife and me / No error
a b c d

30. This is the boy / whom I think had won / the gold medal in the dance competition / No error
a b c d

31. The number of vehicles /plying on this road /is more than on the main road / No error
a b c d e
32. There has not been any rainfall / in this part of the country / since the last two years / No error
a b c d

Direction(Questions 33-40): In each question, a part of sentence is printed in bold. Below each sentence, some
phrases are given which can substitute the italicized part of the sentence. If the sentence is correct as it is, the
answer is 'No correction required'
33. No sooner I saw the tiger than I ran away
(a) No sooner I had seen (b) No sooner did I see (c) as soon as I saw (d) No Correction
required

34. Let he and I work together


(a) him and I (b) him and me (c) he and me (d) No Correction
required

35. The child is father of the man


(a) A child, man (b) Child, a man (c) Child, Man (d) NoCorrection required

36. I am used to hard work


(a) hard working (b) work hard (c) work hardly (d)No Correction
required

37. The doctor examined my pulse


(a) observed (b) saw (c) felt (d)No Correction
required

38. I wish I have all those luxuries


(a) I will have (b) I am having (c) I had (d) No Correction
required

39. She did not like the movie, nor I did


(a) nor did I (b) nor I like it (c) nor I liked it (d) No Correction
required

40. The enthusiastic voters were queuing up outside the polling station since morning.
(a) queued up (b) Had queued up (c) had been queuing up (d) No Correction
required

Directions(Questions 41-45): Read the passage and answer the questions that follow on the basis of the
information provided in the passage.
Much of the information we have today about chimpanzees comes from the groundbreaking, long-term
research of the great conservationist, Jane Goodall.
Jane Goodall was born in London, England, on April 3, 1934. On her second birthday, her father gave her
a toy chimpanzee named Jubilee. Jubilee was named after a baby chimp in the London Zoo, and seemed to

Pankaj Gandhi’s Academy


rd
3 Floor, Above Hotel Woodland, Near Sharda Center, Nal Stop, Pune. Contact No. : 8600972993, 9850424051.
foretell the course Jane’s life would take. To this day, Jubilee sits in a chair in Jane’s London home. From an early
age, Jane was fascinated by animals and animal stories. By the age of 10, she was talking about going to Africa to
live among the animals there. At the time, in the early 1940s, this was a radical idea because women did not go
to Africa by themselves.
As a young woman, Jane finished school in London, attended secretarial school, and then worked for a
documentary filmmaker for a while. When a school friend invited her to visit Kenya, she worked as a waitress until
she had earned the fare to travel there by boat. She was 23 years old.
Once in Kenya, she met Dr. Louis Leakey, a famous paleontologist and anthropologist. He was impressed
with her thorough knowledge of Africa and its wildlife, and hired her to assist him and his wife on a fossil-hunting
expedition to Olduvai Gorge. Dr. Leakey soon realized that Jane was the perfect person to complete a study he
had been planning for some time. She expressed her interest in the idea of studying animals by living in the wild
with them, rather than studying dead animals through paleontology.
Dr. Leakey and Jane began planning a study of a group of chimpanzees who were living on the shores of
Lake Tanganyika in Kenya. At first, the British authorities would not approve their plan. At the time, they thought it
was too dangerous for a woman to live in the wilds of Africa alone. But Jane’s mother, Vanne, agreed to join her so
that she would not be alone. Finally, the authorities gave Jane the clearance she needed in order to go to Africa
and begin her study.
In July of 1960, Jane and her mother arrived at Gombe National Park in what was then called Tanganyika
and is now called Tanzania. Jane faced many challenges as she began her work. The chimpanzees did not accept
her right away, and it took months for them to get used to her presence in their territory. But she was very patient
and remained focused on her goal. Little by little, she was able to enter their world.
At first, she was able to watch the chimpanzees only from a great distance, using binoculars. As time
passed, she was able to move her observation point closer to them while still using camouflage. Eventually, she
was able to sit among them, touching, patting, and even feeding them. It was an amazing accomplishment for
Jane, and a breakthrough in the study of animals in the wild. Jane named all of the chimpanzees that she studied;
stating in her journals that she felt they each had a unique personality.
One of the first significant observations that Jane made during the study was that chimpanzees make and
use tools, much like humans do, to help them get food. It was previously thought that humans alone used tools.
Also thanks to Jane’s research, we now know that chimps eat meat as well as plants and fruits. In many ways, she
has helped us to see how chimpanzees and humans are similar. In doing so, she has made us more sympathetic
toward these creatures, while helping us to better understand ourselves.
The study started by Jane Goodall in 1960 is now the longest field study of any animal species in their
natural habitat. Research continues to this day in Gombe and is conducted by a team of trained Tanzanians.
Jane’s life has included much more than just her study of the chimps in Tanzania. She pursued a graduate degree
while still conducting her study, receiving her Ph.D. from Cambridge University in 1965. In 1984, she received the
J. Paul Getty Wildlife Conservation Prize for "helping millions of people understand the importance of wildlife
conservation to life on this planet." She has been married twice: first to a photographer and then to the director of
National Parks. She has one son.
Dr. Jane Goodall is now the world’s most renowned authority on chimpanzees, having studied their
behavior for nearly 40 years. She has published many scientific articles, has written two books, and has won
numerous awards for her groundbreaking work. The Jane Goodall Institute for Wildlife Research, Education, and
Conservation was founded in 1977 in California but moved to the Washington, D.C., area in 1998. Its goal is to take
the actions necessary to improve the environment for all living things.
Dr. Goodall now travels extensively, giving lectures, visiting zoos and chimp sanctuaries, and talking to
young people involved in environmental education. She is truly a great conservationist and an amazing human
being.
Read this sentence from the article.
41. 'But she was very patient and remained focused on her goal'. What is an antonym for the word focused?
a) bothered (b) tired (c) disinterested (d) concerned

42. What is the author’s purpose in writing this article?


(a) to entertain the reader with stories about chimpanzees
(b) to inform the reader of the importance of wildlife conservation
(c) to warn the reader about the challenges of working in africa
(d) to describe the work and life of Jane Good all.

43. Which of the following is NOT one of the reasons Dr. Leakey chose Jane to work with him?
(a) She knew a lot about Africa. (b) She knew a lot about African wildlife
(c) She earned the money to travel to Africa on her own (d) She was interested in studying animals in the
wild

Pankaj Gandhi’s Academy


rd
3 Floor, Above Hotel Woodland, Near Sharda Center, Nal Stop, Pune. Contact No. : 8600972993, 9850424051.
44. Which of the following is NOT true of chimpanzees?
(a) Chimpanzees are often comfortable with strangers right away.
(b) Chimpanzees eat meat as well as plants and fruit.
(c) Chimpanzees use tools to help them get food.
(d) Different chimpanzees have different personalities.

45. Jane Goodall is now the world’s most renowned authority on chimpanzees, having studied their behavior for
nearly forty years. What does authority mean?
(a) an intelligent person (b) one who studies animals (c) a scientist (d) an expert

Directions(Questions 46-53): Pick out the most effective word from the given words to fill in the blank to make
the sentence meaningfully complete.
46. For a few seconds, Madan was.............blinded by the powerful lights of the oncoming car
(a) Heavily (b) largely (c) greatly (d) powerfully (e) totally

47. His interest in the study of human behavior is indeed very..............


(a) Strong (b) large (c) broad (d) vast (e) deep

48. The police have................a complaint against four persons


(a) Entered (b) lodged (c) registered (d) noted (e) received

49. The improvement made by changes in the system was ....................and did not warrant the large expenses.
(a) Large (b) small (c) minute (d) marginal (e) uncertain

50. The man who is..........................hesitating which of the two things he will do first, will do neither.
(a) Persistently (b) constantly (c) insistently (d) consistently (e) perpetually

51. He is too...................to be deceived easily


(a) strong (b) modern (c) kind (d) honest (e) intelligent

52. The Manager gave her his ..... that the complaint would be investigated
(a) assurance (b) suggestion (c) avowal (d) support

53. I am feeling ...... better today.


(a) rather (b) too (c) fairly (d) very

Directions(Questions 54-59): In each of the following questions, find out which part of the sentence has an error.
If there is no mistake the answer is 'no error'.
54. I going there / will not solve / this complicated problem / No error
a b c d
55. You can get /all the information you want / in this book / No error
a b c d
56. The bus could not /ascend the steep hill /because it was in the wrong gears/No error
a b c d
57. No stronger / a figure than his / is prescribed in the history / No error
a b c d
58. Most people would have /attended the union meeting / if they had / longer notice of it / No error
a b c d e
59. And though one did not / quite believe his claim / one saw no harm / in granting him permission / No error
a b c

Pankaj Gandhi’s Academy


rd
3 Floor, Above Hotel Woodland, Near Sharda Center, Nal Stop, Pune. Contact No. : 8600972993, 9850424051.
English Sample Paper 3
Directions(Questions 1-5): Read the passage and answer the questions that follow on the basis of the
information provided in the passage.
It all started at the beginning of fifth grade. At first, Carmen wasn’t really sure what was happening. In class, she
had to squint to see the blackboard clearly. She had to do the same thing when she read street signs, or when she
watched a movie. As the fuzziness got worse, she became more and more worried. It was important for her to see
the notes and homework assignments the teacher put on the board.
It wasn’t long before Carmen found herself squinting all the time, but she didn’t want anyone to know that she was
having a problem seeing. In class, she asked for a desk that was closer to the blackboard. One day, her teacher
said, "Carmen, are you all right? I’ve noticed you squinting a lot. Are you having trouble seeing the board?"
Carmen shook her head. "I’m fine, Mrs. Cruz," she said, but she knew she couldn’t pretend much longer.
At home, she had to sit closer and closer to the television in order to see the picture. Her mother noticed her
squinting as she watched her favorite shows, and she began to get suspicious.
"Tomorrow I’m calling the eye doctor to set up an appointment for you," she said firmly. Carmen protested, but her
mother’s mind was made up.
Three days later, Carmen had new glasses and instructions from her doctor to wear them all the time. Carmen
frowned in the car the whole way home. "All of the kids at school will think I’m a nerd," she said. Her mother smiled
and shook her head. "You look just as beautiful with those glasses on as you do without them," she said. But
Carmen didn’t believe her. The next day, Carmen kept the glasses in her pocket as she walked into the
schoolyard. She avoided her friends and stood alone, feeling miserable. Suddenly, she heard her friend Theresa
shout. Carmen ran over to the other girls. "What’s wrong?" she asked. "My silver ring is gone!" Theresa cried. "My
sister sent it to me from California. It’s very special and I can’t lose it!" Carmen could tell that Theresa was
very upset. They all looked for the ring in the grassy area of the playground.
Carmen realized that she could search better if she could see better. She took the glasses out of her pocket and
put them on. The objects and people around her came into sharp focus. She caught her breath. Everything looked
so different! So clear! She looked down at the ground and a glimmer of silver caught her eye. It was the ring. "Here
it is," she shouted. "I’ve found it!" She handed it to Theresa, and Theresa slipped the ring back on her finger.
"Thanks Carmen," she said. "I never thought we'd find it." She paused. "Hey, I didn’t know you wore glasses. They
look great!" Carmen had forgotten that she was wearing the new glasses. "Thanks," she replied shyly. As they
walked back toward the school building, two more girls from her class complimented her glasses. Carmen smiled.
"Maybe wearing glasses won’t be so bad after all," she thought.
1. Before Carmen got glasses she
(a)Thought having glasses wouldn’t be so bad. (b) Wasn’t able to see the blackboard clearly.
(c) Found Theresa’s missing ring. (d) Sat far away from the television

2. Based on the end of the story, what do you think Carmen will do next?
(a) She won’t tell her friends that she needs to wear glasses.
(b) She will keep her glasses in her pocket where no one can see them.
(c) She will wear her glasses all the time.
(d) She will wear her glasses only when she is with her family.

3. Read this sentence from the story 'Her mother noticed her squinting as she watched her favorite shows, and she
began to get suspicious'. What is an antonym for the word suspicious?
(a) doubtful (b) guilty (c) innocent (d) trusting

4. Which statement best describes Carmen?


(a) She is willing to overcome her fears in order to help her friends.
(b) She doesn’t care how well she does in school.
(c) She cares more about herself than her friends.
(d) She doesn’t worry about what other people think of her.

5. Read this sentence from the story. 'In class, she had to squint to see the blackboard clearly'.
What does squint mean?
(a) to look with eyes partly closed (b) to move closer (c) to try hard (d) to concentrate

Directions(Questions 6-10): Read the passage and answer the questions that follow on the basis of the
information provided in the passage.
Primitive man was probably more concerned with fire as a source of warmth and as a means of cooking food than
as a source of light. Before he discovered less laborious ways of making fire, he had to preserve it, and whenever
he went on a journey he carried a firebrand with him. His discovery that the firebrand, from which the torch may

Pankaj Gandhi’s Academy


rd
3 Floor, Above Hotel Woodland, Near Sharda Center, Nal Stop, Pune. Contact No. : 8600972993, 9850424051.
very well have developed, could be used for illumination was probably incidental to the primary purpose of
preserving a flame.
Lamps, too, probably developed by accident. Early man may have had his first conception of a lamp while watching
a twig or fibre burning in the molten fat dropped from a roasting carcass. All he had to do was to fashion a vessel
to contain fat and float a lighted reed in it. Such lamps, which are made of hollowed stones or sea shells, have
persisted in identical from up to quite recent times.

6. Primitive man's most important use for five was


(a) to provide warmth (b) to cook food (c) to provide light (d) Both a and b

7. The firebrand was used to


(a) prevent accidents (b) provide light (c) scare animals (d) save labour

8. By 'primary' the author means


(a) primitive (b) fundamental (c) elemental (d) essential

9. Lamps probably developed through mere


(a) hazard (b) fate (c) chance (d) planning

10. Early lamps were made by


(a) using a reed as a wick in the fat (b) letting a reed soak the fat
(c) putting the fat in a shell and lighting it (d) floating a reed in the sea-shell

Directions(Questions 11-18): Pick out the most effective word from the given words to fill in the blank to make the
sentence meaningfully complete.
11. These essays are intellectually ............... and represent various levels of complexity
(a) revealing (b) modern (c) persistent (d) demanding (e) persistent

12. It was almost impossible for him to put out of his mind the ........ words which he heard from his clever father-in-
law
(a) inspiring (b) witty (c) sarcastic (d) soothing (e) exhortative

13. Integrity of character, honesty, dependability and discipline............... with a genuine interest in your work will go
a long way in the achievement of success in your professional life.
(a) coupled (b) adjoined (c) fixed (d) attached (e) joined

14. The soldiers were instructed to.........................restraint and handle the situation peacefully.
(a) exercise (b) control (c) prevent (d) enforce (e) remain

15. Ishwar Chandra Vidyasagar was one of the chief............... of women's rights
(a) promoters (b) facilitators (c) instigators (d) organizers (e) protagonists

16. Her parents will never give their .........to such an unsuitable match.
(a) willingness (b) agreement (c) consent (d) acquiescence

17. He is ......... dancer


(a) a skilled (b) an adept (c) an adapt (d) an adopt

18. The degrees were awarded in the annual.........................


(a) conference (b) convention (c) Convolution (d) convocation

Directions(Questions 19-26): In each question below is given a passage followed by several inferences. You
have to examine each inference separately in the context of the passage and decide upon its degree of truth or
falsity.
Mark your answer as:
(a) if the inference is ' definitely true' i.e., it directly follows from the facts given in the passage
(b) if the inference is ' probably true' though not definitely true in the light of the facts given
(c) if you think the data are in adequate i.e., from the facts given you cannot say whether the inference is likely to
be true or false
(d) if you think the inference is ' probably false' though not definitely false in the light of the facts given; and
(e) if you think inference is ' definitely false' i.e, it contradicts the given facts.

Pankaj Gandhi’s Academy


rd
3 Floor, Above Hotel Woodland, Near Sharda Center, Nal Stop, Pune. Contact No. : 8600972993, 9850424051.
Passage I
A recent survey shows that India has the lowest death rate for blood cancer. China, Thailand and Myanmar
(countries that have taste for spices) also have low rates. Higher rates are found in South America where spices
are not used. The typical American food remains chicken rolls, butter and beef.
19. Americans are unorthodox in their food habits. Ans: c
20. Americans dislike spices Ans: b
21. Spices prevent blood cancer Ans: b
22. Spices promote forms of cancer other than blood cancer Ans: d
23. Chicken rolls, butter and beef promote cancer Ans: d

Passage II
The water resources of our country are very much underutilized . The main reason of this underutilization is the
lack of capital and technology. A large portion of our water resources is wasted due to floods and unwise use of
water for irrigation as well as domestic purposes. we can make full use of our water resources by building dams on
rivers and by adopting policy of awareness among people not to waste water.
24. Occurrence of floods adds to the water resources. Ans: d
25. Some people do not use water resources in a judicious way. Ans: a
26. The country does not have enough funds to develop water resources Ans: b

Directions(Questions 27-32): In each of the following questions, find out which part of the sentence has an error.
If there is no mistake the answer is 'no error'.

27. My father is / in bad mood / today. / No error Ans: b


a b c d
28. The school is / with in hundred yards / from my house / no error Ans: b
a b c d
e
29. As soon as the teacher entered / everyone fell /in a silence / no error Ans: c
a
b c e
30. He took to / reading Times / for better knowledge / of the facts./ no error Ans: b
a b c d e
31. I will put on / a note in this regard / for your consideration / and necessary decision./ no error Ans: a
a b c
d e
32. He has been working on /the problem from a long time /but is still not / able to solve it./ no error Ans: a
a b c d e

Directions(Questions 33-40): In each question, a part of sentence is printed in italics. Below each sentence, some
phrases are given which can substitute the italicized part of the sentence. If the sentence is correct as it is, the
answer is 'No correction required'.
33. He did many mischiefs
(a) made many a mischiefs (b) made much mischief (c) Committed many mischiefs (d) No Correction
required

34. Rohit is as fast as or perhaps faster than Manish.


(a) Equally fast (b) almost as fast (c) as fast (d) No Correction required

35. All his family members are in Kanpur.


(a) All of his family members (b) All the family members if his
(c) All the members of his family (d) No Correction required

36. I often see him dancing the top


(a) rotating (b) encircling (c) dodging (d) No Correction required

37. What is the time in your watch?


(a) on (b) by (c) from (d) No Correction required

38. Columbus invented America


(a) searched (b) traced (c) discovered (d) No Correction required

Pankaj Gandhi’s Academy


rd
3 Floor, Above Hotel Woodland, Near Sharda Center, Nal Stop, Pune. Contact No. : 8600972993, 9850424051.
39. Wise men catch time by the forelock.
(a) Hold (b) seize (c) take (d) No Correction required

40. A bird in hand is worth two in bush


(a) two in the bush (b) two at a bush (c) two on bush (d) No Correction required

Directions for Questions 41-45: Read the passage and answer the questions that follow on the basis of the
information provided in the passage.
There are some men who seem to be always on the lookout for trouble and, to tell the truth, they are seldom
disappointed. Listening to such men one would think that this world is one of the stormiest and most disagreeable
places. Yet, after all it is not such a bad place and the difficulty is often in the man who is too thin- skinned. On the
other hand, the man who goes out expecting people to be like himself, kind and brotherly, will be surprised at the
kindness he meets even in the most unlike quarters. A smile is apt to be met met with a respective smile while the
sneer is just as apt to provoke a snarl. Men living in the same neighborhood may live vastly different lives. But it is
not
the neighborhood which is quarrelsome, but the man within us. And we have it in out power to change our
neighborhood into a pleasant one by simply changing our own ways.
41. The passage is about
(a) Our disagreeable and hostile world (b) A kindly and pleasant world
(c) Our different and unresponsive world (d) The world and what one makes of it.

42. "..............they are seldom disappointed". The statement denotes that such men
(a) Welcome difficulties as a morale booster (b) Do not have face any trouble
(c) Manage to keep unruffled in the face of discomforts (d) Generally do not fail to come across troubles

43. The author's own view of the world is that it is


(a) One of the loveliest and quietest places (b) An unpleasant and turbulent place
(c) One's own excessive sensitivity that makes it a bad place (d) A sordid place for those who suffer in
life

44. Which of the following is opposite in meaning to the expression 'thin-skinned' as sed in the passage?
(a) Insensitive (b) Intelligent (c) Awkward (d) Obstinate

45. "On the other hand............. unlikely quarter" The statement shows that people's reaction to our attitude is
(a) Generally indifferent (b) Surprisingly responsive (c) Often adverse (d) Mainly favorable

Directions(Questions 46-53): In each question below is given a passage followed by several inferences. You
have to examine each inference separately in the context of the passage and decide upon its degree of truth or
falsity.
Mark your answer as:
(a) if the inference is ' definitely true' i.e., it directly follows from the facts given in the passage
(b) if the inference is ' probably true' though not definitely true in the light of the facts given
(c) if you think the data are in adequate i.e., from the facts given you cannot say whether the inference is likely to
be true or false
(d) if you think the inference is ' probably false' though not definitely false in the light of the facts given; and
(e) if you think inference is ' definitely false' i.e, it contradicts the given facts.

Passage I
Urban services have not expanded fast enough to cope with urban expansion. Low investment allocation has
tended to be under spent. Both public (e. g. water and sewage) and private (e.g. low-income area housing)
infrastructure quality has declined. The impacts of the environment in which children live and the supporting
services available to them when they fall ill, seems clear. The decline in average food availability and the rise in
absolute poverty point in the same satisfactory direction.
46. There is nothing to boast about urban services Ans: A
47. The public transport system is in the hands of private sector. Ans :C
48. Birth rate is higher in urban areas compared to rural areas. Ans: C
49. Low-cost urban housing is one of the priorities Ans: B
50. The environment around plays an important role on the health status. Ans: A

Pankaj Gandhi’s Academy


rd
3 Floor, Above Hotel Woodland, Near Sharda Center, Nal Stop, Pune. Contact No. : 8600972993, 9850424051.
Passage II
Though the states cultivate only 3.2 lakh tones of mangoes, they are of premium quality and with mangoes
becoming second most consumed fruit in the world after grapes. The government has been trying exporting it
through sea route which is cheaper. An experiment which was done in this regard last year has proved successful.
51. Quality of mangoes is an important factor in exports.
52. The state also exports good quality grapes
53. The state also cultivates a large number of medium quality mangoes.

Directions 54-60: In each question, a part of sentence is printed in italics. Below each sentence, some phrases
are given which can substitute the italicized part of the sentence. If the sentence is correct as it is, the answer is
'No correction required'
54. The problems of translation are still remain.
(a) are remain (b) wills remain (c) will still remain. (d) No Correction required

55. It is ten years since I have begun living here


(a) begun (b) had begun (c) began (d) No Correction required

56. Education is a strong instrument for molding the character of the young.
(a) striking (b) powerful (c) potent (d) No Correction required

57. He gave the I.A.S. examination in all seriousness.


(a) appeared (b) took (c) undertook (d) No Correction required

58. He has cooked that meal so often he can do it with his eyes closed.
(a) mind blank (b) eyes covered (c) hands full (d) No Correction required

59. The young hikers went as far as they finally got lost in the valley.
(a) so far that (b) too far that (c) so far as that (d) No Correction required

60. He stopped to work an hour ago


(a) to working (b) to have worked (c) working (d) No Correction required

ENGLISH Sample Paper - 4


Directions (Q.1-8): Which of phrases given below each sentence should replace the phrase printed in bold type to
make the grammatically correct? If the sentence is correct as it is, mark 'E' as the answer.
1. You need not come unless you want to.
(a)You don't need to come unless you want to (b) You come only when you want to
(c)You come unless you don't want to (d) You needn't come until you don't want to
(e)No correction required

2. They were all shocked at his failure in the competition.


(a) were shocked at all (b) had all shocked at (c) had all shocked by
(d) had been all shocked on (e) No correction required

3. Despite of their differences on matters of principles, they all agree on the demand of hike is salary?
(a) Despite their (b) Despite of the (c) Despite for
their
(d) Despite off their (e) No correction required

4. He is a singer of repute, but his yesterday's performance was quite disappointing.


(a) performances of yesterday were (b) yesterday performance was (c) yesterday performance were
(d) performances about yesterday were (e) No correction required

5. The crime has growth rapidly in Russia since the disintegration of the communist system.
(a) rapid crime has grown (b) crime has grown rapidly (c) crimes grow rapidly
(d) crimes have been rapidly grown (e) No correction required

6. My hair stood off ends when I saw the horrible sight.


(a) stood at ends (b) stood on ends (c) stood to ends (d) stands on ends

Pankaj Gandhi’s Academy


rd
3 Floor, Above Hotel Woodland, Near Sharda Center, Nal Stop, Pune. Contact No. : 8600972993, 9850424051.
(d) No correction required

7. The crops are dying; it must not had rained.


(a) must had not (b)must not be (c) must not have
(d) must not have been (e) No correction required

8. The intruder stood quietly for few moments


(a) for few time (b) for the few moments (c) for moments
(d) for a few moments (e) No correction required

Directions (Q.9-16): In questions given below, a part of the sentence is italicized. Below are given alternatives to
the italicized part which may improve the sentence. Choose the correct alternative. In case no improvement is
needed, option 'D' is the answer.
9. If the room had been brighter, I would have been able to read for a while before bed time.
(a)If the room was brighter (b) If the room are brighter (c)Had the room been brighter (d)No
improvement

10. If you are not clear about the meaning of a word, it is wise to look to a dictionary.
(a) look for (b) look at (c) look up (d)No
improvement

11. There is no more room for you in this compartment.


(a) there is no more seat (b) there is no more space (c)there is no more accommodation (d) No improvement

12. More than one person was killed in accident.


(a)were killed (b) are killed (c) have been killed (d) No improvement

13. Every time I go in a lift to my sixth floor apartment, I remember the calm and serenity of my ancestral home in
the village.
(a) move in a lift (b) ascend in a lift (c) take a lift (d) No improvement

14. My friend was in hospital for a week after an accident.


(a) through (b) following (c) for (d)No
improvement

15. If you are living near a market place you should be ready to bear the disturbances caused by traffic.
(a)to bear upon (b) to bear with (c) to bear away (d) No improvement

16. In India today many of our intellectuals still talk in terms of the French Revolution and the Rights of Man, not
appreciating that much has happened since then.
(a)much has been happening (b) much had happened (c) much might happen (d)No
improvement

Directions (Q.17-24): Pick out the most effective word(s) from the given words to fill in the blank to make the
sentence meaningfully complete.
17. Catching the earlier train will give us the ...... to do some shopping.
(a) chance (b) luck (c) possibility (d) occasion

18. Many of the advances of civilization have been conceived by young people just on the ...... of adulthood
(a) boundary (b) threshold (c) peak (d) horizon

19. If I take a state roadways bus, I'll get late...............?


(a)isn't it (b) won't I (c) will I (d) is it

20. The paths of glory lead ...... to the grave.


(a)straight (b) but (c) in (d) directly

21. Wheat ...... carbohydrates, vitamins, proteins, and dietary fibre in our daily diet.
(a) has (b) gives (c) yields (d) provides

22. ...... all intents and purposes, the manager is the master of the firm.

Pankaj Gandhi’s Academy


rd
3 Floor, Above Hotel Woodland, Near Sharda Center, Nal Stop, Pune. Contact No. : 8600972993, 9850424051.
(a) in (b) upon (c) with (d) to

23. He ...... in wearing the old fashioned coat in spite of his wife's disapproval.
(a) insists (b) persists (c) desists (d) resists

24. We shall not to be able to use your ability in court unless we can find someone to ...... to statements.
(a) Corroborate (b) avouch (c) verify (d) approve

Directions (Q. 25-30): Each of the following questions contains a small paragraph followed by a question on it.
Read each paragraph carefully and answer the question given below it.

25. Due to enormous profits involved in smuggling, hundreds of persons have been attracted towards this anti-
national activity. Some of them became millionaires overnight. India has a vast coastline both on the Eastern and
Western Coast. It has been a heaven for smugglers who have been carrying on their activities with great impunity.
There is no doubt, that from time to time certain seizures were made by the enforcement authorities, during raids
and ambush but even allowing these losses the smugglers made huge profits. The passage best supports the
statement that
(a) Smuggling hampers the economic development of a nation.
(b) Smuggling ought to be curbed.
(c) Authorities are taking strict measures to curb smuggling.
(d) Smuggling is fast increasing in our country owing to the quick profit it entails.

26. Industrial exhibitions play a major role in a country's economy. Such exhibitions, now regularly held in Delhi,
enable us to measure the extent of our own less advanced industrial progress and the mighty industrial power and
progress of countries like the U.K., U.S.A. and Russia whose pavilions are the centers of the greatest attention and
attractions. The passage best supports the statement that industrial exhibitions -
(a) Greatly tax the poor economies.
(b) Are more useful for the developed countries like U.S.A. whose products stand out superior to those of the
developing countries.
(c) Are not of much use to the countries that are industrially backward.
(d) Boost up production qualitatively and quantitatively by analytical comparison of a country's products
with those of the developed countries.

27. It is up to our government and planners to devise ways and means for the mobilization of about ten crore
workers whose families total up about forty crore men, women and children. Our agriculture is over-manned. A
lesser number of agriculturists would mean more purchasing or spending power to every agriculturist. This will
result in the shortage of man-power for many commodities to be produced for which there will be a new demand
from a prosperous agrarian class. This shortage will be removed by surplus man-power released from agriculture
as suggested above. The passage best supports the statement that:
(a) Employment in production is more fruitful than employment in agriculture.
(b) Indian economy is in a poor shape basically due to improper mobilization of man-power.
(c) A shift of labour from agricultural sector to the industrial sector would uplift the living standard.
(d) The industrial sector is labour-deficient while the agricultural sector is over-manned in our country.

28. The only true education comes through the stimulation of the child's powers by the demands of the social
situations in which he finds himself. Through these demands he is stimulated to act as a member of a unity, to
emerge from his original narrowness of action and feeling, and to conceive himself from the standpoint of the
welfare of the group to which he belongs. The passage best supports the statement that real education -
(a) Will take place if the children imbibe action and feeling.
(b) Will take place if the children are physically strong.
(c) Is not provided in our schools today.
(d) comes through the interaction with social situations

29. The virtue of art does not allow the work to be interfered with or immediately ruled by anything other than itself.
It insists that it alone shall touch the work in order to bring it into being. Art requires that nothing shall attain the
work except through art itself. This passage best supports the statement that:
(a) Art is governed by external rules and conditions.
(b) Art is for the sake of art and life.
(c) Art is for the sake of art alone.
(d) Artist realizes his dreams through his artistic creation.

Pankaj Gandhi’s Academy


rd
3 Floor, Above Hotel Woodland, Near Sharda Center, Nal Stop, Pune. Contact No. : 8600972993, 9850424051.
30. The attainment of individual and organizational goals is mutually interdependent and linked by a common
denominator - employee work motivation. Organizational members are motivated to satisfy their personal goals,
and they contribute their efforts to the attainment of organizational objectives as means of achieving these personal
goals. The passage best supports the statement that motivation -
(a) Encourages an individual to give priority to personal goals over organizational goals.
(b) Is crucial for the survival of an individual and organization.
(c)Is the product of an individual's physical and mental energy?
(d) Is the external force which induces an individual to contribute his efforts?

Directions (Q.31-40): The following section consists of two passages followed by set of questions. Read the
passage thoroughly and answer the set of questions given below the passage.
(31 to 35) In the world today we make health and end in itself. We have forgotten that health is really means to
enable a person to do his work and do it well. A lot of modern medicine and this includes many patients as well as
many physicians pays very little attention to health but very much attention to those who imagine that they are ill.
Our great concern with health is shown by the medical columns in newspapers. The health articles in popular
magazines and the popularity of television programmers and all those books on medicine. We talk about health all
the time. Yet for the most part the only result is more people with imaginary illness. The healthy man should not be
wasting time talking about health: he should be using health for work. The work does the work that good health
possible.

31. Modern medicine is primarily concerned with


(a) promotion of good health (b) people suffering from imaginary illness
(c) people suffering from real illness (d) increased efficiency in work

32. The passage suggests that


(a) health is an end in itself (b) health is blessing
(c) health is only means to an end (d) we should not talk about health

33. Talking about the health all time makes people


(a) always suffer from imaginary illness (b) sometimes suffer from imaginary illness
(c) rarely suffer from imaginary illness (d) often suffer from imaginary illness

34. The passage tells us


(a) how medicine should be manufactured (b) what healthy man should or should not do
(c) what television programmers should be about (d) how best to imagine illness

35. A healthy man should be concerned with


(a) his work which good health makes possible (b) looking after his health
(c) his health which makes work possible (d) talking about health

(Q. no. 36 - 40): Organizations are institutions in which members complete for status and power. They compete for
resource of the organization, for example finance to expand their own departments, for career advancement and
for power to control the activities of others. In pursuit of these aims, grouped are formed and sectional interests
emerge. As a result, policy decisions may serve the ends of political and career systems rather than those of the
concern. In this way, the goals of the organization may be displaced in favor of sectional interests and individual
ambition. These preoccupations sometimes prevent the emergence of organic systems. Many of the electronic
firms in the study had recently created research and development departments employing highly qualified and well
paid scientists and technicians. Their high pay and expert knowledge were sometimes seen as a threat to the
established order of rank, power and privilege. Many senior managers had little knowledge of technicality and
possibilities of new developments and electronics. Some felt that close cooperation with the experts in an organic
system would reveal their ignorance and show their experience was now redundant.
36. The theme of the passage is
(a) Groupism in organizations (b) Individual ambitions in organizations
(c)Frustration of senior managers (d) Emergence of sectional interests in organizations
Ans: D

37. "Organic system" as related to the organization implies its


(a) growth with the help of expert knowledge (b) growth with input from science and technology
(c) steady all around development (d) natural and unimpeded growth

38. Policy decision in organization would involve

Pankaj Gandhi’s Academy


rd
3 Floor, Above Hotel Woodland, Near Sharda Center, Nal Stop, Pune. Contact No. : 8600972993, 9850424051.
(a) Cooperation at all levels in the organization (b) modernization of the organization
(c) attracting highly qualified personnel (d) keeping in view the larger objectives of the
organizations

39. The author makes out a case for


(a)organic system (b) Research and Development in organizations
(c) an understanding between senior and middle level executives (d) a refresher course for senior
managers

40. The author tends to the senior managers as


(a) ignorant and incompetent (b) a little out of step with their work environment
(c) jealous of their younger colleagues (d)robbed of their rank, power and privilege

I English Sample Paper - 5


Directions for (Q.1 – 10): Pick out the most effective pair of words from the given pair of words to make the
sentence/sentences meaningfully complete.
1. The teacher must …………….the unique style of a learner in order to …………..it to the desired knowledge.
(a) advocate, direct (b) perpetuate, develop (c) appreciate, focus (d)
discover………harness

2. Not all countries benefit ……..from liberalization. The benefits tend to ………first to the advantaged and to those
with right education to be able to benefit from the opportunities presented.
(a) equally, generate (b) richly, downgrade (c) suitably, ascribe (d) uniformly, percolate

3. He has ………..sense of words. Therefore, the sentence he constructs are always…………with rich meaning.
(a) profound, pregnant (b) distinguishes, loaded (c) terrific, tempted (d) meaningful………..full

4. He was an ……….musician, had been awarded the George Medal during the second world war and ……….with
the title of Rai Bahadur.
(a) outstanding, popularized (b) underestimated, declared
(c) accomplished, honoured (d)obdurate,proclaimed

5. Whether it be shallow or not, commitment is the ………, the bedrock of any ……….loving relationship.
(a) expression, perfunctory (b) foundation, genuinely (c) manifestation, deep (d) key, alarmingly

6. Many people take spirituality very seriously and ……….about those who don’t, worrying about them and
………..them to believe.
(a) think, criticizing (b)pride, appraising (c)rationalize, enabling (d) wonder, prodding

7. Unless new reserves are found soon, the world’s supply of coal is being ………..in such a way that with demand
continuing to grow at present rates, reserves will be……………by the year 2050.
(a) consumed, completed (b) depleted, exhausted (c) reduced, .argument (d) burnt, destroyed

8. If you are ……….you tend to respond to stressful situations in a calm, secure, steady and ……….way.
(a) resilient, rational (b) obdurate, manageable (c) propitious, stable (d) delectable, flexible

9. Management can be defined as the process of ………….organizational goals by working with and through
human and non-human resources to ………improve value added to the world.
(a) getting, deliberately (b) managing, purposefully (c) targeting, critically (d) reaching,
continuously

10. If you are an introvert, you……….to prefer working alone and, if possible, will ………towards projects where
you can work by yourself or with as few people as possible.
(a) like, depart (b) advocate, move (c) tend, gravitate (d) express, attract

Directions for (Q. 11 – 15): In each sentence below, a word/group of words has been printed in bold. From the
four answer choices given below each sentences, pick out the one which can substitute the bold word/group of
words correctly without changing the meaning of the sentence.
11. America’s leadership will look to identify and hunt down perpetrators of this heinous crime.

Pankaj Gandhi’s Academy


rd
3 Floor, Above Hotel Woodland, Near Sharda Center, Nal Stop, Pune. Contact No. : 8600972993, 9850424051.
(a) hunt in (b) hunting down (c) hunt them down (d) No correction
required

12. The intelligence establishment in the US was convinced that if any threats were there to the US, they were
external.
(a) if there were any threats (b) if any threat (c) threats if any were there (d) no correction required

13. The illicit cultivation of opium is taking place on various part of India and Nepal.
(a) is placed on various part (b) is taking place there into various part
(c) is taking place in various parts (d) no correction is required

14. The scheme underline the needs of bridging the digital divide, which is threatening to assume dangerous
proportions.
(a) is underlining the need of (b) underlines the need for (c) underlined the need to (d) no correction
required

15. The government has decided of proceeding with its disinvestment policy.
(a) to proceed with its (b) proceeding on its (c) on to proceed to (d) no correction
required

Direction for (Q. 16 – 20) Spot the error:


16.India today stands at a crucial (1)/ crossroads with its history as (2)/ an independent nation (3)/ No error(4)
17. Honesty and integrity are (1)/ the qualities which cannot be (2)/ done away with(3)/ and hence assume a lot of
importance(4)/ No error (5)
18. Inspite of the doctor's stern warning,(1) / he continued taking(2) / sugar in his tea(3) / No error(4)
19. myself and Gopalan (1)/ will take care of (2)/ the function on Sunday(3) / No error
20. Being a very(1) / hot day I(2) / remained indoors(3) / No error(4)

Direction for (Q.21 - 25) : Read the passage and answer the questions that follow on the basis of the information
provided in the passage.
Justin was always prepared. His motto was "Never throw anything out, you never know when it might come in
handy." His bedroom was so full of flat bicycle tires, bent tennis rackets, deflated basketballs, and games with
missing pieces that you could barely get in the door. His parents pleaded with him to clean out his room.
"What use is a fish tank with a hole in the bottom?" his father asked. But Justin simply smiled and repeated his
motto, "Never throw anything out, you never know when it might come in handy." When Justin was away from
home, he always carried his blue backpack. He liked to think of it as a smaller version of his bedroom—a place to
store the many objects that he collected. It was so worn and stretched that it hardly resembled a backpack
anymore. It was full of the kind of things that seemed unimportant, but when used with a little imagination, might
come in handy. Justin had earned a reputation for figuring things out and getting people out of otherwise hopeless
situations. Many of his classmates and neighbors sought him out when they needed help with a problem. On the
first day of school, his friend Kenny, came looking for Justin. "Do you think you have something in your bag that
could help me remember my locker combination?" he asked. "I lost the scrap of paper it was written on. I have
science class in two minutes and if I’m late on the first day it’ll make me look bad for the rest of the year." Kenny
looked genuinely worried. "Relax," Justin said, taking his backpack off and unzipping the top. "Remember how you
borrowed my notebook in homeroom to write the combination down? Well, I know how we can recover what you
wrote." He took the notebook and a soft lead pencil out of his bag. The page that Kenny had written on had left faint
indentations on another page in the notebook. Justin held the pencil on its side and rubbed it lightly over the
indentations. Slowly but surely the numbers of the locker combination appeared in white, set off by the gray pencil
rubbings. "That’s amazing!" Kenny said. "I owe you one." And he dashed off to open his locker. During science
class, Mr. Tran was lecturing on the structure of the solar system using a model. He made a sudden gesture and
the model fell apart. Planets and rings and connector rods went everywhere, rolling and clattering and disappearing
under desks. The students scrambled around on the floor for ten minutes and were finally able to recover every
piece except one—a connector rod that was lodged in a crack between two lab stations. "If we had a magnet," said
Mr. Tran, "we could easily coax it out that way. But I loaned all of the magnet kits to the elementary school
yesterday." Justin was already searching through his backpack. "I have some materials that will work just as well, I
think," he told Mr. Tran. He pulled out a battery, an iron nail, and some electrical wire and tape, while Mr. Tran and
the other students looked on in amazement. "Why do you have all of that stuff?" Louise Baxter asked. Justin just
smiled and repeated his motto. "Never throw anything out, you never know when it might come in handy." By
wrapping the wire around the nail and taping each end to a battery terminal, he was able to make a magnet strong
enough to lift the rod out of the crack. "Bravo!" said Mr. Tran. "No problem," said Justin.

Pankaj Gandhi’s Academy


rd
3 Floor, Above Hotel Woodland, Near Sharda Center, Nal Stop, Pune. Contact No. : 8600972993, 9850424051.
After school, Justin rode the bus to the mall where he worked at a music store. His boss, Gail, was taking inventory
of all of the CDs and tapes in the classical music section. As he helped a customer at the register, Justin heard her
exclaim, "Oh, no! I forgot my glasses! There’s no way I can read this list without them." Justin sighed, picked up his
backpack, and walked over to Gail.
"I think I can help you out," he said, unzipping the bag. While Gail watched in surprise, he pulled out a jar of
petroleum jelly, a washer, a glass slide, and a small bottle of water. He put the jelly on the bottom of the washer,
placed it securely, jelly-side down, on the glass slide, and then put a drop of water in the center of the washer. He
put the contraption on top of the inventory list and said to his boss, "See what happens when you look through the
water droplet." Gail looked and her eyes widened with delight.
"Wow!" she cried. "It enlarges the print that I’m looking at, just like a magnifying glass!" She patted Justin on the
back. "I’m all set now," she said. "Thanks."
Justin smiled. "No problem," he said, returning to the register. It was just another day in the life of the boy whose
motto was "Never throw anything out, you never know when it might come in handy."
21. Why is Justin’s room such a mess?
(a) He always forgets to clean. (b) He never throws anything away.
(c) He has no time to clean. (d) He shares a room with his brother.

22. Read this sentence from the story. In what way is Justin’s backpack a smaller version of his bedroom?
(a) He uses it as a place to store objects. (b) He uses it to carry his books and sports equipment.
(c) His parents tell him to clean it all the time. (d) He’s had for as long as he can remember.

23. Read this sentence from the story. His parents pleaded with him to clean out his room. Which word is a
synonym for pleaded?
(a) ignored (b) asked (c) pushed (d) begged

24. How does Justin help his friends?


(a) He offers them advice. (b) He loans them his backpack
(c) He listens to their problems. (d) He uses the objects in his backpack.

25. How do most of the characters in the story feel toward Justin?
(a) annoyed (b) grateful (c) disinterested (d) angry

Directions for (Q. 26-30): Read the passage and answer the questions that follow on the basis of the information
provided in the passage
It is difficult to reconcile the ideas of different schools of thought on the question of education. Some people
maintain that pupils at school should concentrate narrow range of subjects which will benefit them directly in their
subsequent careers. Others contend that they should study a wide range of subjects so that they have not only the
specialized knowledge necessary for their chosen careers but also sound general knowledge about the world they
will have to work and live in. Supporters of the first theory state that the greatest contributions to civilization are
made by those who are most expert in their trade profession. Those on the other side say that, unless they have a
broad general education, the experts will be too narrow in their outlook to have sympathy with their follows or a
proper sense of responsibility towards humanity as a whole.
26. 'Schools of thought' can be explained as
(a) groups of people whose job is to think
(b) groups of people who are schooled to think
(c) groups of people who study in a particular school thoughtfully
(d) groups of people having the same ideas but with different perception on a particular subject.

27. Broad general knowledge is necessary because


(a) specialization is incomplete without it. (b) without it no one would get a job
(c) it teaches us about different things (d) it broadens one's outlook

28. The idea of the first school of thought in the passage is that
(a) students should concentrate on studies
(b) students should not undertake any specialized work
(c) students should study all the subjects they want to
(d) students should study a few subjects that will help them in their profession

29. Supporters of the first theory say that


(a) experts have contributed most to progress in the modern world
(b) people with general knowledge have contributed to civilization

Pankaj Gandhi’s Academy


rd
3 Floor, Above Hotel Woodland, Near Sharda Center, Nal Stop, Pune. Contact No. : 8600972993, 9850424051.
(c) experts have done nothing to help mankind
(d) people with general knowledge are more useful than experts

30. According to the second school of thought, education will not be very effective if pupils
(a) have inadequate knowledge of their own work
(b) do not have a wide general education
(c) ignore the study of fine arts
(d) have nothing but general knowledge

Direction for (Q.31 to 35 ) Choose the most logical order of sentences from among the given choices to construct
a coherent paragraph.
31. 1.1971 war changed the political geography of the subcontinent
2. Despite the significance of the event. There has been no serious book about the conflict
3. Surrender at Dacca aims to fill this gap
4. It also profoundly altered the geo-strategic situation in South-East Asia
(a)1324 (b) 3142 (c) 2143 (d)1423

32. 1. Nonetheless, Tocqueville was only one of the first of a long line of thinkers to worry whether such rough
equality could survive in the face of a growing factory system that threatened to create divisions between industrial
workers and a new business elite.
2. "The government of democracy brings the nation of political rights to the level of the humblest citizens. He wrote
," Just as the dissemination of wealth brings the notion of property within the reach of all the members of the
community".
3. Tocqueville was far too shrewd an observer to be uncritical about the US, but his verdict was fundamentally
positive.
4. No visitor to the US left a more enduring record of his travels and observations than the French writer and
political theorist Alexis de Tocqueville, whose ‘Democracy in America’, first published in 1835, remains one of the
most trenchant and insightful analyses of American social and political practices.
(a) 4132 (b) 2134 (c) 4321 (d)4213

33. 1.The potential exchanges between the officials of IBBF and the Maharashtra Body-Building Association has all
the trappings of a drama we are accustomed to.
2. In the case of sports persons, there is room for some sympathy, but the apathy of the administrators, which has
even led to sanctions from international bodies, is unpardonable.
3. A case in the point is the hefty penalty of US $10,000 slapped on the Indian Body-Building Federation for not
fulfilling its commitment for holding the Asian Championships in Mumbai in October.
4. It is a matter of deep regret and concern that the sports administrators often cause more harm to the image of
the country than sportsmen and sportswomen do through their dismal performances.
(a) 3124 (b) 4231 (c) 4123 (d)3421

34. 1. Over the years, I have had the opportunities to observe and understand the thought processes behind the
ads that have been flooding both the print and the TV media.
2). Although there is a huge shift in the quality of ads that we come across on a daily basis-- thanks essentially to
improvement in technology--I somehow can't help but feel that the quality of communication of the message has
become diluted.
3. Proportionally, the number of ads that lack in quality, have gone up exponentially as well!!
4. There is an increasing attempt by most companies to be seen as cool and funky.
5. Another reason could be the burgeoning number of companies, which means an exponential increase in the
number of ads that are being made.
(a) 43125 (b) 12453 (c) 43512 (d)21435

35. 1. His political career came to an abrupt end with China's military operation.
2. He attracted as as repelled.
3. He was responsible for the debacle.
4. A man of paradoxes, Menon remained an enigma.
(a) 4312 (b) 1342 (c) 4213 (d) 4123

Direction (Q. 36-40): In each question below is given a passage followed by several inference. You have to
examine each inference separately in the context of the passage and decide upon its degree of truth or falsity,
mark your answer as :
(a) if the inference is ' definitely true' i.e. , it directly follows from the facts given in the passage

Pankaj Gandhi’s Academy


rd
3 Floor, Above Hotel Woodland, Near Sharda Center, Nal Stop, Pune. Contact No. : 8600972993, 9850424051.
(b) if the inference is ' probably true' though not definitely true in the light of the facts given
(c) if you think the data are in adequate i.e., from the facts given you cannot say whether the inference is likely to
be true or false
(d) if you think the inference is ' probably false' though not definitely false in the light of the facts given; and
(e) if you think inference is ' definitely false' i,e , it contradicts the given facts.
Passage: Primary education in Bihar is in a poor shape. Pupils in over 50% of the schools read in the open
throughout the year. Over six million children born to lower income group parents remain unlettered . Not even 1%
of the 63,000 primary schools have facilities of furniture, toilet, drinking water and games. 3113 new teachers have
been appointed in the current financial year out of which 2747 are women. Now each of the 13270 primary school
have at least two teachers.

36. 630 primary schools in Bihar have all the facilities like like furniture, toilet, drinking water , games ANS: (e)

37. In Bihar, 90% of the primary teachers are women ANS: (a)

38. In Bihar, 50% of the children are illiterate ANS: (c)

39. A large number of primary schools in Bihar are one teacher schools ANS: (e)

40. Classroom for most of the primary schools in Bihar inadequate. ANS: (e)

I English Sample Paper – 6


Directions for (Q. 1-5): Pick out the most effective word from the given words to fill in the blank to make the
sentence meaningfully complete.
1. Everyday, in the school, one period is..................to games, sports and physical exercise.
(a) entrusted (b) dedicated (c) conceded (d) devoted (e) conferred

2. What frustrates the opposition's effort of unity is the absence of a ...................long term strategy
(a) credible (b) contestable (c) creditable (d) compatible (e) conceivable

3. Some people...................themselves into believing that they are indispensable to the organization they work for
(a) keep (b) fool (c) force (d) denigrate (e) delude

4. The Government sought to set at.....................speculation that there may be a shift in nuclear policy
(a) pace (b) rest (c) peace (d) fire (e) right

5. How do you expect that country to progress when her government is corrupt...............and still largely feudal?
(a) devalued (b) dwindling (c) despotic (d) demeaning (e) demobilised

Direction for (Q. 6-8): In each question below is given a passage followed by several inference. You have to
examine each inference separately in the context of the passage and decide upon its degree of truth or falsity,
mark your answer as :
(a) if the inference is ' definitely true' i.e. , it directly follows from the facts given in the passage
(b) if the inference is ' probably true' though not definitely true in the light of the facts given
(c) if you think the data are in adequate i.e., from the facts given you cannot say whether the inference is likely to
be true or false
(d) if you think the inference is ' probably false' though not definitely false in the light of the facts given; and
(e) if you think inference is ' definitely false' i,e , it contradicts the given facts.
Passage: A tiger when killing its natural prey, which it does not either by stalking or lying in wait for it, depends for
the success of its attack on its speed and, to a lesser extent, on the condition of its teeth and claws. When ,
therefore, a tiger is suffering from one or more painful wounds or when its teeth are missing or defective and its
claws worn down, and it is unable to catch animals it has been accustomed to eating , it is driven by the necessity
to killing human beings
6. Human beings are natural prey of tigers ANS: (e)
7. Old age propels tigers to take to man eating ANS: (b)
8. tiger kills man only when it has been incapacitated through wounds. ANS: (e)

Pankaj Gandhi’s Academy


rd
3 Floor, Above Hotel Woodland, Near Sharda Center, Nal Stop, Pune. Contact No. : 8600972993, 9850424051.
Direction for (Q. 9-11): In each of the following questions, find out which part of the sentence has an error. if there
is no mistake the answer is 'no error'
9. I saw (1)/ a few children (2) /playing on sand(3) / No error(4)

10. Interviews for (1) / the post of lectures(2) / will begin from Monday(3) /No error(4)

11. As he is rich (1)/ so he spends(2) / lavishly (3)/ No error(4)

Directions for (Q. 12-20): In each question, a part of sentence is printed in italics. Below each sentence, some
phrases are given which can substitute the italicized part of the sentence. If the sentence is correct as it is, the
answer is 'No correction required'
12. He was born to rich parents
(a) of (b) with (c) by (d) No Correction required

13. Each boy and each girl was busy in her work
(a) were busy in their work (b) was busy in his work
(c) was busy in one's work (d) No Correction required .

14. Place a ladder on the wall


(a) along (b) with (c) against (d) No Correction required

15. 'Hard Days' was his last novel


(a) latter (b) recent (c) latest (d) No Correction required

16. It is most unique piece of art.


(a) a unique (b) the most unique (c) an unique (d) No Correction required

17. A boy has been sunk in this river


(a) submerged (b) immersed (c) drowned (d) No Correction required

18. Two hours have elapsed since he had fallen asleep.


(a) fell (b) has fallen (c) was fallen (d) No Correction required

19. With a thundering roar the huge rocket soared up from the launching pad.
(a) took off (b) went upwards (c) flew up (d) No Correction required

20. In the run-up to the conference, various political and social issues were brought up by interested activists and
human rights groups.
(a) In running up (b) On the run-up (c) Over the running up (d) no correction required

Direction for (Q. 21 to 25) Rearrange the following sentences in appropriate order.
21.
1.Thus begins the search for relief: painkillers, ice, yoga, herbs, even surgery
2.Most computer users develop disorders because they ignore warnings like tingling fingers, a numb hand or a
sore shoulder
3. They keep pointing and dragging until tendons chafe and scar tissue forms, along with bad habits that are almost
impossible to change
4. But cures are elusive, because repetitive stree injuries present a bag of ills that often defy easy diagnosis.
(a) 2413 (b) 2143 (c) 2314 (d) 1234

22. 1. Let us take a look at the manner in which the traditional bank adds value to the customer.
2. The ability to retain deposits, in itself, is not enough to ensure long-term survival and growth.
3. The ability to deploy invested funds into productive economic activity at a higher rate of return, hence
contributing to the prosperity of both the economy and the institution, is the other loop in the banking cycle.
4. Further, as only a small portion of the actual deposit base is retained with the bank in a liquid form, the very
survival of the bank lies in building enough trust with its clientele so as to prevent the occurrence of a sizeable
chunk of simultaneous customer withdrawal (a run on the bank).
5. The bank's basic job is risk absorption- it takes money, which has a lot of attached risk, and provides the
customer an assured rate of return.
(a) 31425 (b) 51234 (c) 15423 (d) 24153

Pankaj Gandhi’s Academy


rd
3 Floor, Above Hotel Woodland, Near Sharda Center, Nal Stop, Pune. Contact No. : 8600972993, 9850424051.
23.1. What came out was very large garland made out of currency notes.
2. The unsuspecting governor opened the box in full view of the gathering
3. When the RBI governor came to inaugurate the new printing press, the local unit of the BJP handed him a gift
wrapped box
4. There was a twist – the notes were all as tattered as notes could get
(a) 4132 (b) 3124 (c) 3214 (d) 4312

24. 1. Otherwise the Congress would not have opposed PSU disinvestment today.
2. It is clear that there is not consensus on economic reform.
3. Nor would allies of ruling NDA opposes privatisation.
4. All this would stop India from becoming the next superpower.
(a) 1234 (b) 1243 (c) 2134 (d) 2314

25. 1. I suggested that Ford should buy up a company called NCP, which owned most of the car parks in the city
centres throughout the UK.
2. We were discussing competing in the European market.
3. If NCP became a Ford company, a notice could be placed at the entrance to all city centres car parks indicating
that only Ford cars could use them
4. At one time I was giving a seminar for the British marketing department of Ford, the biggest Ford operation
outside of Detroit.
(a) 2134 (b) 4213 (c) 2431 (d) 2314

Direction for (Q. 26 – 30) Read the following passage and answers the questions based on it.
Persuasion is the art of convincing someone to agree with your point of view. According to the ancient
Greek philosopher Aristotle, there are three basic tools of persuasion: ethos, pathos, and logos.
Ethos is a speaker’s way of convincing the audience that she is a credible source. An audience will consider a
speaker credible if she seems trustworthy, reliable, and sincere. This can be done in many ways. For example, a
speaker can develop ethos by explaining how much experience or education she has in the field. After all, you
would be more likely to listen to advice about how to take care of your teeth from a dentist than a firefighter. A
speaker can also create ethos by convincing the audience that she is a good person who has their best interests at
heart. If an audience cannot trust you, you will not be able to persuade them.
Pathos is a speaker’s way of connecting with an audience’s emotions. For example, a speaker who is trying to
convince an audience to vote for him might say that he alone can save the country from a terrible war. These
words are intended to fill the audience with fear, thus making them want to vote for him. Similarly, a charity
organization that helps animals might show audience pictures of injured dogs and cats. These images are intended
to fill the viewers with pity. If the audience feels bad for the animals, they will be more likely to donate money.
Logos is the use of facts, information, statistics, or other evidence to make your argument more convincing. An
audience will be more likely to believe you if you have data to back up your claims. For example, a commercial for
soap might tell you that laboratory tests have shown that their soap kills all 7,000,000 of the bacteria living on your
hands right now. This piece of information might make you more likely to buy their brand of soap. Presenting this
evidence is much more convincing than simply saying “our soap is the best!” Use of logos can also increase a
speaker’s ethos; the more facts a speaker includes in his argument, the more likely you are to think that he is
educated and trustworthy.
Although ethos, pathos, and logos all have their strengths, they are often most effective when they are used
together. Indeed, most speakers use a combination of ethos, pathos, and logos to persuade their audiences. The
next time you listen to a speech, watch a commercial, or listen to a friend try to convince you to lend him some
money, be on the lookout for these ancient Greek tools of persuasion.
26. As used in paragraph 2, what is the best antonym for credible?
(a)unintelligent (b) boring (c) dishonest (d) amazing

27. Amy is trying to convince her mother to buy her a pair of $200 shoes. She says: “Mom, the shoes I have are
really old and ugly. If I don’t get these new shoes, everyone at school is going to laugh at me. I will be so
embarrassed that I will want to die.” What form of persuasion is Amy using here?
(a) pathos (b) ethos (c)logos (c) a combination of ethos, pathos, and
logo

28. According to the passage, logos can build ethos because


(a) an audience is more easily convinced by facts and information than simple appeals to emotions like pity or fear
(b) an audience is more likely to trust a speaker who uses evidence to support his argument
(c) a speaker who overuses pathos might make an audience too emotional; audiences who are too frightened or
too sad are unlikely to be persuaded

Pankaj Gandhi’s Academy


rd
3 Floor, Above Hotel Woodland, Near Sharda Center, Nal Stop, Pune. Contact No. : 8600972993, 9850424051.
(d) a speaker can use misleading or false information to make his argument seem more convincing

29. Gareth is running for mayor. He tells his audience: “Under our current mayor, there have been 15,000 new
cases of unemployment. If he stays in office, who knows how many more people will lose their jobs? The number
could go up even higher. When I was the CEO of Magnatech, I helped to create over 1,000 new jobs. I can do the
same thing for this city if you vote for me.” Which form of persuasion is Gareth using here?
I. pathos II. logos III. ethos
(a) I only (b) I and II only (c) II and III only (d) I, II, and III

30. According to the passage, the most effective tool of persuasion is


(a) ethos, because you cannot persuade an audience that does not trust you
(b) logos, because it can also be used to build ethos
(c) a combination of ethos, pathos, and logos
(d) pathos, because human beings are most easily persuaded by emotion.

Passage 2:
It is breakfast time. You have been looking forward to eating a nice ripe banana ever since you woke up. Just when
you reach for the delicious piece of fruit on your counter, you see something that makes you much less hungry: a
swarm of fruit flies!

Fruit flies are tiny insects that are attracted to ripe or rotting fruits and vegetables. The flies not only eat the fruit,
they also lay their eggs there. A single fruit fly can lay up to 500 eggs on the surface of a piece of fruit. Within eight
days, the fruit flies that hatch from these eggs are full adults that can then lay their own eggs. As you can see,
what might start out as a small fruit fly problem can become very large very quickly.

Although there is a chance fruit flies can carry germs on to your food, this is not very likely. Fruit flies are annoying,
but they probably will not hurt you. Because they are such a nuisance, however, most people want to get rid of
these pesky bugs as quickly as possible. Some people use pesticide sprays on the fruit flies. Although this will kill
the flies, it will also spread harmful poison all over your kitchen. Luckily, there is also a completely safe way for you
to get rid of fruit flies in your house.
The first step is for you to remove all fruits or vegetables from your counter. Store these items in the refrigerator or
in sealed containers. Clean up any spilled juice or bits of food that might be on the floor. Take out the trash and
empty the recycling bin. Wash any dirty dishes that are in your sink. Doing all of these things will stop new fruit flies
from finding food or places to lay their eggs.

Next, make a trap to catch all of the remaining fruit flies in your house. First, fill a small bowl with a few tablespoons
of vinegar. Then, put a piece of very ripe or rotting fruit into the vinegar. Cover the bowl very tightly with a sheet of
plastic wrap and poke a few very small holes in the wrap with a fork. If all goes according to plan, the flies will enter
the trap through the holes but will be unable to fly back out. This trap will catch all of the remaining fruit flies. You
can either kill these flies or release them outdoors.

Fruit flies can be a pest, but they do not have to make you crazy. With a little effort, you can get existing flies out of
your house and prevent new ones from taking over your kitchen.
31. In reading this passage, we learn that fruit flies
I. die immediately after they lay their eggs II. can grow into adults after only 8 days III. are mostly
harmless
(a) I only (b) I and II only (c) II and III only (d) I, II, and III

32. As used in paragraph 3, a nuisance is something that


(a) bothers you (b) can be dangerous (c) grows fast (d) is very small

33. According to the passage, why should you not use pesticide spray to kill fruit flies?
(a) most sprays do not actually kill all of the fruit flies
(c) buying sprays can get expensive if you need to use a lot of them
(c) The sprays take too much time to work
(d) it can be dangerous to use them in your kitchen

34. Based on the information in the penultimate (next-to-last) paragraph, we can understand that the author thinks
that
(a) some people do not mind having fruit flies in their house
(b) some people do not like killing insects

Pankaj Gandhi’s Academy


rd
3 Floor, Above Hotel Woodland, Near Sharda Center, Nal Stop, Pune. Contact No. : 8600972993, 9850424051.
(c) fruit flies do not like vinegar
(d) making a fruit fly trap can be difficult

Direction for (Q. 35 to 40) Each sentences below consist of a word or a phrase which is bold. It is followed by four
words or phrases. Select the word or pharse which is closes to the OPPOSITE in meaning of the bold word or
phrase.
35. History abounds in instances of courage.
(a) shines (b) lacks (c) suffices (d) fails

36. The inhabitants of the island were barbarians.


(a) civilized (b) cruel (c) uncivilized (d) bad

37. The members thought that the task was feasible.


(a) impractical (b) impossible (c) difficult (d) impracticable

38. crestfallen he returned as he had never faced such humiliation in the whole of his life.
(a) vainglorious (b) indignant (c) triumphant (d) disturbed

39. Feasibility of the project is under study.


(a) unsuitability (b) cheapness (c) impropriety (d) impracticability

40. Sathish point of view was correct but his behavior with his father was quite impertinent.
(a) healthy (b) respectful (c) inadequate (d) smooth

I English Sample Paper - 7


Directions for (Q. 1-9): Read the following passage carefully and answer the question given below it. Certain
words have been printed in bold to help you locate them while answering some of the questions.
Passage I The education sector in India is in ferment, hit by a storm long waiting to happen. The butterfly that
flapped its wings was the much-reiterated statement in a much publicised report that hardly a fourth of graduating
engineers, and an even smaller percentage of other graduates, w as of employable quality for IT -BPO jobs. This
triggered a cyclone when similar views were echoed by other sectors which led to widespread debate. Increased
industry academic interaction, " finishing schools", and other efforts were initiated as immediate measures to bridge
skill deficits. These, however, did not work as some felt that these are but band-aid solutions; instead, radical
systemic reform is necessary.
Yet, there will be serious challenges to overdue reforms in the educat ion system. In India-as in many countries-
education is treated as a holy cow sadly, the administrative system that oversees it has also been deceived. Today,
unfortunately, there is no protest against selling drinking water or paying to be cured of illness, or for having to buy
food when one is poor and starving; nor is there an out cry that in all these cases there are commercial companies
operating on a profit-making basis. Why then, is there an instinctively adverse reaction to the formal entry of 'for-
profit' institutes in the realm of education ? Is potable water, health or food, less basic a need, less important a
right, than higher education ?
While there are strong arguments for free or subsidized higher education, we are not writing on a blank page.
Some individuals and businessmen had entered this sector long back and found devious ways of making money,
though the law stipulates that educational institutes must be 'not-for profit' trusts or societies. Yet, there is
opposition to the entry of for-profit" corporate, which would be more transparent and accountable. As a result,
desperately needed investment in promoting the wider reach of quality education has been stagnated at a time
when financial figures indicate that the allocation of funds for the purpose is but a fourth of the need.

Well-run corporate organisations, within an appropriate regulatory framework, would be far better than the so -
called trusts which - barring some noteworthy except ions-are a blot on education. However, it is not necessarily a
question of choosing one over the other : different organisational forms can coexist, as they do in the health sector.
A regulatory framework which creates competition, in tandem with a rating system, would automatically ensure the
quality and relevance of education. As in sectors like telecom, and packaged goods, organisations will quickly
expand into the hinterland to tap the large unmet demand. Easy Loan/scholarship arrangements would ensure
affordability and access.

The only real structural reform in higher education was the creation of the institutes for technology and
management. They were also given autonomy and freedom beyond that of the universities. However, in the last
few years, determined efforts have been underway to curb their autonomy. These institutes, however, need
freedom to decide on recruitment, salaries and admissions, so as to compete globally.

Pankaj Gandhi’s Academy


rd
3 Floor, Above Hotel Woodland, Near Sharda Center, Nal Stop, Pune. Contact No. : 8600972993, 9850424051.
However, such institutes will be few. Therefore, we need a regulatory framework that will enable and encourage
States and the Center, genuine philanthropists and also corporate to set up quality educational institutions. The
regulatory system needs only to ensure transparency, accountability, competition and widely-available independent
assessments or ratings. It is time for radical thinking, bold experimentation and new structures; it is time for the
government to bite the bullet.

1. Why, according to the author, did the initiatives such as increased industry-academia and finishing schools did
not help to bridge the skill deficit ?
(A) These steps were only superficial remedies and the problem could be answered only by reforming the entire
education system.
(B) These initiatives operated on a profit-making basis rather than aiming at any serious systemic reforms.
(C) The allocation of funds of such initiatives was only one-fourth of the need.
(a) Only A (b) Only B (c) Only B and C (d) Only A and B (e)None of these

2. Which of the following suggestions have been made by the author to improve the state of education in India ?
(A) Allowing the corporate organisations to enter the education sector.
(B) Easy availability of loans and scholarships for making education more affordable.
(C) A rating system for all the organisations to ensure quality
(a) Only A (b) Only A and B (c) Only A and C
(d) All A, B and C (e) None of these

3. According to the author, what 'triggered a cyclone' which saw similar views on the state of education being
echoed across other sectors as well ?
(a) The campaign for allowing corporates in the education sector on a 'for-profit' basis'
(b) The support for the increase in the industry-academia interaction
(c) The report mentioning that only a small percentage of graduates were employable in software industry
(d) The report supporting the idea of making the education completely 'for-profit' in order to improve upon the
standards
(e) E None of these

4. Which argument does the author put forward when he compares the education sector with sectors catering to
health and potable water etc.
(a) Education should also be provided free of cost to all as health services and water.
(b) Taking an example from these sectors, there should be a protest against the commercialisation of education as
well
(c) Allowing corporate entry in education would result in rampant corruption as in the sectors of health and potable
water etc.
(d) As in these sectors, commercial organisations should also be allowed to enter the education sector
(e) None of these

5. What does the author mean by the phrase we are not writing on a blank page' in context of the passage ?
A. Corporates would never enter education if they are forced to function on a non-profit making basis
B. The commercialisation of education has already started in India
C. Education has been reduced to a profit making sector by some corporate organisations
D. Government will not allow corporates to enter education as India can't afford to have costly education
E. None of these

6. What is the author's main objective in writing the passage?


(a) To suggest the ways to improve quality of education in India
(b) To highlight the corruption present in the education sector
(c) To compare the education sector with other sectors
(d) To suggest some temporary solutions to the problems in education

7. According to the author, which of the following was the only step taken in order to reform the higher education ?
(a) Allowing organisations to enter the education sector on a 'for-profit' basis
(b)Creation of autonomous institutes for management and technology which were not under university
control
(c) Setting up the regulatory framwork for all the existing universities
(d) Making the availability of educational loans and scholarships easier

Pankaj Gandhi’s Academy


rd
3 Floor, Above Hotel Woodland, Near Sharda Center, Nal Stop, Pune. Contact No. : 8600972993, 9850424051.
8. Which suggestion does the author make in order to make the institutes of higher learning for technology and
management capable of competing globally ?
(a) To limit their autonomy to acceptable limit and give partial controls to the government
(b) To allow corporate organisations to take them over in order to privide more funds
(c) To increase the allocation of funds to such institutes
(d) To provide freedom to decide on recruitment, salaries and admissions
(e) None of these

9. Which of the following is not true in context of the given passage ?


(a) According to the law, education institutes should not be run for profit
(b) There has been no protest against the selling of drinking water and paying for the health services
(c) Orily either corporate organisations or government controlled organisations can exist in the education
sector
(d) The introduction of'for-profit' corporates in the education sector has been facing a lot of criticism
(e) All are true

Passage II
Supernatural events have always held a fascination for mankind. At various times in history, men have claimed to
possess extraordinary mental or psychic powers, such as the ability to see ghosts or spirits, to talk with the dead
and to foretell the future. Scientists have ignored such claims as fictitious and unscientific. But as reports of such
phenomena continued to pour in from all quarters, psychologists were forced to take notice of them. In 1882
professor Sidgwick founded the Society for Psychical Research. The aim of this society was to gather anecdotes
and stories of psychic phenomena that could not be explained by the law of nature. Since then the study of such
phenomena has gained some importance. It is known by the name of parapsychology. When people have ability to
'communicate' without the aid of their senses, they are said to possess extra-sensory perception
(ESP).Psychologists divide ESP into four types. The first is telepathy or thought transference. In telepathy a person
is able to communicate with another who may be far away. A soldier on the battle front
receives a message from his wife, or a father hears a call for help from his son who is far away. Stories like those
have been recorded and investigated by psychologists. Another type of ESP is clairvoyance, the ability to see, or
otherwise be aware of events, persons or things not known to anyone else. A clairvoyant person may be able to
locate a document that is hidden in a secret place or to find a missing child. The third type of ESP is precognition,
the ability to foresee what is going to happen in future. Finally there is psychokinesis (PK) the power of mind over
matter: a person having this ability will be able to move objects or displace them without touching them. The
scientist who has done most to study these phenomena is professor J.B. Rhine of Duke University in America. He
started with a study of clairvoyance from the written records of communication with spirits. He also conducted tests
with mediums, men or women who had the power to establish contact with spirits. But he could not get sufficient
evidence to accept such phenomena as scientific.

Rhine then went to conduct more practical tests in controlled conditions. He made use of cards to test people who
claimed to be clairvoyant. He used 25 cards in which 5 different colours and 5 different figures were used. The
subject (the person who was being tested) was asked to guess the figure or the colour on the card. The statistical
probability of guessing right was calculated and found to be 1 in 5. If any person consistently guessed more than 1
in 5, he probably had ESP. Rhine published the results of his study in 1937 under the title, "New Frontiers of the
Mind". This publication led to a big controversy among psychologists. Many of them still viewed parapsychology
with scepticism.The position today is that ESP is accepted by about 10 per cent of psychologists and rejected by
another 10 per cent. The remaining 80 per cent feel that the evidence now available is insufficient. In the meantime
authentic reports of ESP continue to come. Ingo Swann was able to copy with remarkable accuracy figures that
were placed on a shelf above her head, clearly out of range other vision. Examples of psychokinesis have come
from different places. Madame Kulagine of Russia moved matches that were scattered on a table into one heap
without touching them. In 1971 Uri Geller created a sensation when he showed that he was able to bend or
displace pieces of metal by the power of his mind.
10. If you have to talk about the main point from the first paragraph, you would say
(a) Supernatural Events (b) Professor Sidgwick’s Contribution
(c) Para Psychology-Answer (d) Law of Nature

11. What is ESP in one word?


(a) Telepathy (b) Clairvoyance (c) Communicate (d) Perception-Answer

12. ESP is accepted by about


(a)80 per cent of psychologists. (b) 10 per cent of psychologists.
(c) 20 per cent of psychologists. (d) 90 per cent of psychologists.

Pankaj Gandhi’s Academy


rd
3 Floor, Above Hotel Woodland, Near Sharda Center, Nal Stop, Pune. Contact No. : 8600972993, 9850424051.
13. The scientist who has done most to study these phenomena is
(a) J.B. Rhine-Answer (b) Ingo Swann (c) Sidgwick (d) Madame Kulagine

14. In the year _______ Uri Geller created a sensation when he showed that he was able to bend or displace
pieces of metal by the power of his mind.
(a)1937 (b) 1971 (c) 1882 (d) 1961

Directions for (Q.15-20): Fill in the blanks with the appropriate choice
15. Latin America has for long been a forgotten wasteland…………..Indian foreign policy
( a) to the politics controllers (b) to the politics controllers by
(c) to the political controllers by (d) to the politics controllers in-Answer

16. …………ahead of the herd.


(a) Intelligence investors make their move (b) Intelligence investors make its move
(c) Intelligent investors make their moves (d) Intelligent investors make their move

17. Hakeem said the Muslims had been overlooked in…………….


(a) earlier peace bids which ended in disaster (b) earliest peace bids which ended in disaster
(c) early peace bids which ended in disaster (d) earlier peace birds which end in disaster

18. The speaker was so ……and elaborate that the audience started getting fidgety
(a) terse (b) prolix (c) curt (d) brief

19. The Asian Games concluded with a colorful………….., in which, in addition to the athletes and players, a
number of artists also participated.
(a) ovation (b) tablet (c) pageant (d) finale

20. She…..her talents at the National School of dramatics.


(a) fine-tuned (b) stupefied (c) abraded (d) downgraded

21. Dr. Mak was so sad after seeing the __ conditions of the area affected by the disaster that she decided to
donate a million dollars to try to improve the situation.
(a) corrosive (b) squalid (c) commodious (d) discern (e) inter

22. he prisoners were ___ in demanding their rights, but they were very quiet when asked about their victims’
rights.
(a) extant (b) vociferous (c) abhor (d) martinet (e) voluminous

Luis studied vocabulary every night to try to ____ the chances of failing English again.
(a) turbulent (b) reprehensible (c) implicate (d) waive (e) obviate

23. The class agreed that most people _ crimes that involve the injury of children.
(a) inter (b) discern (c) corrosive (d) abhor (e) squalid

24. His father is so ___ that he wouldn’t even give Nicky a dollar.
(a) parsimonious (b) multifarious (c) consecrate (d) condolence (e). decrepit

25. The police were called because the noise coming from the wild __ at James’ house was disturbing the
neighbors.
(a) animosity (b) quandary (c) revel (d) reprisal (e) omnivorous

26. The police were called because the noise coming from the wild __ at James’ house was disturbing the
neighbors.
(a) animosity (b) quandary (c) revel (d) reprisal (e) omnivorous

27. The coach _(ed) to his past accomplishments as a football player when he first spoke to the new team.
(a) placate (b) allude (c) superficial (d) exemplary (e) plagiarism

28. Tiffany’s writing skills were so ___ that Avinash held her up as a model for all students to follow.
(a) placate (b) allude (c) superficial (d) fathom (e) exemplar

Pankaj Gandhi’s Academy


rd
3 Floor, Above Hotel Woodland, Near Sharda Center, Nal Stop, Pune. Contact No. : 8600972993, 9850424051.
Directions for (Q. 30 – 34) : In each of the following questions, find out which part of the sentence has an error.
The error may be idiomatic or grammatical. If there is no mistake, the answer is ‘No error’.
30. He drank once again (a)/ as he was (b)/ feeling thirsty. (c)/ No error (d)
31. Bacteria is probably (a)/ the most common form (b)/ of life on earth. (c)/ No error (d)
32. While going (a)/ through the report (b)/ yesterday I find (c)/ several factual mistakes. (d)/ No error (e)
33. On reaching the railway station (a)/ he was disappointed to learn (b)/ that the train left. (c)/ No error (d)
34. It is high time (a)/ that we send (b)/ the answer (c)/ No error (d)

Direction for (Q. 35 to Q.39) Rearrange the following sentences.


35. A. After doing so, the heart of your cash flow will be strong and healthy.
B. Cash is your business’s lifeblood.
C. If managed poorly, then your company could go into cardiac arrest.
D. To prevent your business from suffering heart attacks, you should learn to manage cash flow in a well thought-
out manner.
E. Several ways to do this is by generating a project rate of returns as well as determining possible problems with
liquidity.
(a) ABECD (b) CBADE (c) BCDEA (d) ABDEC

36. A. This factor is exclusion – access to these technologies remains excluded by class, race and gender.
B. In comparing these two things, we must realize that there is one important factor for the limitation the former.
C. The rise of digital technologies has the potential to open new directions in ethnography.
D. Despite the ubiquity of these technologies, their infiltration into popular research methods is still limited
compared to the number of online scholarly research portals.
(a) CDBA (b) BACD (c) CDAB (d) BCDA

37. A. Adam Smith is often described as the “founding father of economics”.


B. One prominent book he wrote was “Theory of Moral Sentiments”.
C. A great deal of what is now considered standard theory about markets was developed by Adam Smith.
D. It is a very important text in the history of moral and political thought because he talks about individual freedom.
E. According to Smith, this freedom is rooted in self-reliance and the ability of an individual to pursue his self-
interest.
(a) EDACB (b) ACBED (c) EBADE (d) ACBDE

38. A. Today, the study of the fruit fly has expanded to research for human diseases.
B. This is because the fruit fly’s robust genetic system makes it an invaluable tool for scientists studying current
inheritance diseases.
C. These diseases include Alzheimer’s, Parkinson’s and Huntington’s disease.
D. In past decades, scientists have used the fruit fly as a model organism for examining biological systems.
(a) DABC (b) DACB (c) BACD (d) CBAD

39. A. It is one of a group of disorders known as parasomnias: unusual activities that occur during sleep.
B. Sleepwalking occurs in the deep stage of sleep when slow brain waves begin to appear.
C. These range from teeth grinding and restless leg syndrome to eating while asleep.
D. Our bodies function according to a 24 hour cycle called a circadian rhythm.
E. Some researchers believe that slight differences in this cycle could be linked to sleepwalking.
(a) BACED (b) CDBAE (c) EBACD (d) BACDE

Pankaj Gandhi’s Academy


rd
3 Floor, Above Hotel Woodland, Near Sharda Center, Nal Stop, Pune. Contact No. : 8600972993, 9850424051.
I English Sample Paper – 8

Direction for (Q. 1 - 5) Fill in the blanks.


1. This sort of an aggressive attitude would only serve to ………the supporters you already have.
(a) Encourage (b) exhort (c) alienate (d) cooperate

2. Focus on solving the refugee problems and drawing plans for …………development was less evident.
(a) Short-range (b) sustainable (c) needless (d) merit

3. The groups were feared but at the same time …….by the locals
(a) Revered (b) detested (c) rebelled (d) loathed

4. The defending champion justified his top........ by clinching the title


(a) Skill (b) form (c) technique (d) billing

5. We must ...... our students on subjects like health and sanitation besides the usual subjects
(a) Learn (b) teach (c) insist (d) educate

Direction for (Q. 6 – 10) In each of the following questions, find out which part of the sentence has an error. The
error may be idiomatic or grammatical. If there is no mistake, the answer is ‘No error’.
6. The presumption that the average investor did not understand (a)/ or take interest in the affairs of the
company (b)/ is not correct. (c)/ No error (d)
7. The two last (a)/ chapters of the book (b)/ are very interesting. (c)/ No error (d)
8. Passengers should be prohibited (a)/ to smoke in (b)/ the trains and buses. (c)/ No error (d)
9. The world (a)/ comprises (b)/ good and bad people. (c)/ No error (d)
10. Honesty and integrity are (1)/ the qualities which cannot be (2)/ done away with (3)/ and hence assume a lot of
importance (4)/ No error (5)

Direction for (Q. 11 - 20) Read following passages and answer the questions based on these passage.
When a young student from Chandigarh was told that he had developed a tumour in a critical area of the brain and
worse, that it was too dangerous to perform surgery in the area, paralysis and eventual death seemed to be his
fate. Until doctors at Apollo Hospital, Hyderabad, Told him about a revolutionary treatment, radio-surgery which
eradicates deep-seated lesions in the most critical areas of the brain without opening the skull. He is now in
Sweden awaiting treatment and a return to a completely normal life.
A Swedish doctor, Lars Leskell, developed radio surgery, or the destruction of diseased brain tissue through the
delivery of a concentrated dose of radiation to pinpointed area in the brain. Since brain tissue, unlike tissue
elsewhere in the body, does into regenerate itself, mistakes in neurosurgery can be crippling and permanent.
Leskell first conceived the idea of radio surgery and after two decades of research to identity the ideal radiation
source and equipment, developed the Gamma knife in 1968. While the procedure takes three hours, actual
treatment time in the Gamma knife unit is 10 to 20 minutes. The patient's head is positioned in a helmet which
places the radiation target at the exact point of intersection of 201 cobalt 60 beams. At this point of convergence,
enough radiation is delivered to destroy deceased tissue without affecting the adjacent normal tissue. Radiation is
precisely delivered to the affected tissue and the dose absorbed by surrounding cerebral tissue is minimal. In 1968,
the first Gamma knife was installed at the Karolinska Institute, Stockholm. A second Gamma knife was installed at
the institute in 1974. The Gamma knife however, came into its own following the development of sophisticated
imaging techniques. "Today there are around 15 units in use in leading institutions all over the world, while 15 are
now being installed," says Percy Shroff, manager of the India operations of Elekta Instrument AB, Sweden that
manufactures medical equipment in the field of neuroscience.
"No such systems have been installed in India which has led quite a few patients to seek the treatment overseas.
Taking India's patient population, the number of cases for which the treatment is suitable would be around 100 per
million," reckons Shroff. At Rs. 12 crore, the Gamma Knife is a hefty investment but one that is cost-saving in the
long-run. Treatment costs are reduced by 30 per cent to 70 percent as compared to conventional surgery.
Furthermore, the hidden costs of prolonged convalescence after open surgery of the brain are eliminated since the
patient leaves hospital the same day or a day after treatment. The producer is virtually painless as there is neither
incision nor any need for general anaesthesia. "Not only is the patient spared the pain, he is also spared the
problems associated with traditional invasive surgery such as bleeding, infection, formation of scar tissue and
lengthy hospitalisation and convalescence" says Dr. Keki E. Turel, consultant neurosurgeon and professor of
neurosurgery, Mumbai Hospital Institute of Medical Sciences. Given the cost benefits, the Gamma knife is a
suitable option for Government hospitals. In China, it has been installed in four large public hospitals where

Pankaj Gandhi’s Academy


rd
3 Floor, Above Hotel Woodland, Near Sharda Center, Nal Stop, Pune. Contact No. : 8600972993, 9850424051.
treatment is provided at a cost of $3000 to $5000 compared to $20,000 to $25,000 in the US 10,000-15,000 in
Japan and Europe.
Neurosurgeons in India expect to make the treatment available at the same cost as in China. The continuing
refinement of imaging techniques has correspondingly increased the potential applications of the Gamma knife,
with further development, it can be used to ameliorate the symptoms of Parkinson's diseases, and provide relief to
patients suffering from intractable pain neurolgia, psyche-neurosis and epilepsy. Already of the 21,428 cases
treated worldwide around 2 percent account for such therapy, Tumours account for 62 percent of treatments of
which malformations of blood vessels account for 36 percent. With ongoing research, the applications of the
Gamma knife should spread to the other specialties also. Research related to inner ear and retinal cancers is
currently underway.

11. The gamma knife reduces the cost of treatment of brain tumour by
(a) 80% to 90%. (b) 20% to 80%. (c) 30% to 70%. (d) By 50%.

12. In China, the treatment through gamma knife costs


(a) $3,000.00 to $5,000.00 (b) $20,000.00 to $25,000.00
(c) $10,000.00 to $15,000.00 (d) $10,000.00 to $17,000.00

13. Consultant Neuro surgeon, Mumbai hospital Institute of Medical Science, is ___ (as mentioned in the passage)
(a)Lars Leskell. (b) Keki.E. Turel. (c) Percy Shroff. (d) N. Ezekiel.

14. The first gamma knife was installed in


(a) Karolinska Institute(b) Apollo Hospital (c) Mumbai Hospital Institute of Medical Science (d) Elekta
Instruments

15. Gamma knife is use to cure


(a) Parkinson’s Disease (b) Epilepsy (c) Brain Tumour (d) Neurolgia

Passage II
Philadelphia is a city known for many things. It is where the Declaration of Independence was signed in 1776, and
it was also the first capital of the United States. But one fact about Philadelphia is not so well-known: it is home to
nearly 3,000 murals painted on the sides of homes and buildings around the city. In fact, it is said that Philadelphia
has more murals than any other city in the world, with the exception of Rome. How did this come to be?
More than 20 years ago, a New Jersey artist named Jane Golden started a program pairing troubled youth with
artists to paint murals on a few buildings around the city. From this small project, something magical happened.
The young people involved helped to create magnificent pieces of art, but there were other, perhaps more
important benefits. The young people learned to collaborate and get along with many different kinds of people
during the various steps required to paint and design a mural.
They learned to be responsible, because they needed to follow a schedule to make sure the murals were
completed. They also learned to take pride in their community. It is hard for any resident to see the spectacular
designs and not feel proud to be a part of Philadelphia.
Take a walk around some of the poorest neighbourhoods in Philadelphia, neighbourhoods full of broken windows
and littered front steps, and you will find beautiful works of art on the sides and fronts of buildings. Of course the
murals are not just in poor neighbourhoods, but more affluent ones as well.
Special buses take tourists to different parts of the city to see the various murals, which range from huge portraits
of historical heroes, to cityscapes, to scenes depicting the diverse ethnic groups that call Philadelphia home.
As a result of its success, the mural program created by Jane Golden has now become the nation’s largest public
art program and a model for other cities throughout the country seeking to help troubled youth.
16. The main focus of the passage is
(a) An art program designed to help troubled youth
(b) The many tourists who come to Philadelphia to see murals
(c) The reasons why Philadelphia is a unique city
(d) How Jane Golden came up with the idea to start a mural program

17. As used in paragraph, the phrase “it is said” suggests that the author is
(a) Knowingly misleading the reader (b) using a quote from someone else
(c) Referring to something that is widely believed, but may be untrue
(d) Referring to something that he or she does not personally believe

18. As used in paragraph 1, the phrase “with the exception of Rome” means that
(a) Rome has fewer murals than Philadelphia (b) Philadelphia has fewer murals than Rome

Pankaj Gandhi’s Academy


rd
3 Floor, Above Hotel Woodland, Near Sharda Center, Nal Stop, Pune. Contact No. : 8600972993, 9850424051.
(c) Rome has the most beautiful murals of all (d) Rome and Philadelphia are the only cities with murals

19. According to the passage, the murals in Philadelphia


I. draws tourists who want to see them
II. Instil responsibility and pride in the people who paint them
III. Are solely designed by the youth who paint them
(a) I only (b) I and II only (c) II and III only (d) I, II, and III

20. As used in paragraph 3, which is the best definition of affluent?


(a) Popular (b) clean (c) well-known (d) wealthy

Direction for (Q. 21 to 25) Rearrange the following sentences.


21. A. Based on these findings such as these, scientists agree that we will soon discover intelligent alien life and
make contact with it.
B. But this complacency has all changed.
C. Until recently, we have looked at life in a pretty conservative way.
D. In just the past three or four years, biologists have discovered that life on Earth can exist in the most extreme
places, and in the most bizarre forms.
E. For example, microbes have been found down boreholes two kilometres deep.
(a) BAECD (b) CEDAB (c) CBDEA (d) EABDC

22. A. For me, it was a restaurateur, who screamed at me for cutting my finger while preparing a salad.
B. He did not even care so much as to wonder if I was unharmed.
C. Everyone has a story about working for an uninspiring boss.
D. As confidence in workplaces dims because of turmoil in the financial markets, now is the time for leaders to step
up and communicate more.
E. Of course, before you can commit yourself to becoming the type of boss people want to follow, you must
recognize the qualities of bosses who fail to elicit enthusiasm.
(a) BADEC (b) CABDE (c) CBADE (d) BECDA

23. A. These mediums allowed for a greater number of listeners, they quickly garnered popularity in America as
well.
B. They would also play in clubs that were popular at the time.
C. What they were trying to do was play “live music” on the Bucharest radio station.
D. The very first signs of Romanian jazz music appeared about seventy years ago, when some jazz bands
appeared, like the ones led by Emil and Mihai Berinde.
(a) DCBA (b) BCDA (c) BCAD (d) ABCD

24. A. These mediums allowed for a greater number of listeners, they quickly garnered popularity in America as
well.
B. They would also play in clubs that were popular at the time.
C. What they were trying to do was play “live music” on the Bucharest radio station.
D. The very first signs of Romanian jazz music appeared about seventy years ago, when some jazz bands
appeared, like the ones led by Emil and Mihai Berinde.
(a) DCBA (c) BCDA (c) BCAD (d) ABCD

25. A. The issue of abolishing the monarchy as a whole and declaring itself a republic, free of the Queen, has
been suggested several times in Canada’s Parliament.
B. Legislation to abolish this has never passed, as we never cease to recognize key function of the Queen as a
symbol of loyalty.
C. Most importantly, though, the Queen symbolizes stability, tradition and certain romanticism about political affairs
within the constitutional monarchy.

Pankaj Gandhi’s Academy


rd
3 Floor, Above Hotel Woodland, Near Sharda Center, Nal Stop, Pune. Contact No. : 8600972993, 9850424051.
D. The concept of loyalty as often been viewed as an important part of ethics, and we should not forget this vital
fact.
(a) BCDA (b) CADB (c) ABDC (d) CABD

26. Which of the following is an appropriate synonym for the word Debauch?
(a) Demoralize (b) Encourage (c) Cultivate

27. Find the synonym of Decreed?


(a) Made up one's mind (b) disagree (c) decrease in quantity

28. What is an appropriate synonym for Bifid?


(a) Divided (b) Divided in two (c) Timid
29. What is an antonym of incidental?
(a) Permissible (b) usual (c) conventional (d) intentional
30. What is an antonym of tactful?
(a) Disciplined (b) naive (c) strict (d) loos

I English Sample Paper - 10


Direction for (Q. 1 - 10) Read the following passage answer the question based on these passage.
With the development of modern theoretical astrophysics, astronomers are able to explain data collected by early
observers and to amass data about previously unknown phenomena in extragalactic optical astronomy. Despite
our lack of understanding of these new phenomena , the data is being used for a variety of purposes. In certain
cases it is possible, by5 application of known physical laws, to make theoretical predictions that are subject to
observational tests. he prediction of neutron stars is a classic example of the use of this type of theory for purposes
of prediction. Another is to be found in the field of cosmology .Much of the current activity in extragalactic
astronomy is directed towards deciding which if any, of the theoretical models of the universe is the most accurate
.Starting from Einstein's 10 MISS, that 15 proposed a radically different approach - the steady-state theory of the
universe. This cosmology dispensed with the idea of a primordial singularity demanded by Hubble's theory and
substituted the concept of continuous creation of matter, which would maintain approximately constant mean
density. It is largely within the context of these two theory models that observational work has found both its
inspiration and its justification. Theoretical 20 predictions of the type outlined above are of particular importance to
astronomy, which is an observational rather than an experimental science. Predictive theory that is concerned with
the consequences of physical laws is thus immensely productive in an astronomical context and can lead to lively
interaction with the observational aspects of the subject .It would, however, be unfair to suggest that theory usually
plays such a guiding role 25 in astronomical endeavor. Indeed, more often, theories are motivated by discoveries.
As a result the second major role of theory in astronomy is that of interpretation. In the field of stellar evolution, for
example, the general features of the Hertzprung-Russell diagram (essentially a plot of luminosity versus
temperature) had been known for many years before a theoretical interpretation in terms of nuclear processing in
unmixed stars could be given.30 This quantitative explanation opened the way to the development of a method for
using the results I of stellar-evolution calculations to provide us with quantitative information about stars. In
principle, this method can be used to determine the approximate stellar abundances of the chemical elements,
stellar masses, and hence stellar ages. Indeed, stellar-evolution theory provides us with the only widely applicable
method of dating stellar systems
1. The primary purpose of the passage is to
(a)Elucidate ways in which astronomy differs from other sciences.
(b) Discuss the roles of observation and theory in astronomy.
(c) Present two conflicting views of the origin of the universe.
(d) Demonstrate how the laws of physics apply to astronomy

2. Explain the importance of stellar-evolution theory. Which of the following represents a methodological handicap
in the field of astronomy?
(a) The difficulty of formulating predictive theories.
(b) The impossibility of evaluating data collected by early observers.
(c) The difficulty of correlating observed data with theoretical predictions
(d) The theoretical problem of reconciling the laws of astronomy with known laws of physics.

3. The difficulty of testing hypotheses through experimentation It can be inferred that the author considers
predictive theory to be important in astronomy , because it

Pankaj Gandhi’s Academy


rd
3 Floor, Above Hotel Woodland, Near Sharda Center, Nal Stop, Pune. Contact No. : 8600972993, 9850424051.
(a) Can be easily verified on the basis of objective data.
(b) May provide a framework for further astronomical observation
(c) May result in confirmation of known physical laws
(d) Confirms the validity of data on the origins of the universe.
(e) Often contributes to the advancement of other sciences

4.The passage would be most likely to appear in which of the following


(a) A specialized monograph on astrophysics and its methods.
(b) A newspaper article discussing cosmology.
(c) A popular discussion of basic physical laws.
(d) A theoretical essay on chemical elements in the universe.

5. A popular article on techniques of optical observation. With which of the following statements concerning both
the predictive and the interpretive roles of astronomical theory would the author be most likely to agree?
(a) Both stimulate new research and provide a background against which astrophysical phenomena can be
understood.
(b) Both focus on discovering which model of the universe is correct
(c) Both are useful for validating information collected by observation as well as for testing the application of known
physical laws.
(c) Both have contributed about equally to every advance in astronomy
(d) Both have as their primary objectives the formulation of new hypotheses

Passage 2
The issue of women, art, and feminism has been most urgently raised by a number of women artists. Several
approaches to the problem of defining feminist art have evolved and are being discussed and developed within the
feminist art movement. One particular approach has suggested that some sort of female aesthetic or sensibility
exists, involving an imagery and formal style specific to women. Proponents insist that an authentic artistic
language is being created, corresponding to the distinct social experience of women, independent of "male-
defined" art, and essentially liberating. Others argue that the theory of a female aesthetic really restricts women in
that it limits them to certain "feminine" shapes, colors, forms, and images. In other words, the female aesthetic
seems possibly to be no more than a rehabilitated artistic ghetto, furbished with less than satisfactory answers to
the hard question of how to define feminist art. Moreover, some see the rise of a trendy "feminine sensibility" as
clearly opportunist. They point, for example, to the odd coincidence that the so-called female aesthetic is strangely
reminiscent of the conventions of much currently fashionable art, and they predict further shifts in the aesthetic as
art-world fashions change.
The theory of a female sensibility seems to be based on two equally extreme premises, implicit and not explicit.
First, it assumes that an individual's experience is primarily and perhaps completely determined by gender. Women
and men are held to inhabit utterly separate worlds, and variations of social or ethnic experience are considered
clearly subordinate to gender distinctions. Its second assumption is that whatever exists today must be essentially
unchangeable as the battle of the sexes is eternal and historical. It follows, then, that the only way women artists
can operate is to accept these terms and develop their own artistic strengths, autonomously and in opposition to
men. Another approach, both balanced and sensible, would argue for a more transcendental view of social
experience and of art. Such a point of view corresponds to the opinion within some sectors of the women's
movement that the meaning of one's personhood and the nature of relationships between the sexes are an
evolving phenomena that can be grasped and acted upon. Pat Mainardi has outlined one interpretation of what this
might mean for women artists: "The only feminine aesthetic worthy of the name is that women artists must be free
to explore the entire range of art possibilities. We who have been labeled, stereotyped, and gerrymandered out of
the very definition of art must be free to define art, not to pick up the crumbs from the Man's table ... We must begin
to define women's art as what women (artists) do, not try to slip and squeeze ourselves through the loophole of the
male art world."
6. Which of the following the best describes the content of the passage
(a) The Impact of the Women's Art Movement on Aesthetic Theory.
(b) The Female Aesthetic: Its impact on Artistic Conventions.
(c) An Examination of the Principles and Assumptions of the Theory of a Female Aesthetic.
(d) Feminism, Women's Liberation, and Aesthetic Theory
(e) The Uniqueness of Women's Art in the Male Art World.

7. The primary purpose of the passage is to


(a) Initiate a debate. (b)Summarize related points of view. (c)Define terms and illustrate their
applications
(d) Criticize an approach and suggest an alternative one (e) Criticize competing theories

Pankaj Gandhi’s Academy


rd
3 Floor, Above Hotel Woodland, Near Sharda Center, Nal Stop, Pune. Contact No. : 8600972993, 9850424051.
8. It can be inferred from the passage that the author would most probably agree with which of the following
statements about relationships between men and women?
(a) Women can develop their own talents most successfully by working completely independent of men
(b) Women and men share no common ground of personal experience
(c) The contemporary relationships between men and women should not affect the work of women artists
(d) Relationships between men and women are not static and can be influenced by new ideas about women's
identities
(e) A consequence of the nature of relationships between men and women is that women can develop their art only
by seeing men as opponents

9. History has not beer kind to Sara Teasdale, but she won a Pulitzer award and saw a book of her verse or the
best-seller list, a feat none of the poets of today will likely duplicate
(a) None of the poets of today will likely duplicate. (b) No poet today is likely to duplicate.
(c) No poet today will likely duplicate. (d) Poets of today are not likely to reduplicate
(e) Likely to be unduplicated by poets of today.

Directions (10-20) Find the correct alternative for the underlined part
10. According to a government study, the lush swamps and marshes of the Mississippi ,one among the finest
wetlands in the world, are vanishing at a rate of 39 square miles a year, as fast as two and a half times the rate that
was previously thought.
(a) As fast as two and a half times the rate that was (b) Two and a half times as fast as it had been
(c) A rate two and a half times as fast as (d) Which is a rate two and a half times faster
than had been

11. Later he became unpopular because he tried to lord it on his followers


(a) to lord it for (b) to lord over (c) to lord it over (d) to lord it over on (e) No correction required

12. The crops are dying; it must not had rained.


(a) must had not (b) must not be (c) must not have (d) must not have been
(e) No correction required

13. The courts are actively to safeguard the interests and the rights of the poor
(a) are actively to safeguarding (b) have been actively safeguarding (c) have to active in safeguarding
(d) are actively in safeguarding (e) No correction required

14. The drama had many scenes which were so humorous that it was hardly possible to keep a straight face.
(a) hardly possible for keeping (b) hardly impossible for keeping (c) hardly impossible for keep
(d) hardly possible keeping (e) No correction required

15. Hardly does the sun rise when the stars disappeared.
(a) have the sun rose (b) had the sun risen (c) did the sun rose
(d) the sun rose (e) No correction required

16. You will be late if you do not leave now


(a) did not leave (b) left (c) will not leave (d) do not happen to (e) No correction
required

17. The train will leave at 8.30 pm, we have been ready by 7.30 pm so that, we can reach the station in time.
(a) were (b) must be (c) are (d) should have (e) No correction required

18. All the allegations levelled against him were found to be baseless
(a) level against (b) level with (c) levelling with (d) levelled for (e) No correction
required

19. Ramesh is as tall if not, taller than Mahesh.


(a) not as tall but (b) not so tall but as (c) as tall as, if not (d) as if not (e) No correction required

20. He hesitated to listen to what his brother was saying


(a) listened to hesitate (b) hesitated listen to (c)hesitates to listening
(d) is hesitated to listen (e) No correction required

Pankaj Gandhi’s Academy


rd
3 Floor, Above Hotel Woodland, Near Sharda Center, Nal Stop, Pune. Contact No. : 8600972993, 9850424051.
Directions for (Q. 21-25) find the proper sequence
21. S1: We now know that the oceans are very deep.
P: For example, the Indian ocean has a range called the Indian Ridge.
Q: Much of it is fairly flat.
R:However, there are great mountain ranges as well.
S:On average the bottom is 2.5 miles to 3.5 miles down
S6:This reaches from the India to the Antarctic.
(a) SQPR (b) PQSR (c) RSQP (d) QPRS

22. S1 : Minnie went shopping one morning.


P : Disappointed She turned around and returned to the parking lot
Q : She got out and walked to the nearest shop.
R : She drove her car into the parking lot and stopped
S : It was there that she realised that she'd forgotten her purse at home.
S6:She drove home with an empty basket.
(a) RSQP (b) RQSP (c) PQRS (d) QPRS

23. S1: Far away in a little street there is a poor house


P: Her face is thin and worn and her hands are coarse, pricked by a needle, for she is a seam stress
Q: One of the windows is open and through it I can see a poor woman
R: He has a fever and asking for oranges
S: In a bed in a corner of the room her little boy is lying ill
S6: His mother has nothing to give but water, so he is crying
(a) SRQP (b) PQSR (c) QPSR (d) RSPQ

24. S1: A noise started above their heads


P : But people did not take it seriously
Q : That was to show everyone that there was something wrong
R : It was a dangerous thing to do
S : For, within minutes the ship began to sink
S6 :Nearly 200 lives were lost on the fateful day
(a) PQSR (b) PRQS (c) QPRS (d) QPSR

25. S1: American private lies may seem shallow


P: Students would walk away with books they had not paid for
Q: A Chinese journalist commented on a curious institution: the library
R: Their public morality, however, impressed visitors
S: But in general they returned them
S6: This would not happen in china, he said
(a) PSQR (b) QPSR (c) RQPS (d) RPSQ

Direction for (Q.26 - 32) Pick out the most effective word(s) from the given words to fill in the blank to make the
sentence meaningfully complete.
26. Fate smiles ...... those who untiringly grapple with stark realities of life
(a) with (b) over (c) on (d) round

27. The miser gazed ...... at the pile of gold coins in front of him.
(a) avidly (b) admiringly (c) thoughtfully (d) earnestly

28. Catching the earlier train will give us the ...... to do some shopping.
(a) chance (b) luck (c) possibility (d) occasion

30. To err is ...... to forgive divine.


(a) beastly (b) human (c) inhuman (d) natural

31. The ruling party will have to put its own house ...... order.
(A) in (b) on (c) to (d) into

32. ...... of old paintings is a job for experts.


(a) Resurrection (b) Retrieval (c) Restoration (d) Resumption

Pankaj Gandhi’s Academy


rd
3 Floor, Above Hotel Woodland, Near Sharda Center, Nal Stop, Pune. Contact No. : 8600972993, 9850424051.
Directions (Q.33- 40): In the following questions, out of four alternatives, choose the one which can be substituted
for the given word/sentence.
33. Extreme old age when a man behaves like a fool
(a) Imbecility (b) Senility (c) Dotage (d) Superannuation

34. To be courageous and have the ability to keep calm in difficult situations
(a) vapid (b) sanguinary (c) pliocene (d) sangfroid

35. It is very difficult to retain all that you hear in the class.
(a) keep (b) recall (c) preserve (d) conserve

36. The great artist life was full of vicissitudes.


(a) sorrows (b) misfortunes (c) changes (d) surprises

37. She has an insatiable love for music.


(a) unsatisfiable (b) unchanging (c) irreconcilable (d) undesirable

38. The great dancer impressed the appreciative crowd by his nimble movements.
(a) Unrhythmic (b) lively (c) quickening (d) clear

39. The visitor had a bohemian look.


(a) hostile (b) unconventional (c) sinister (d) unfriendly

40. The bullet wound proved to be fatal and the soldier died immediately.
(a) grievous (b) dangerous (c) serious (d) deadly

I English Sample Paper 11


Direction for (Q 1-7). Select the most appropriate antonym of each word.
1. Abstract
(a) Writ (b) Unencumbered (c) Loquacious (d) Concrete
2. Surcharge
(a) Dilate (b) Undercharge (c)Precursory (d) Gregarious
3. Astringent
(a)Turgid (b) Impervious (c) Nonastringent (d) Grandiloquent
4. Moderate
(a) Immoderate (b) Resuscitation (c) Sullied (d) Yarn
5. Precise
(a) Desultory (b) Imprecise (c) Trite (d) Exigency
6. Migratory
(a) Implicate (b) Perfunctory (c) Nonmigratory (d) Inculcate
7. Acknowledge
(a) Deny (b) Inured (c) Obtrusive (d) Dislodge
8. The plantation workers were on a collision course before the labor officer intervened.
(a) retaliatory (b) perfunctory (c) conciliatory (d) circuitous

Direction for (Q. 9 to 11) Correct the underline sentences.


9. Although the average Girl Scout joins the organization at age 14, they have already been Brownies for several
years.
(a) they have already been Brownies (b) they have already been Brownies
(c) she has already been a Brownie (d) she had already been a Brownie
(e) she was in the Brownies

10. Despite seeming tame and intelligent, the state troopers had no choice but to shoot the loose Gorilla.
(a) Despite seeming tame and intelligent (b) Despite a tame and intelligent appearance
(c) Although appearing to be tame and intelligent (d) Although it seemed tame and intelligent
(e) However tame and intelligent in appearance

Pankaj Gandhi’s Academy


rd
3 Floor, Above Hotel Woodland, Near Sharda Center, Nal Stop, Pune. Contact No. : 8600972993, 9850424051.
11. The number of workers have increased steadily each year.
(a) have increased steadily (b) steadily have increased (c)have been increasing steadily
(d) has increased steadily has steadily been going through an increase

Direction (Q. 12 - 14) Select the most appropriate meaning of each word.
12. Bliss
(a) Repudiate (b) Turbid (c) Cant (d)happiness

13. Officious
(a) Busy (b) Turmoil (c) Aggravate (d) Finicky

14. Extensive
(a) Capacious (b) Intensive (c) Piquant (d) Insularity

Direction for (Q. 15 – 20) There are two gaps in the following sentence. From the pairs of words given, choose the
one that fills the gaps most appropriately. The first word in the pair should fill the first gap.
15. The British retailer, M&S, today formally ..... defeat in its attempt to ..... King's, its US subsidiary, since no
potential purchasers were ready to cough up the necessary cash.
(a). admitted, acquire (b). conceded, offload (c). announced, dispose (d). ratified,
auction

16. Early.....of maladjustment to college culture is ..... by the tendency to develop friendship networks outside
college which mask signals of maladjustment.
(a) treatment, compounded (b) detection, facilitated (c) identification, complicated (d) prevention, helped

17. The ..... regions of Spain all have unique cultures, but the ..... views within each region make the issue of an
acceptable common language of instruction an even more contentious one.
(a) different, discrete (b) distinct, disparate (c) divergent, distinct (d) different, competing

18. A growing number of these expert professionals ..... having to train foreigners as the students end up ..... the
teachers who have to then unhappily contend with no jobs at all or new jobs with drastically reduced pay packets.
(a) resent, replacing (b) resist, challenging (c) welcome, assisting (d) are, supplanting

19. Companies that try to improve employees' performance by ..... rewards encourage negative kinds of behaviour
instead of ..... a genuine interest in doing the work well.
(a) giving, seeking (b) bestowing, discouraging (c) conferring, discrediting (d) withholding, fostering

20. God is ......


(a) graceful (b) gracious (c) grateful (d) greatful

Direction for (Q. 21 - 23) Read the each sentence to find out whether there is any grammatical error in it. The
error, if any will be in one part of the sentence. The letter of that part is the answer. If there is no error, the answer
is 'D'.
21. (a) Will you lend me (b) little money (c) to tide over this crisis. (d) No error.

22. (a) He gave them no money (b) nor did help them (c) in any way (d) No
error.

23. (a) The Sharmas (b) are living in this colony (c) for the last eight years. (d) No error
.
Direction for (Q. 24 - 26) In the following the questions choose the word which best expresses the meaning of the
given word.
24. Virtuoso:
(a) Skilled performer (b) Amateur (c)Good person (d) Professional

25. Arena: Conflict


(a) Mirage: Reality (b) Forum: Discussion (c) Asylum: Pursuit (d) Utopia: Place

26. Hierarchy: Ranked


(a) Equation: Solved (b) Critique: Biased (c)Chronology: Sequential (d)Infinity:
Fixed

Pankaj Gandhi’s Academy


rd
3 Floor, Above Hotel Woodland, Near Sharda Center, Nal Stop, Pune. Contact No. : 8600972993, 9850424051.
Direction for (Q. 27 - 31) Pick out the most effective word(s) from the given words to fill in the blank to make the
sentence meaningfully complete.
In the question, the word at the top (Bold) is used in four different ways, numbered 1 to 4. Choose the option in
which the usage of the word is INCORRECT or INAPPROPRIATE.
27. Bundle
(a) The newborn baby was a bundle of joy for the family.
(b) Mobile operators are offering a bundle of additional benefits.
(d) He made a bundle in the share market.
(e) It was sheer luck that brought a bundle of boy-scouts to where I was lying wounded.

28. Distinct
(a) He is distinct about what is right and what is wrong.
(b) Mars became distinct on the horizon in the months of August.
(c) The distinct strains of Ravi's violin could be heard above the general din.
(d) Ghoshbabu's is a distinct case of water rising above its own level.

29. Implication
(a) Everyone appreciated the headmaster's implication in raising flood relief in the village.
(b) This letter will lead to the implication of several industrialists in the share market scam.
(c) Several members of the audience missed the implication of the minister's promise.
(d) Death, by implication, is the only solution the poem offers the reader.

30. Host
(a) If you host the party, who will foot the bill (b) Kerala's forests are host to a range of
snakes.
(c) Ranchi will play the host to the next national film festival. (d) A virus has infected the host computer.

Direction for (Q. 31 – 35) Rearrange the Sentences.


31. 1. By the time he got to Linjeflug four years later, he had learned many lessons, in fact, he began his second
stint as top dog by calling the entire company together in a hanger and asking for help, a far cry from his barking
out commands just 48 months back.
2. At SAS, he arrived at a time crisis.
3. This book is chock-a-block full of intrusive stories and practical advice, describing Carton’s activities at Vingresor
(where he assumed his first presidency at age 32), Linjeflug, and SAS in particular.
4. He began at Vingresor as an order giver, not a listener – neither to his people nor to his customers and made
every mistake in the book.
(a)2143 (b) 2134 (c) 3214 (d) 3412

32. 1. They argue that it is this, which has led to the bankruptcy in many states.
2. Here was a commission whose members worked very hard, did exemplary research and homework, before
coming up with a list of recommendations that balanced economic efficiency with safety nets for disadvantaged
labour.
3. It reminds us of the political shenanigans during the implementation of the Fifth pay Commission.
4. How many times have you heard experts, politicians and the finance minister refer to the implementation of the
pay hikes following the commission's report as the singular cause for the increase in government expenditure?
5. Barring P. Chidambram, who was then the finance minister, every single political party and politician opposed
the implementation of the recommendations and are directly responsible for the current fiscal crises in the Centre
and the states.
(a)42513 (b) 34125 (c) 25143 (d) 45213

33.1. arrived here on Sunday by an early morning flight.


2). Dubai-based Win Gautam who is the
3). accused in the Rs 50 crore Before guns kickback case.
4). He is scheduled to appear in the trial court By Wednesday.
(a) 2134 (b) 2314 (c) 1432 (d) 1342

34. 1. the main difference is that efficiency is a ration and effectiveness is not.
2. But they reach efficiency in a different way than American businesses.
3. The Japanese are very efficient and such concepts as "just in time" are a witness to their efficiency.
4.They reach efficiency through the route of effectiveness.

Pankaj Gandhi’s Academy


rd
3 Floor, Above Hotel Woodland, Near Sharda Center, Nal Stop, Pune. Contact No. : 8600972993, 9850424051.
(a) 3241 (b) 1432 (c) 3142 (d) 4312

35. 1. The general impressions that skilled negotiators seem to convey is they are people who keep their cards
close to their chest and do not reveal their feelings.
2. Hence, they used a surrogate method- they countered the number of times that the negotiators talked about their
feelings or motives.
3. This contrasts sharply with the amount of information given about external events such as facts, clarifications
and general expressions of opinion.
4. The results showed that contrary to the general impressions, skilled negotiators are more likely to give
information about internal events than are average negotiators.
5. Feelings are in themselves not observable and Huthwaite's researchers could not measure them directly.
(a) 15243 (b) 15432 (c) 12543 (d) 12534

Direction for (Q. 36 – 40) The worst days of any summer are the rainy ones. We spend all year looking forward to
nice weather and long, hot days. All of winter, with its dreary gray days and bitter cold, we dream of those endless
days at the beach, laying on the sand and soaking in the bright and burning sun. And then, summer comes, and it
rains.
As a child, I would wake up to rainy summer days and come close to crying. It wasn’t fair. We suffered through
months of school and miserable weather for those scant ten weeks of freedom and balmy weather. Any day that I
could not spend at the beach or playing ball with my friends seemed like a punishment for something I didn’t even
do.
On those rainy summer days, I had nothing fun to do and could only sit inside, staring out at the rain like a
Dickensian orphan. I was an only child, so there was no one else to play with. My father worked from home, so I
was not truly alone, but he could not actively play with me since he was technically at work. It was those days that I
would resign myself to whatever was on television or any books that I could find lying around. I’d crawl through the
day and pray each night that the rain would not be there the next day.
As an adult, though, my opinion of summer rain has changed. When you have to work every day, summer is not
as eagerly anticipated. Mostly, the days run together, bleeding into each other so that they no longer seem like
separate entities and instead feel like continuations of the same long day. Everything seems monotonous and dull,
and an ennui or listlessness kicks in. Such a mindset makes you cheer for anything new or different. I spend the
winter dreaming of summer and the summer dreaming of winter.
When summer comes, I complain about how hot it is. And then I look forward to the rain, because the rain brings
with it a cold front, which offers a reprieve—admittedly one that is all too short—from the torture of 100° and humid
days. Rainy days are still the worst days of the summer, but summer rain today means positively beautiful—and
considerably cooler—weather tomorrow.
36. The passage makes use of language that is
(a) metaphorical (b) rhetorical (c) formal (d) ambiguous

37. According to the passage, summer is different for adults because


(a) rain brings with it cold temperatures for the following days (b) the weather is much warmer than it is for
children
(c)they do not get a long time off from work for the season (d) they better know how to occupy their
downtime

38. According to the passage, which of the following is a true statement about the narrator as a child?
(a) He or she was often bored on summer days. (b) He or she preferred cooler weather.
(c) He or she liked staying indoors. (d) He or she had no siblings.

39. Compared to how he or she was as a child, the narrator as an adult is


(a) more realistic (b) less excitable (c) more idealistic (d) less calm

40. As used in the final paragraph, the word reprieve most nearly means
(a) a permanent conclusion (b)a short continuation (c) a higher level of pain (d) a temporary break

Pankaj Gandhi’s Academy


rd
3 Floor, Above Hotel Woodland, Near Sharda Center, Nal Stop, Pune. Contact No. : 8600972993, 9850424051.
I English Sample Paper -12
Directions for (Q. 11-20): Fill in the blanks with the appropriate choice
1. Latin America has for long been a forgotten wasteland…………..Indian foreign policy
(a) to the politics controllers (b) to the politics controllers by
(c) to the political controllers by (d) to the politics controllers in

2. …………ahead of the herd.


(a) Intelligence investors make their move (b) Intelligence investors make its move
(c) Intelligent investors make their moves (d) Intelligent investors make their move

3. Hakeem said the Muslims had been overlooked in…………….


(a) earlier peace bids which ended in disaster (b) earliest peace bids which ended in disaster
(c) early peace bids which ended in disaster (d) earlier peace birds which end in disaster

4. The speaker was so ……and elaborate that the audience started getting fidgety
(a) terse (b) prolix (c) curt (d) brief

5. The Asian Games concluded with a colorful………….., in which, in addition to the athletes and players, a number
of artists also participated.
(a) ovation (b) tablet (c) pageant - (d) finale

6. She…...............her talents at the National School of dramatics.


(a) fine-tuned (b) stupefied (c) abraded (d) downgraded

7. This sort of an aggressive attitude would only serve to ………the supporters you already have.
(a) encourage (b) exhort (c) alienate (d) cooperate

8. Focus on solving the refugee problems and drawing plans for …………development was less evident.
(a) short-range (b) sustainable (c) needless (d) merit

9. The groups were feared but at the same time …….by the locals
(a) revered (b) detested (c) rebelled (d) loathed

10. …………………; the logo and the product were repeatedly flashed with jingle that dinned the name into our
heads
(a) There was a time when visual advertising styles was blatantly direct
(b) It was this kind of advertising that paved the way.
(c) There was no escape from the bombardment
(d) Similar products are clubbed together by a brand inspired generic name

11. Hard work --------- with smart preparation can help a long way in passing your exams.
(a) Coupled (b) Attached (c) Adjoined with (d) None

12. The anti riot force was asked not to resort to lathi charge but to -------- restraint till a final command.
(a) Exercise (b) Commit (c) Undertake (d) None

13. -------- of parents is a most important factor when deciding partner in Indian weddings.
(a) Agreement (b) Consent (c) Selection (d) None

(Directions for questions 14-18): In each of the question given below. one or more statement is/are followed by
inferential conclusions. The conclusion, which can be derived without supposing anything else. i.e.. without adding
anything extra to the statement (s). is your answer.
14. Statement:
The Education Secretary said that it was extremely necessary that the professional researchers along with practical
school teachers conduct the practical research.
Conclusions:
(a) Practicing teachers know the technique of research.
(b) Professional researchers are quite aware of the problems related to the school education.
(c) Educational Secretary was speaking to the school students.
(d) There is a communication gap between researchers and the practicing school teachers.

Pankaj Gandhi’s Academy


rd
3 Floor, Above Hotel Woodland, Near Sharda Center, Nal Stop, Pune. Contact No. : 8600972993, 9850424051.
(e) School education is quite a specialized field and so long as teacher is not trained, the standard of education
cannot be qualitatively better.

15. Statement: Vehicular source contributes 60% to the total pollution level of Delhi, as informed in the Rajya
Sabha today.
Conclusions:
(a) The court has ordered that the main source of pollution should be identified.
(b) The problem of pollution is high in the work list.-Answer
(c) In the other cities also, vehicular source contributes around the same percentage level of pollution.
(d) This matter was raised first in the Lok Sabha.
(e) Not many towns in India face the problem of pollution.

16. Statement: PVC shoe producers union has welcomed the announcement of relief on production surcharge from
Rs. 50 per pair to Rs 75 per pair.
Conclusions:
(a) All PVC shoe producers are members of this union.
(b) Relief on production surcharge of Rs 50 on every pair to shoes, had been given only recently.
(c) Union had demanded an increase in relief.
(d) Govt. will collect more production surcharge.
(e) Other producers will also demand relief on production surcharge.

17. Statement:
There are chances of hike in the electricity tariff soon. This proposal will bring about a sharp increase for the
industrial consumers and marginal increase for the domestic consumers.
Conclusions:
(a) At present there is a disparity in the electricity tariff between the domestic and the industrial consumers.
(b) Industrial consumers consume more electricity than the domestic users.
(c) Consumption of electricity by the industrial consumer is comparatively lower than the domestic user.
(d) After this hike, the electricity board will start making profit.
(e) If this proposal is accepted, then industrial output will get positively affected.

18. Statement:
State government imposes a monthly tax on the salary of all the employees.
The tax amount varies according to the income slab.
The Central govt. also imposes tax on the same income, which is known as the income tax.
This is against the theory of taxation.
Conclusions:
(a) A regular collection, irrespective of the income slab, by the Central govt. is the income tax.
(b) The Central govt. should not collect any tax on income.
(c) Income tax should not vary according to the income slabs.
(d) Tax should be imposed only once on an individual.(e) The State govt. should not impose any tax on
income.

Directions for(Q. 19 - 23) Find the error.


19. She wore(1) /a dress to the party that (2) /was more attractive (3) /than the other girls(4). /No error
20. If I would have been there,(1) / I certainly(2)/ would have taken care (3)/of the problem./No error
21. Unless there can be(1)/ some assurance of (2) /increased wages, workers(3) morale all ready low, will
collapse
22. No sooner had he began/(1) to speak when the(2)/ opposition members started(3)/ shouting slogans(4).No
error
23. Each of the hotel’s (a) /150 rooms were equipped with (2)/central air-conditioning (3)/ and colour television.(4)

Directions for (Q. 24-28): Select the pair of words, which do have a relationship similar to the relationship between
the given pair.
24. NOCTURNAL: BAT
(a) amphibian: frog - (b) sly: cat (c) carnivorous: cow (d) aquatic: lizard
25 TETHER: COW
(a) trap: rat (b) manacle: man (c) aerial: radio (d) ring:
finger
26. FOOT: FINGERS
(a) bottom: top (b) finger: thumb (c) pedal: handicap (d) pedicure:

Pankaj Gandhi’s Academy


rd
3 Floor, Above Hotel Woodland, Near Sharda Center, Nal Stop, Pune. Contact No. : 8600972993, 9850424051.
manicure
27. NATURAL: SYNTHETIC
(a) hypothesis: conclusion (b) cotton: wool (c) fragrance: perfume (d) honey: sugarcane
28. MOVING: STATIC
(a) rain: cloud (b) glacier: snow (c) river: alluvium (d) dynamics : electronics
I English Sample Paper - 13
PASSAGE -1
Long considered an intensely subjective and deeply personal experience, dreams are slowly opening out to
research methodology and in the process spilling the secrets of consciousness, volition and the nature of imaginary
worlds. At the absolute cutting edge of dream research today lies the study of an elusive and unusual type of
dream called a 'lucid dream'. Simply put, a lucid dream is a dream in which one knows, at the time of the dream,
that one is dreaming and that one can then control the course of the dream. Although only a small percentage of
people have lucid dreams, there are now methods available which can induce such dreams in people. Keath
Hearne of the University of Hull and Stephen LaBerge of Stanford University, California, realized that if a person
(who was conscious that he or she was dreaming) could somehow communicate to the outside world, it would
open a whole new process of dialogue between the conscious and the unconscious.
Hearne exploited the fact that in dream sleep, the eyes move. He thought perhaps a lucid
dreamer could signal by moving the eyes in a predet ermined pattern and, in 1980, succeeded in making a
volunteer move his eyes left and right eight times in succession. From there it was a simple step to use the
movements in Morse Code to make whole sentences One of the next steps envisaged is to link the Morse signals
to a speech synthesizer for a real conversation to take place. What can we expect to get out of all this? For one
thing, there is a great debate going on at present about the nature of dreaming that could possibly be resolved. An
older school still clings to the Freudian view point of dreams being representations of unconscious with-fulfilment
symbolism while another newer one maintains they are nothing more than random electrochemical bursts which
the brain then does its best to put into some sort of coherent sce nario as visual imagery. If a person could talk
about what he or she is seeing while it is happening, dream interpretation could finally take place with the dreamer
participating in real time. On the other hand, investigators could try to stimulate various areas of the brain through
electrodestosee what, if any, effect it has on the content of dreams while getting simultaneousfeedback from the
dreamer. This could lend supportto their theory. For both sides, though,research right now looks like a dream come
true.
1. The author is unlikely to agree with any of the following except
(a) dream research is chiefly developing strategies for research into the eye movement.
(b) research shows that the dreamer can be made to simu ltaneously participate in the real world.
(c) visual imagery and its relation with Morse Code is indeed very complex.
(d) None of the above

2. The most important aspect of dream research is the way it has made
(a) its link of subconscious with the unconscious. (b) its link of the conscious with the Morse Code.
(c) Its relationship of volition n with the Morse Code (d) None of the above

3. According to the passage, the Freudian concept ofdreams


(a) has now been antiquated. (b) relies heavily on the unfulfilled part of the unconscious.
(c) relies heavily on the wishful thinking of the subconscious. (d) Both 1 and 2

4. The concept of 'lucid dreamer' has been highlighted in the passage because
(a) it shows a wayof analysing dreams simultaneously with the real world.
(b) lucid dreamers are a typical class of dreamers.
(c) dream research required the help of non-lucid dreamers.
(d) real dreamers and lucid dreamers are closely related.

5. The tone of the author can be bestdescribed as


(a) informative. (b) critical. (c) disillusioned. (d) argumentative.

Passage II : There's no escape. You are consuming mass quantitie s of genetically modified food. The milk on your
corn flakes this morning came from a genetically modified cow, and the cornflakes themselves featured genetically
modified whole grain goodness. At lunch you'll enjoy French fries from genetically modified potatoes and perhaps a
bucket of genetically modified fried chicken. If you don't have any meetings this afternoon, maybe you'll wash it all
down with the finest genetically modified hops,grains and barley, brewed to perfection. Everything you eat is the
result of genetic modification. When a rancher in Wyoming selected his stud bull to mate with a certain cow t
o produce the calf that ultimately produced, the milk on your breakfast table, he was manipulating genes. Sounds
delicious, doesn't it? Sorry, but you get the point. Long before you were ever born, farmers were splicing genes,

Pankaj Gandhi’s Academy


rd
3 Floor, Above Hotel Woodland, Near Sharda Center, Nal Stop, Pune. Contact No. : 8600972993, 9850424051.
manipulating seeds to create more robust plants. Genetic modification used to be called "breeding," and people
have
been doing it for centuries. Thomas Jefferson did it at Monticello, as he experimented in his gardens with literally
hundreds of varieties of fruits and vegetables. Anyway, to return to the topic at hand, breeding is n't a scary word,
so people who oppose technology call it "genetic modification." They want to cast biotechnology, which is just a
more precise and effective breeding tool, as some kind of threat to our lives, instead of the blessing that it is. Have
you ever seen corn in its natural state without genetic modification? It's disgusting. We're talking about that nasty,
gnarled, multicoloured garbage used as ornamentation in Thanks giving displays. The fear mongers should eat that
the next time they want to criticise technology. In fact, the fear mongers are waging a very successful campaign
against biotechnology,especially in Europe where they've lobbied to limit the availability of "genetically
modified"foods. Even in the United States, where we generally embrace technology and its possibilities, the fear is
spreading. Not because of some horrible event related to the food supply, but because of more aggressive
spinning of the media. In fact, you've been enjoying foods enhanced by biotechnology for most of the last decade.
And the news is all good - lower prices and more abundant food. As for the future, the potential to eliminate human
suffering is enormous. Right now, according to the World Health Organisation, more than a million kids die every
year because they lack Vitamin A in their diets. Millions more become blind. WHO estimates that more than a
billion people'suffer from anaemia,caused by iron deficiency. What if we could develop rice or corn plants with all of
the essential vitaminsfor children?
Personally, I'd rather have an entire day's nutrition bio-engineered into a Twinkie or a pan pizza, but I recognise the
benefits of more-nutritious crops. Reasonable people can disagree on the best applications for this technology.
Still, the critics want to talk about the dangers of genetically modified crops. The Environmental Protection Agency
wants to regulate the use of certain bio-engineered corn seeds because they include a resistance to pests.
Specifically, the seeds are bred to include a toxin called BT that kills little creatures called corn borers, so farmers
don't need to spray pesticides. Turns out, according to the EPA, that the toxin in the corn can kill Monarch
butterflies, too. The butterflies don't eat corn, but the EPA is afraid t hat the corn pollen will blow over and land on a
milkweed and stick to it and then confused Monarch caterpillars will inadvertently eat the pollen. Not exactly the
end of the world, but it sounds bad Until you consider the alternatives. According to Professor Nina Fedoroff,
Willaman Professor of Life
Sciences at Penn State, "A wide-spectrum pesticide sprayed from a plane is going to kill a lot more insects than will
be killed by an in - plant toxin."Of course, the anti-tech crowd will say that they don't like pesticides either. They
promote organic farming meaning we use more land to produce our food, and we clear more wilderness. We also
paymore for food, since we're not using the efficiencies that. come from technology. Maybe that's not a problem for
you or me, but it's bad news for those millions of malnourished kids around the world. Says Fedoroff, "I think that
most inhabitants of contemporary urban societies don't have a clue about how tough it is to grow enough food for
the human population in competition with bacteria, fungi, insects and animals and in the face of droughts, floods,
and other climatic variations." That may be true, but I do th ink that most Americans understand the positive impact
of technology. And that's why they' ll ultimately reject the scare campaign against biotechnology.
6. Which of the following statements is going to meet with an approval from the author?
(a). Cloning can have disastrous implications but it is okay to clone for medical research.
(b). Genetically modified food is gradually taking atoll on human life.
(c). For a poor nation genetically modified food may be the only source of cheap food.
(d). All technological inventions and achievements should be embraced with open arms.

7. The central idea of the passage is to


(a). propagate the idea that genetically modified food is the only solution to feed the growing population.
(b). dissuade critics against opposing genetically modified food.
(c). make the readers aware of the benefits that can be reaped out of genetically modified food.
(d). make the readers take what opponents of genetically modified foods say with apinch of salt.

8. The passage seems to be culled from


(a) a journal ongenetic engineering. (b) a popular science magazine.
(c) an article in a weekly or a magazine (d) an article in a biology book for high-school students.

9. The assumption that critics make regarding Monarch Butterflies is that


(a) they might eat corn mistaking it for milkweed.
(b) bio-engineering will have a direct impact on Monarch Butterflies.
(c) Monarch caterpillars which feed on milkweed may eat the pollens of corn injected with BT.
(d) None of the above.

10. The nature of the passage can be best described as


(a) Factual (b) narrative (c) Analytical (d) argumentative

Pankaj Gandhi’s Academy


rd
3 Floor, Above Hotel Woodland, Near Sharda Center, Nal Stop, Pune. Contact No. : 8600972993, 9850424051.
Directions for (Q. 11 to 15) : A sentence is written in four different forms. Only one of them is grammatically
correct. Mark the number of the correct sentence as your answer.
11. (a). Rajiv Gandhi was executed while he was addressing a meeting.
(b) Rajiv Gandhi was slaughtered while he was addres sing to a meeting.
(c) Rajiv Gandhi was murdered while he was addressing ina meeting.
(d) Rajiv Gandhi was assassinated while he was addressing a meeting.

12. (a)She was annoyed despite of that she maintained her composure.
(b) She maintained her composure despite of being annoyed.
(c) Despite of being annoyed she maintained her composure.
(4) She maintained her composure despite being annoyed.

13. (a) His employees were asking him for a raise for a long time.
(b) His employees were asking him for a raise from a long time.
(c) His employees have been asking him for a raise for a long time.
(d) His employees were been asking him for a raise for a long time.

14. (a) There are no less than fifty students in the class. (b)There are no fewer than fifty students in the class.
(c)There are not fewer than fifty students in the class. (d) There are not lesser fewer than fifty students in the
class.

15. (a) I request your favour of granting me permission to leave early.


(b) I am requesting your favour in granting me permission to leaveearly.
(c)I request for your favour to grant me permission in leavingearly.
(d) I request the favour of your granting me permission to leave early.

Directions for (Q16 to 18): In the questions below fill in the blanks with the appropriate choices:
16. Thomas Paine, whose political writing was often flamboyant, was in private life a surprisingly __________ man:
he lived in rented rooms, ate little, and wore drab clothes.
(a) Simple (b) controversial (c) Sordid (d) comfortable

17. As long as nationals cannot themselves accumulate enough physical power to dominate all others, they must
depend on their ___________
(a) Allies (b) resources (c) Freedom (d) education

18. While nurturing parents can compensate for adversity, cold or inconsistent parents many it
(a) exacerbate (b) neutralize (c) eradicate (d) ameliorate

Directions for (Q19 to 26):


In each problem below, either part or the entire sentence is underlined. The sentences are followed by five ways of
writing the underlined part. Answer choices [1] repeats the original: the other answer choices vary. If you think that
the original phrasing is the best, choice [1]. If you thing one of the other answer choices is the best, select that
choice.

19. Steve, along with his oldest brothers, are going to make a large real estate investment.
(a) Steve,along with his oldest brothers, are (b) Steve, along with his oldest brothers, is
(c) Steve, in addition to his oldest brothers, are (d) Steve, as well as his oldest brothers, are

20. Many travellers state unequivocally that the streets in Paris are more beautiful than any other city.
(a) that the streets in Paris are more beautiful than any other city.
(b) that the streets in Paris are more beautiful than those in any other city.
(c) that, unlike any other city, Parisian streets are more beautiful
(d) that the streets of Paris are more beautiful than the streets in any other city.

21. Dr. Smith's findings that emotions affect blood pressure are different from those published by his colleague,
Dr.Loeb
(a) affect blood pressure are different from those (b) affect blood pressure are different from that
(c)effect blood pressure are different than those (d) affect blood pressure are different than those

22. Was the letter written with pencil or with ink?

Pankaj Gandhi’s Academy


rd
3 Floor, Above Hotel Woodland, Near Sharda Center, Nal Stop, Pune. Contact No. : 8600972993, 9850424051.
(a) with pencil or with ink (b) with pencil or in ink (c) in pencil or in ink (d) with lead or with pen

23. He has lain his racquetball glove on the beach.


(a) has lain (b) has laid (c) have lain (d) have laid

24. No sooner had he come from Vijayawada when he was asked to proceed to Delhi
(a) had he come from Vijayawada than he was asked to proceed
(b) did he come from Vijayawada when he was asked to proceed
(c) had he come from Vijayawada when he was asked to proceed
(d) when he came from Vijayawada when he was asked to proceed

25. He is one of the shrewdest men that is in the administration.


(a) that is in the administration (b) that the administration is having
(c) that will be in the administration (d) that are in the administration

26. His speech was broadcasted from Delhi.


(a) was broadcast (b) was broadcasted (c) will be broadcasted (d) had been
broadcasted

Directions for (Q 27 to 40):


Fill in the blanks with the appropriate choices
27. When I left school, I saw it as a temporary move. I assumed I'd ________ .
(a) go back (b) return back (c) go forth (d) go and come back

28. For young people today, there are business opportuneities unlike any that have existed ____.
(a) after (b) before (c) now (d) then and there

30. She feared that her show was elitist and perhaps wouldn't reach out _________.
(a) to the rich (b) to the famous masses (c) to the masses (d) to the masseurs

31. He writes faster


(a) than me (b) than I (c) to me (d) to I

32. Unknown to her mother, she kept meeting him on the ________.
(a) sly (b) sty (c) cry (d) sloth

33. We need a good number of ________ judgers.


(a) uninterested (b) disinterested (c) over-interested (d) under-interested

34. Biofeedback training is based on the ____________that we can gain control on our bodily functions.
(a) understanding (b) aspect (c) feeling (d) premise

35. The sharp cracking of a twig mixed with ________ made a great impact on him.
(a) cry (b) shriek (c) squawk (d) scream

I Sample Paper – 14
Direction (Q. 1 to 6): Each of the following questions has a paragraph from which the last sentence has been
deleted .From the given options, choose the one that completes the paragraph in the most appropriate way.
1.Most firms consider expert individuals to be too elitist, temperamental, egocentirc, and difficult to work with. Try to
force such people to collaborate on a high-stakes project and they just might come to fisticuffs. Even the very
notion of managing such a group seems unimaginable. So most organizations fall into default mode, setting up
project teams of people
who get along nicely. ______________
(a) The result, however, is disastrous. (b) The result is mediocrity.
3] The result is creation of experts who then become elitists. (d) Naturally, they drive innovations.

2. The audiences for crosswords and sudoku, understandably, overlap greatly but there are differences, too. A
crossword attracts a more literary person, while sudoku appeals to a keenly logical mind. Some crossword
enthusiasts turn up their noses at Sudoku because they feel it lacks depth. A good crossword requires vocabulary,
knowledge, mental flexibility and sometimes even a sense of humor to complete. It touches numerous areas of life
and provides an "Aha!" or two along the way________________________

Pankaj Gandhi’s Academy


rd
3 Floor, Above Hotel Woodland, Near Sharda Center, Nal Stop, Pune. Contact No. : 8600972993, 9850424051.
(a) Soduku, on the other hand, is just a logical excercise, each one similar to the last.
(b) Sudoku, incidentally, is growing faster in popularity than crosswords. even among theliterati.
(c) Sudoku, on the other hand, can be attempted and enjoyed even bychildren.
(d) Sudoku, however, is not exciting in any sense of the term.

3. Farmers in the North have observed that heavy frost is usually preceded by a full moon. They are convinced that
the full moon somehow generates the frost. Which of the following, if true, would weaken the farmers conviction?
(a) The temperature must fall below 10 degrees Celsius (50 degrees Fahrenheit) for frostto occur.
(b) Absence of a cloud cover cools the ground whichcauses frost
(c) Farmers are superstitious.
(d) No one has proven that the moon causes frost.

4. Professor Tembel told his class that the method of student evaluation of teachers is not a valid measure of
teaching quality. Students should fill out questionnaires at the end of the semester when courses have been
completed. Which of the following, if true, provides support for Professor Tembel's proposal?
(a) Professor Tembel received low ratingsfrom his students.
(b)Students filled out questionnaires after the midterm exam.
(c) Students are interested in teacher evaluation.
(d) Teachers are not obligated to use the survey results.

5. In a famous experiment by Pavlov, when a dog smelled food, it salivated. Subsequently, abell was rung
whenever food was placed near the dog After a number of trials, only the bell was rung, whereupon the dog would
salivate even though no food was present.
Which of the following conclusions may be drawn from the above experiment?
(a) Two stimuli are stronger than one.
(b) Dogs are motivated only by the sound of a bell.
(c) The ringing of a bell was associated with food.
(d) A conclusion cannot be reached on the basis of one experiment.

6. In winning its bitter, protracted battle to acquire Blue Industries, Inc., Belle Industries has fulfilled its goal to
lessen its reliance on tobacco holdings, while the $5.2 billion deal may spur more takeover activity in the insurance
industry, analysts said.
Which of the following can be inferred from the passage?
(a) Blue Industries is in the tobacco industry. (b) Belle Industries is inthe insurance business.
(c) Blue Industries is in the insurance business.
(d) More divestment takes place in the tobacco Industry than in the insurance industry.

Directions for (Q 7 to 14):


Fill in the blanks with the appropriate choices
7. When I left school, I saw it as a temporary move. I assumed I'd ________ .
(a) go back (b) return back (c) go forth (d) go and come back

8. For young people today, there are business opportuneities unlike any that have existed ____ .
(a)after (b) before (c) now (d) then and there

9. She feared that her show was elitist and perhaps wouldn't reach out _________ .
(a) to the rich (b) to the famous masses (b) to the masses (d) to the masseurs

10. He writes faster_____________


(a) than me (b) than I (c) to me (d) to I

11. Unknown to her mother, she kept meeting him on the _________ .
(a) sly (b) sty (c) cry (d) sloth

12. We need a good number of ________ judgers.


(a) uninterested (b) disinterested (c) over-interested (d) under-interested
13. Biofeedback training is based on the ____________that we can gain control on our bodily functions.
(a) understanding (b) aspect (c) feeling (d) premise
14. The sharp cracking of a twig mixed with ________ made a great impact on him.
(a) cry (b) shriek (c) squawk (d) scream

Pankaj Gandhi’s Academy


rd
3 Floor, Above Hotel Woodland, Near Sharda Center, Nal Stop, Pune. Contact No. : 8600972993, 9850424051.

You might also like